Maternal exam notes bulk 2

Ace your homework & exams now with Quizwiz!

2nd period of reactivity

-The newborn is alert and awake -The period lasts 4 to 6 hours in the normal newborn -The heart and respiratory rates increase; however, the nurse must be alert for apneic periods, which may cause a drop in the heart rate and oxygen levels (desaturation). The newborn is stimulated to continue breathing during such times. -May develop rapid color changes and become mildly cyanotic or mottled during these fluctuations. -Production of respiratory and gastric mucus increases, and the newborn responds by gagging, choking, and regurgitating. -GI tract becomes more active -The newborn shows that he or she is ready to be fed by such behaviors as sucking, rooting, and swallowing. -If feeding was not initiated during the first period of reactivity, it should be done during this time. -Continued close observation and intervention may be required to maintain a clear airway during this period of reactivity.

Psychologic Adaption Assessment Postpartum

-During first 24 hours: passive; preoccupied with own needs; may talk about her labor and birth experience; may be talkative, elated, or very quiet -Very quiet and passive; sleeps frequently - fatigue from long labor, feelings of disappointment about some aspect of the experience, maybe following cultural expectation -Usually by 12 hours: beginning to assume responsibility; some women eager to learn; easily feel overwhelmed -Excessive weepiness, mood swings, pronounced irritability - postpartum blues, feelings of inadequacy, culturally prescribed behavior

Attachment Assessment Postpartum

-En face position: holds baby close; cuddles and soothes; calls by name; identifies characteristics of family members in infants; may be awkward in providing care -Continued expressions of disappointment in sex, appearance of infant; refusal to care for infant; derogatory comments; lack of bonding behaviors - difficulty in attachment, following expectations of cultural/ethnic group -Initially may express disappointment over sex of infant but within 1-2 days demonstrates attachment behaviors.

Nursing care for the active phase of labor

-Encourage woman to maintain breathing patterns -Provide quiet environment -Provide reassurance, encouragement, support -Keep couple informed of progress -Promote comfort by giving backrubs -Sacral pressure -Cool cloth on forehead -Assistance with position changes -Support with pillows -Effleurage -More ice chips -Ointment for dry mouth and lips -Encourage to void every 1 to 2 hours -Offer whirlpool warm bath if available

Nursing care for the transition phase of labor

-Encourage woman to rest between contractions. If she sleeps between contractions wake her at the beginning so she can do breathing pattern. -Provide support, encouragement, and praise for efforts. -Keep couple informed of progress. -Encourage continued participation of support persons -Promote comfort but recognize many woman do not want to be touched when in transition -Provide privacy, ice chips, ointment for lips -Encourage voiding

Chemical Changes

-Important chemical stimulator that contributes to the onset of breathing is transitory asphyxia of the fetus and newborn. -The first breath is an inspiratory gasp, the result of central nervous system reaction to sudden pressure, temperature change, and other external stimuli. -The first breath is triggered by normal chest recoil as well as the chemical factors of an elevation in PCO2 (Hypercapnia), decrease in pH (acidosis), and decrease in PO2 (hypoxia), which are the natural results of normal vaginal labor and birth and cessation of placental gas exchange when the cord is clamped.

Physical changes postpartum: UT

-Increased bladder capacity, swelling and bruising of the tissues around the uretha, -Decreased sensitivity to fluid pressure, and decreased sensation of bladder filling. -She is at risk for overdistention, incomplete emptying, and buildup of residual urine. -Immediate postpartal use of oxytocin to faciliate uterine contractions following expulsion of the placenta has an antidiuretic effect. -Increased output after 12 hours -Returns to normal at 6 weeks

Factors contributing to atony

-Over distention of the uterus due to multiple gestation polyhydramnios or a large infant -Dysfunctional or prolonged labor, which indicates that the uterus is contracting abnormally -Oxytocin augmentation or induction of labor -Grandmultiparity, because stretched uterine musculature contracts less vigorously -Use of anesthesia or other drugs such as magnesium sulfate, calcium channel blockers which all cause the uterus to relax. -Prolonged third stage of labor - more than 30 minutes -Preeclampsia -Asian or Hispanic heritage -Operative birth (includes a vacuum extraction or forceps assisted births) -Retained placental fragments -Placental previa

5 P's of Labor

-Passageway -Passenger -Physiologic forces of labor -Position of the mother -Psychological considerations

Nursing care for Sepsis

-Prevention of infection -Starts prenatally and continues throughout pregnancy and birth -Should include prenatal screening for STIs and monitoring for rubella titers in women who test negative -Intrapartally, sterile technique is essential -Local eye treatment is given to all newborns to prevent damage from gonorrhea and Chlamydia

Nursing Response to Psychologic adaption assessment postpartum

-Provide opportunities for adequate rest; provide physical comfort measures; provide nutritious meals and snacks that are consisten with what the woman desires to eat and drink; provide opportunities to discuss birth experience in nonjudgmental atmosphere if the women desires to. -Explain PP blues; provide supportive atmosphere; determine support available for mother; consider referral for evidence of profound depression

Nursing response to attachment assessment postpartum

-Provide reinforcement and support for incant care-taking behaviors; maintain nonjudgmental approach and gather more information if care-taking behaviors are not evident.

Nursing Response to Cultural Assessment Postpartum

-Provide specific request if possible -If woman is unable to provide specific information, the nurse may draw from general information regarding cultural variation -Mexican women may want food and fluids that restore hot-cold balance to the body. Women of European background may ask for iced fluids. -Proved privacy as desired by mother.

REEDA

-R - redness -E - edema or swelling -E - ecchymosis or bruising -D - drainage -A - approximation ( how well the edges of an incision seem to be holding together)

Lochia rubra

-Red, bloody discharge -Last from birth to 3 days -See physician if still have bloody discharge at 4 days -Should be feeling better/less bleeding each day

Fetal attitude

-Refers to the relation of the fetal body parts to one another -Describes the posture the fetus assumes as it conforms to the shape of the uterine cavity.

Station

-Refers to the relationship of the presenting part to an imaginary line drawn between the ischial spines of the maternal pelvis

FHR variability

-Reliable indicator of cardiac and CNS -Reflects an intact nervous system -Measure of fetal oxygen in tissue -Decreases when baby is in sleeping state

Nursing care for hypoglycemia

-Routine screening for all at risk infants -Early feedings -D10W infusion

Nursing care for late decelerations

-STOP pitocin! -Elevate legs of mother -Increase her IV -Put her on Oxygen

Assessment of Lochia Postpartum

-Scant to moderate amount, earthy odor; no clots -Large amount, clots - hemorrhage -Foul smelling lochia - infection

Rubin phases

-Taking in: occurs 1-2 days. Trying to recover from labor and delivery. Focused on needs. Woman tends to be passive and somewhat dependent. Food and sleep are major needs. Taking Hold: 2-3 day the new mother was observed to be ready to resume control of her body, her mothering, and her life in general. If the baby spits up she might consider this a failure. She requires assurance that she is doing good -Letting go: 10 days to 6 weeks. Adjusting to new role.

Self-quieting

-The ability of the newborns to use their own resources to quiet and comfort themselves. -Their repertoire includes hand-to-mouth movements, sucking on a fist or tongue, and attending to external stimuli.

Expulsion

-The anterior shoulder meets the undersurface of the symphysis pubis and slips under it. -As lateral flexion of the shoulder and head occurs, the anterior shoulder is born before the posterior shoulder. -The body follows quickly.

Lightening

-The fetus head moves down in the pelvis -Primigravida experiences this in the last few weeks of pregnancy which will give relief to her shortness of breath -Multigravida will not experience this until labor so she will still experience shortness of breath until labor.

Visualization of the fetus with an ultrasound

-The gestational sac can be observed by 4 to 5 weeks gestation (2 to 3 weeks after conception) -Fetal parts and fetal heart movement can be seen as early as 8 weeks

Period of Inactivity to Sleep Phase

-The heart rate and respirations decrease as the newborn enters the sleep phase. -May last from a few minutes to 2 to 4 hours. -The newborn may be difficult to awaken and will show no interest in sucking. -Bowel sounds become audible, and cardiac and respiratory rates return to baseline values

4) A nurse in a newborn nursery is performing an assessment of a newborn infant. The nurse is preparing to measure the head circumference of the infant. The nurse would most appropriately: 1. Wrap the tape measure around the infant's head and measure just above the eyebrows. 2. Place the tape measure under the infants head at the base of the skull and wrap around to the front just above the eyes 3. Place the tape measure under the infants head, wrap around the occiput, and measure just above the eyes 4. Place the tape measure at the back of the infant's head, wrap around across the ears, and measure across the infant's mouth.

1. 3. To measure the head circumference, the nurse should place the tape measure under the infant's head, wrap the tape around the occiput, and measure just above the eyebrows so that the largest area of the occiput is included. (The 4th option was pretty damn funny though.)

A newborn has small, whitish, pinpoint spots over the nose, which the nurse knows are caused by retained sebaceous secretions. When charting this observation, the nurse identifies it as: 1. Milia 2. Lanugo 3. Whiteheads 4. Mongolian spots

1. 1. Milia occur commonly, are not indicative of any illness, and eventually disappear.

13) The primary critical observation for Apgar scoring is the: 1. Heart rate 2. Respiratory rate 3. Presence of meconium 4. Evaluation of the Moro reflex

1. 1. The heart rate is vital for life and is the most critical observation in Apgar scoring. Respiratory effect rather than rate is included in the Apgar score; the rate is very erratic.

A nurse in a newborn nursery receives a phone call to prepare for the admission of a 43-week-gestation newborn with Apgar scores of 1 and 4. In planning for the admission of this infant, the nurse's highest priority should be to: 1. Connect the resuscitation bag to the oxygen outlet 2. Turn on the apnea and cardiorespiratory monitors 3. Set up the intravenous line with 5% dextrose in water 4. Set the radiant warmer control temperature at 36.5* C (97.6*F)

1. 1. The highest priority on admission to the nursery for a newborn with low Apgar scores is airway, which would involve preparing respiratory resuscitation equipment. The other options are also important, although they are of lower priority.

12) A baby is born precipitously in the ER. The nurses initial action should be to: 1. Establish an airway for the baby. 2. Ascertain the condition of the fundus 3. Quickly tie and cut the umbilical cord 4. Move mother and baby to the birthing unit

1. 1. The nurse should position the baby with head lower than chest and rub the infant's back to stimulate crying to promote oxygenation. There is no haste in cutting the cord.

While assessing a 2-hour old neonate, the nurse observes the neonate to have acrocyanosis. Which of the following nursing actions should be performed initially? 1. Activate the code blue or emergency system. 2. Do nothing because acrocyanosis is normal in the neonate 3. Immediately take the newborn's temperature according to hospital policy 4. Notify the physician of the need for a cardiac consult

1. 2. Acrocyanosis, or bluish discoloration of the hands and feet in the neonate (also called peripheral cyanosis), is a normal finding and shouldn't last more than 24 hours after birth.

By keeping the nursery temperature warm and wrapping the neonate in blankets, the nurse is preventing which type of heat loss? 1. Conduction 2. Convection 3. Evaporation 4. Radiation

1. 2. Convection heat loss is the flow of heat from the body surface to the cooler air.

The expected respiratory rate of a neonate within 3 minutes of birth may be as high as: 1. 50 2. 60 3. 80 4. 100

1. 2. The respiratory rate is associated with activity and can be as rapid as 60 breaths per minute; over 60 breaths per minute are considered tachypneic in the infant.

) When teaching umbilical cord care to a new mother, the nurse would include which information? 1. Apply peroxide to the cord with each diaper change 2. Cover the cord with petroleum jelly after bathing 3. Keep the cord dry and open to air 4. Wash the cord with soap and water each day during a tub bath

1. 3. Keeping the cord dry and open to air helps reduce infection and hastens drying.

After reviewing the client's maternal history of magnesium sulfate during labor, which condition would the nurse anticipate as a potential problem in the neonate? 1. Hypoglycemia 2. Jitteriness 3. Respiratory depression 4. Tachycardia

1. 3. Magnesium sulfate crosses the placenta and adverse neonatal effects are respiratory depression, hypotonia, and Bradycardia.

A client has just given birth at 42 weeks' gestation. When assessing the neonate, which physical finding is expected? 1. A sleepy, lethargic baby 2. Lanugo covering the body 3. Desquamation of the epidermis 4. Vernix caseosa covering the body

1. 3. Postdate fetuses lose the vernix caseosa, and the epidermis may become desquamated. These neonates are usually very alert. Lanugo is missing in the postdate neonate.

) A nurse on the newborn nursery floor is caring for a neonate. On assessment the infant is exhibiting signs of cyanosis, tachypnea, nasal flaring, and grunting. Respiratory distress syndrome is diagnosed, and the physician prescribes surfactant replacement therapy. The nurse would prepare to administer this therapy by: 1. Subcutaneous injection 2. Intravenous injection 3. Instillation of the preparation into the lungs through an endotracheal tube 4. Intramuscular injection

1. 3. The aim of therapy in RDS is to support the disease until the disease runs its course with the subsequent development of surfactant. The infant may benefit from surfactant replacement therapy. In surfactant replacement, an exogenous surfactant preparation is instilled into the lungs through an endotracheal tube.

A client has just given birth at 42 weeks gestation. When assessing the neontate, which phsycial finding is expected? 1. a sleepy, lethargic baby 2. lanugo covering body 3. desquamation of the epidermis 4. vernix caseosa covering the body

3. desquamation of the epidermis postdate fetuses lose the vernix caseosa and the epidermis may become desquamated these neonates are usually VERY alert lanugo is missing in the postdate neonate

Which nursing ntervention helps prevent evaporative heat loss in the neonate immed after birth? 1. admin warm oxygen 2. controlling drafts in the room 3. immed drying the neonate 4. placing neonate on a warm, dry towel

3. immed drying the neonate decreases evaoporative heat loss from moist body from birth placing the neonate on a warm, dry towel decreases CONDUCTIVE losses. controlling draft in the room and admin warm oxygen helps reduce CONVECTIVE LOSS

3 day old neonate needs phototherapy for hyperbilirubinemia. nursery care of a neonate getting phototherapy would include which nursing intervention? 1. tube feedings 2. feeding the neonate under phototheraphy lights 3. mask over the eyes to prevent retinal damage 4. temp monitored every 6 hours during phototherapy

3. mask the neonate's eyes must be covered with eye patches to prevent damage the mouth of the neonate doesnt need to be covered the neonate can be removed from the lights and held for feeding the neonates temp should be monitored at least every 2-4 hours due to risk of hyperthermia w/ phototherpahy

Which of the following conditions isnt dx by abdominal US during the prenatal period? 1. fetal presentation 2. fetal heart activity 3. maternal diabetes 4. amniotic fluid volume

3. maternal diabetes abdominal US evals fetal presentation, fetal heart activity, amniotic fluid volume although it may show increased amnitoic flud, thus helping to diagnose maternal diabetes, it isnt used for that purpose.

Which of the following complications is possible with an episiotomy? 1. blood loss 2. uterine disfigurement 3. prolonged dyspareunia 4. hormonal flucuation postpartum

3. prolonged dyspareunia - painful intercourse may result when complications such as infection interefere with wound healing. minimal blood loss occurs when an episitomy is done the uterus isnt affected bc its the perineum that's cut to accomodate the fetus hormonal fluctuations that occur during the postpartum period arent the result of an episiotomy

A woman who's 36 week preg comes into L&D with mild contractions. Which of the following complications should the nurse watch for when the client informs her that she has placenta previa? 1. sudden ROM 2. Vaginal bleeding 3. emesis 4. fever

2. Vaginal bleeding contractions may disrupt the microvascular network in the placenta of the client with placenta previa and result in bleeding. if the separation of the placenta occurs at the margin of the placenta, the blood will escape vaginally. sudden ROM isnt r/t placenta previa fever would indicate an infections process, and emesis isnt r/t placenta previa

A client at 33 weeks gestation and leaking amniotic fluid is place on an EFM. The monitor indicates uterine irritability and contractions occuring every 4-6 min. The doctor orders terbutaline. Which of the following teaching statements is approp for this client? 1. This medicine will make you breathe better 2. You may feel fluttering or tight sensation in your chest 3. This will dry your moth and make you thirsty 4. You'll need to replace potassium lost by this drug

2. You may feel fluttering or tight sensation in your chest A fluttering or tight sensation in the chest is a common adverse reaction to terbutaline It relives bronchospasm but the client is getting it to reduce uterine motility. Mouth dryness and thirst occur w/ the inhaled form but are unlikely with subcut form Hypokalemia is a potential adverse reaction following large doses of terbutaline but not at doses of 0.25 mg

The phrase gravida 4, para 2 indicated which of the following prenatal histories? 1. a client has been pregnant 4 times and had 2 miscarriages. 2. a client has been pregnant 4 times and had 2 live born children 3. a client has been pregnant 4 times and had 2 c-sections 4. a client has been pregnant 4 times and 2 spontaneous abortions.

2. a client has been pregnant 4 times and had 2 live born children Gravida refers to the number of times a client has been pregnant. Para refers to the # of viable children born. Therefore, the client who's gravida 4, para2 has been pregnant 4x and had 2 live born children.

Lochia alba follows lochia serosa and usually lasts from the 1st to 3rd week PP. Which of the following statements best describes lochia alba? 1. creamy white-brown, stale odor 2. creamy white to brown, contains decidual cells, may have stale odor 3. brown to red, tissue fragments, odor 4. brown to red contains decidual cells and leukocytes

2. creamy white to brown, contains decidual cells, may have stale odor also contais leukocytes but it shouldnt contain tissue fragments or have a foul odor

Before the placenta functions, the corpus luteum is the primary source for synthesis of which of the following hormones? 1. cortisol and thyroxine 2. estrogen and progesterone 3. LH and FSH 4. T4 and T3

2. estrogen and progesterone The CL produces progesterone and estrogen for the 1st 8-10 weeks of pregnancy until the placenta takes over this function. The high levels of estrogen and progesterone cause suppression of LH and FSH. T4 and T3 are produced in the adrenal gland

The most common neonatal sepsis and meningitis infections seen within 24 hours after birth are caused by which organism? 1. Candida albicans 2. Chlamydia trachomatis 3. Escherichia coli 4. Group B beta-hemolytic streptococci

1. 4. transmission of Group B beta-hemolytic streptococci to the fetus results in respiratory distress that can rapidly lead to septic shock.

A client, 34 weeks pregnant, arrives at the ER with SEVERE abdominal pain, uterine tenderness and an increased uterine tone. The client denies vaginal bleeding. The external fetal monitor shows fetal distress with severe, variable decels. The client most likely has which of the following?

1. Abruptio placentae a client w/ severe abruptio placentae will often have SEVERE abdominal pain. The uterus will have increased tone w/ little to no return to resting tone btw/ contractions. The fetus will start to show signs of distress, with decels in the HR or even fetal death w/ large placental separation. Placenta previa usually involves PAINLESS vaginal bleeding w/out UCs. A molar preg. generally would be detected before 34 weeks gestation. An ecoptic preg. which usually occurs in the FALLOPIAN TUBES, would rupture well before 34 weeks gestation

A woman with preeclampsia has a seizure. The nurse's duties include which of the following interventions? Select all the apply

1. Ensure an open airway 2. Turn her to her side to prevent aspiration 3. Stay with the client and call for help

Which of the following complications can be potentially life threatening and can occur in a client receiving a tocolytic agent? 1. diabetic ketoacidosis 2. hyperemesis gravidarum 3. pulmonary edema 4. sickle cell anemia

3. pulmonary edema Tocolytics are used to stop labor contractions. The most common adverse effect associated with the use of these drugs is pulmonary edema. Clients who dont have diabetes dont need to be observed for diabetic ketoacidosis. Hyperemesis gravidium doesnt result from tocolytic use. Sickle cell anemia is an inherited genetic condition and doesnt develop spontaneously

Which of the following rationales best explains why a pregnant client should lie on her left side when resting or sleeping in the later stages of pregnancy? 1. to facilitate digestion 2. to facilitate bladder emptying 3. to prevent compression of vena cava 4. to avoid fetal anomalies

3. to prevent compression of vena cava the weight of the preg uterus is sufficiently heavy to compress the vena cava, which could impair blood flow to the uterus, possibily decreasing oxygen to the fetus. The side lying position hasnt been shown to prevent fetal anolmaies nor bladder emptying and or digestion

A woman with a term, uncomplicated pregnancy comes into L&D in early labor saying that she thinks her water broke. Which action should the nurse take? 1. prep the woman for delivery 2. note color, amt and odor of fluid 3. immed contact doctor 4. collect sample of fluid for microbial analysis

2. Noting color, amount and odor of the fluid as well as the time of the rupture, will help guide the nurse in her next action. There's no need to call the doctor immed or prep the client for delivery if the fluid is clear and delivery isnt imminent. ROM isnt unusual in early stages of labor. Fluid collection for microbe analysis isnt routine and theres no concern for infection/maternal fever.

A male neonate has just been circumcised. Which nursing intervention is part of the initial care of a circumcised neonate? 1. apply alcohol to the site 2. Change the diaper as needed 3. keep the neonate in supine position 4. apply petroleum gauze to the site for 24 hours

4. Petroleum gauze is applied to the site for the 1st 24 hours to prevent the skin edges from sticking to the diaper neonates are initially kept in the prone position diapers are changed more freq to insepct the site alcohola is CONTRAINDICATED

A 21y.o. client has been diagnosed with hydatidiform mole. Which of the following factors is considered a risk factor for developing hydatidiform mole? 1. age in 20s or 30s 2. high in SES 3. Primigravida 4. prior molar gestation

4. previous molar gestation increases risk for developing subsequent molar gestation by 4-5 times. Adolescents and women ages 40+ are at increased risk for molar pregs. MULTIGRAVIDAS, esp women with prior preg loss, and women with LOWER SES are at increased risk for this problem.

Which of the following factors would contribute to a high risk pregnancy? 1. Blood type O positive 2. first pregnancy at age 33y.o. 3. Hx of allergy to honey bee pollen 4. Hx of insulin dependent DM

4. a woman w/ a hx of diabetes has an increased risk for perinatal complications, including HTN, preeclampsia, and neonatal hypoglycemia. The age of 33 years w/out other risk factors doesn't increase risk, nor does type O positive blood or environmental allergens.

A client in early labor states that she has a thick yellow discharge from both of her breasts. Which of the following actions by the nurse would be correct? 1. tell her that her milk is starting to come in bc she's in labor 2. complete a thorough breast exam and document the results in the chart. 3. perform a culture on the discharge and inform the client that she might have mastitis 4. inform the client that the discharge is colostrum, normally present after the 4th month of preg.

4. after the 4th month, colostrum may be expressed. the breasts normally produce colostrum for the first few days after delivery. milk production begins 1-3 days PP A clinical breast exam isnt indicated in intrapartum although a culture may be indicated, it requires advanced assessment as well as a medical order

A neonate has been diagnosed with caput succedaneum. which statement is true? 1. usually resolves in 3-6 weeks 2. collection of blood btw/ skill and periosteum 3. doesnt cross the cranial suture line 4. it involves swelling of the tissue over the presenting part of the head

4. it involves swelling of the tissue over the presenting part of the head due to sustained pressure this boggy edematous swelling is present at birth, CROSSES the suture line occurs in occipital area a cephalohematoma is a collection of blood btw/ the skill and periosteum that DOESNT cross the suture lines and resolves in 3-6 weeks caput seccedaneum resolves within 3-4 DAYS

Which of the following answers best describes the stage of pregnancy in which maternal and fetal blood are exchanged? 1. conception 2. 9 weeks gestation when the fetal heart is well developed 3. 32-34 weeks gestation 3rd Trimester 4. maternal and fetal blood are never exchanged

4. maternal and fetal blood are never exchanged only nutrients and waste products are transferred across the placenta. blood exchange never occurs complications and some medical procedures can cause an exchange to occur accidentally

The SGA neonate is at increased risk during the transitional period for which complication? 1. anemia probably due to chronic fetal hypoxia 2. hyperthermia due to decreased glycogen stores 3. hyperglycemia due to decreased glycogen stores 4. polycythemia probably due to chronic fetal hypoxia

4. polycythemia probably due to chronic fetal hypoxia the SGA baby is at risk for developing polycythemia during the transitional period in an attempt to decrease hypoxia the neonates are also at increased risk for developing hypoglycemia due to decreased glycogen stores

Which Sx would indicate the neonate was adapting approp to extrauterine life w/out difficulty? 1. nasal flare 2. light audible grunting 3. resp rate 40-60 breaths/min 4. resp rate 60-80 breaths/min

4. resp rate of 40-60 breaths/min is normal for a neonate during the transitional period nasal flaring, resp rate > 60 and audible grunting = SIGNS OF RESP DISTRESS!

At 1 min after birth the nurse assesses the infant and notes: a HR of 80 beats/min, some flexion of extremities, a weak cry, grimacing, and a pink body but blue extremities. The nurse would calculate and Apgar score of:

5

In most healthy newborns, blood glucose levels stabilize at _________ mg/dL during the first hours after birth:

50-60 In most healthy term newborns, blood glucose levels stabilize at 50 to 60 mg/dL during the first several hours after birth. A blood sugar level less than 40 mg/dL in the newborn is considered abnormal and warrants intervention. This infant can display classic symptoms of jitteriness, lethargy, apnea, feeding problems, or seizures. By the third day of life, the blood glucose levels should be approximately 60 to 70 mg/dL.

The nurse should know that once human immunodeficiency virus (HIV) enters the body, seroconversion to HIV positivity usually occurs within:

6 to 8 weeks.

At 1 minute following birth, the newborn exhibited the following: heart rate of 155; loud, vigorous crying with active movement of all extremities; sneezing when nose is stimulated with a catheter; hands and feet bluish and cool to the touch. The Apgar score of this newborn should be recorded as________.

9

What PPH conditions are considered medical emergencies that require immediate treatment? A. Inversion of the uterus and hypovolemic shock B. Hypotonic uterus and coagulopathies C. Subinvolution of the uterus and idiopathic thrombocytopenic purpura D. Uterine atony and disseminated intravascular coagulation

A A. Correct: Inversion of the uterus and hypovolemic shock are considered medical emergencies. B. Incorrect: Although this is a serious condition, it is not necessarily a medical emergency that requires immediate treatment. C. Incorrect: Although this is a serious condition, it is not necessarily a medical emergency that requires immediate treatment. D. Incorrect: Although this is a serious condition, it is not necessarily a medical emergency that requires immediate treatment. pp. 980, 983

The perinatal nurse caring for the postpartum woman understands that late postpartum hemorrhage is most likely caused by: A. Subinvolution of the placental site B. Defective vascularity of the decidua C. Cervical lacerations D. Coagulation disorders

A A. Correct: Late PPH may be the result of subinvolution of the uterus, pelvic infection, or retained placental fragments. B. Incorrect: Although this condition may cause PPH, late PPH typically results from subinvolution of the uterus, pelvic infection, or retained placental fragments. C. Incorrect: Although this condition may cause PPH, late PPH typically results from subinvolution of the uterus, pelvic infection, or retained placental fragments. D. Incorrect: Although this condition may cause PPH, late PPH typically results from subinvolution of the uterus, pelvic infection, or retained placental fragments. p. 977

In caring for the woman with disseminated intravascular coagulation (DIC), what order should the nurse anticipate? A. Administration of blood B. Preparation of the client for invasive hemodynamic monitoring C. Restriction of intravascular fluids D. Administration of steroids

A A. Correct: Primary medical management in all cases of DIC involves correction of the underlying cause, volume replacement, blood component therapy, optimization of oxygenation and perfusion status, and continued reassessment of laboratory parameters. B. Incorrect: Central monitoring would not be ordered initially in a client with DIC, because this can contribute to more areas of bleeding. C. Incorrect: Management of DIC would include volume replacement, not volume restriction. D. Incorrect: Steroids are not indicated for the management of DIC. p. 985

What would be a warning sign of ineffective adaptation to extrauterine life if noted when assessing a 24-hour-old breastfed newborn before discharge? A Apical heart rate of 90 beats/min, slightly irregular, when awake and active B Acrocyanosis C Harlequin color sign D Weight loss representing 5% of the newborn's birth weight

A The heart rate of a newborn should range from 120 to 140 beats/min, especially when active. The rate should be regular with sharp, strong sounds. Acrocyanosis is a normal finding in a newborn at 24 hours of age. A harlequin sign is a normal finding related to the immature neurologic system of a newborn. A 5% weight loss is acceptable in the newborn.

Fetal Bradycardia

A FHR baseline less than 110 bpm for at least a 10 minute period

The nurse is caring for a patient in augmented labor. The nurse recognizes that the Oxytocin should be discontinued immediately if there is evidence of which?

A FHR of 180 with absence of variability

The microscopic examination of scrapings from the cervix, endocervix, or other mucous membranes to detect premalignant or malignant cells is called:

A Papanicolaou (Pap) test.

You encourage a pregnant woman to eat a diet high in complete protein. Assuming she likes all the following foods, which of them would you recommend?

A boiled egg

A primigravida is being monitored in her prenatal clinic for preeclampsia, which finding should concern her nurse

A dipstick value of 3+ for protein in her urine

Which statement about family systems theory is inaccurate?

A family as a whole is equal to the sum of the individual members.

A newly graduated nurse is attempting to understand the reason for increasing health care spending in the United States. Her research finds that these costs are much higher compared with other developed countries as a result of:

A higher rate of obesity among pregnant women.

A woman had unprotected intercourse 36 hours ago and is concerned that she may become pregnant because it is her "fertile" time. She asks the nurse about emergency contraception. The nurse tells her that:

An over-the-counter antiemetic can be taken 1 hour before each contraceptive dose to prevent nausea and vomiting.

The doctor suspects that the client has an ectopic pregnancy. Which symptom is consistent with a diagnosis of ectopic pregnancy? a. Painless vaginal bleeding b. Abdominal cramping c. Throbbing pain in the upper quadrant d. Sudden, stabbing pain in the lower quadrant

Answer D is correct. The signs of an ectopic pregnancy are vague until the fallopian tube ruptures. The client will complain of sudden, stabbing pain in the lower quadrant that radiates down the leg or up into the chest. Painless vaginal bleeding is a sign of placenta previa, abdominal cramping is a sign of labor, and throbbing pain in the upper quadrant is not a sign of an ectopic pregnancy, making answers A, B, and C incorrect.

Which disorder seen in newborns consists of right ventricular hypertrophy, stenosis of the pulmonary artery, ventricular septal defect, and overriding of the aorta? A. Tetralogy of Fallot B. Atrial septal defect C. Coarctation of the aorta D. Ventricular septal defect

Answer: A Rationale: Tetralogy of Fallot is a major heart defect resulting in cyanosis at birth. The classic cyanotic symptoms result from these four congenital defects: right ventricular hypertrophy, stenosis of the pulmonary artery, ventricular septal defect, and overriding of the aorta. Atrial septal defect is an abnormal opening between the right and the left atria with a left-to-right shunting of blood. Coarctation of the aorta is a localized narrowing of the aorta causing increased pressure proximal to the defect (head and neck). There is decreased pressure distal to the defect, which is the body and lower extremities. Ventricular septal defect is an abnormal opening between the right and left ventricles. High pressure in the left ventricle causes blood to shunt from the left to the right ventricle.

As part of their teaching function at discharge, nurses should tell parents that the baby's respiratory status should be protected by the following procedures: (Select all that apply.) A Prevent exposure to people with upper respiratory tract infections B Keep the infant away from secondhand smoke C Avoid loose bedding, waterbeds, and beanbag chairs D Do not let the infant sleep on his or her back E Keep a bulb suction available at home.

A, B, C, E Infants are vulnerable to respiratory infections; infected people must be kept away. Secondhand smoke can damage lungs. Infants can suffocate in loose bedding and furniture that can trap them. The infant should be laid down to sleep on his or her back for better breathing and to prevent sudden infant death syndrome. A bulb syringe will be useful if the baby needs suctioning of the mouth and nose at home to protect the airway.

A newborn infant is diagnosed with a patent ductus arteriosus (PDA). The nurse is aware that this is indicative of a defect that: A. typically results in cyanosis B. may result in congestive heart failure C. also causes pulmonary stenosis D. normally does not close after birth

Answer: B Rationale: Defects that result in increased pulmonary blood flow such as patent ductus arteriosus (PDA) and other atrial and ventricular septal defects may cause congestive heart failure. PDA is a vascular connection that during fetal life bypasses the pulmonary vascular bed and directs blood from the pulmonary artery to the aorta. Defects that involve decreased pulmonary blood (such as tetralogy of Fallot) or obstruction to blood flow out of the heart (such as pulmonary stenosis) typically result in cyanosis. PDA does not cause pulmonary stenosis. A PDA normally closes soon after birth. If the ductus does remain open after birth, the direction of blood flow in the ductus is reversed by the higher pressure in the aorta, so there may not be any signs of the disorder.

A nurse assesses a newborn with asymmetric gluteal and thigh skinfolds, a left leg shorter than the right, and a clicking sound of the right hip. What condition does this information indicate that the newborn most likely has? A. fractured pelvis B. fractured right leg C. congenital hip dysplasia D. underdeveloped femur

Answer: C Rationale: With congenital hip dysplasia, the hip is not correctly situated or rotating in the socket. The affected leg will appear longer, and a clicking sound can be heard when the leg is moved. The symptoms listed do not describe a fracture pelvis or leg; however a newborn could sustain a fracture as a result of a traumatic birth. An underdeveloped femur is not a newborn condition.

A 48-hour-old infant who is being breast-fed is diagnosed with physiological jaundice and is prescribed phototherapy treatment. Which measure taken by the nurse would enhance bilirubin excretion? A. keeping the infant snugly wrapped B. placing the infant in a quite, darkened area C. providing the infant with additional oral fluids every 3 hours D. encouraging the mother to temporarily suspend breast-feeding her infant

Answer: C Rationale: Phototherapy can cause insensible water loss, thus it is important to assess for dehydration and provide fluids. Proper fluid balance will promote bilirubin excretion. Keeping the infant wrapped in the dark and suspending breast-feedings will not enhance the excretion of bilirubin.

Which observation in a 24-hour-old newbon should be reported to the physician immediately? A. jaundice B. positive Babinski reflex C. heart rate of 130/bpm D. High-pitched crying and arching of the back

Answer: D Rationale: A high-pitched cry and arching of the back (opisthotonos) are cardinal signs of a neurological abnormality. Physiological jaundice occurs in 50% of term infants after the first 24 hours. A serum bilirubin level should be drawn to determine if treatment with phototherapy is necessary. A positive babinski reflex is a normal response in newborns. It is characterized by all the toes hyperextending with dorsiflexion of the big toe. A heart rate of 130 bpm falls within the range.

A nurse should observe for which particular complication in infants who are born breech? A. cyanosis B. fracture hip C. hydrocephalus D. brachial palsy

Answer: D Rationale: Brachial palsy results from the stretching of nerve fibers in the neck, shoulder, and arm when the shoulder is being pulled away from the neck during breech delivery. Cyanosis, fractured hip, and hydrocephalus are not directly associated with breech deliveries.

Compared with an infant born vaginally an infant born via cesarean section is more likely to manifest which condition? A. crib death syndrome B. neurological deficits C. failure to thrive syndrome D. Respiratory distress syndrome

Answer: D Rationale: Research has shown that respiratory distress syndrome (RDS) is more common in infants born by cesarean birth without labor than in those born vaginally. The other answer options A, B, and C are not associated with cesarean births

A nurse caring for the newborn of a diabetic mother would give highest priority to assessing for which conditions? A. Hypocapnia B. Hyponatremia C. Hyperglycemia D. Hyperinsulinism

Answer: D Rationale: The infant of a diabetic mother has become accustomed to high levels of glucose in the maternal circulation and therefore makes insulin accordingly. After deliver, the infant's glucose source is withdrawn and the infant is then prone to hyperinsulinism thus manifesting as hypoglycemia. Blood glucose levels should be closely monitored and early feedings are recommended. Hypocapnia and hyponatremia are not associated with newborns of diabetic mothers. Hyperglycemia is a risk, especially if the newborn is receiving parenteral glucose to treat hypoglycemia.

A nursery nurse performs an initial newborn assessment and checks the umbilical cord to identify which normal finding? A. One artery and two veins B. One artery and one vein C. Two arteries and two veins D. Two arteries and one vein

Answer: D Rationale: The umbilical cord consists of two arteries and one vein. In the umbilical cord, the vein provides oxygen and nutrients, and the arteries pump oxygen-depleted blood back to the placenta. The other answer options A, B, and C are incorrect.

How can a nurse decrease the risk of vulvar or vaginal hematoma?

Applying an ice pack to the woman's perineum during the first hour after birth and intermittently thereafter for the next 8 to 12 hours.

A pregnant woman's last menstrual period began on April 8, 2009, and ended on April 13. Using Nagele's rule, her est DOB would be: a. January 15, 2010 b. January 20,2010 c. December 15, 2009 d. November 5, 2009

A. Nagele's rule - subtract 3 months + add 7 days and 1 year from the first day of mom's LMP.

The nurse must evaluate a male patient's knowledge regarding the use of a condom. The nurse would recognize the need for further instruction if the patient states that he: a. lubricates the condom with a spermicide containing nonoxynol-9 b. leaves an empty space at the tip of the condom c. leaves a small amount of air int eh tip d. removes his still erect penis from the vagina while holding onto the base of the condom.

A. Nonoxynol-9 is no longer recommended. Recent data suggest that frequent use of nonoxynol-9 may increase human immunodeficiency virus transmission and can cause genital lesions.

Families in the launching stage of the family life cycle are involved in accomplishing which developmental task? a. renegotiating the marital relationship as a dyad b. establishing financial independence c. maintaining own and/or couple functioning and interests in the face of physiologic decline d. negotiating tasks related to childrearing and household maintenance

A. Renegotiating is the stage of launching children and moving on.

When planning a diet with a pregnant woman, the nurse's first action would be to: a. review the woman's current dietary intake b. teach the woman about the food pyramid c. caution the woman to avoid large doses of vitamins, especially those that are fat soluble d. instruct the woman to limit the intake of fatty foods.

A. Reviewing the woman's dietary intake as the first step will help to establish if she has a balanced diet or if changes in the diet are required.

A woman with severe preeclampsia is receiving a magnesium sulfate infusion. The nurse becomes concerned after assessment when the woman exhibits: A a sleepy, sedated affect. B a respiratory rate of 10 breaths/min. C deep tendon reflexes of 2+. D absent ankle clonus.

B Because magnesium sulfate is a central nervous system (CNS) depressant, the client will most likely become sedated when the infusion is initiated. A respiratory rate of 10 breaths/min indicates that the client is experiencing respiratory depression (bradypnea) from magnesium toxicity. Deep tendon reflexes of 2+ are a normal finding. Absent ankle clonus is a normal finding.

The most prevalent clinical manifestation of abruptio placentae (as opposed to placenta previa) is: A bleeding. B intense abdominal pain. C uterine activity. D cramping.

B Bleeding may be present in varying degrees for both placental conditions. Pain is absent with placenta previa and may be agonizing with abruptio placentae. Uterine activity may be present with both placental conditions. Cramping is a form of uterine activity that may be present in both placental conditions.

A pregnant woman at 7 weeks of gestation complains to her nurse midwife about frequent episodes of nausea during the day with occasional vomiting. She asks what she can do to feel better. The nurse midwife could suggest that the woman: A drink warm fluids with each of her meals. B eat a high-protein snack before going to bed. C keep crackers and peanut butter at her bedside to eat in the morning before getting out of bed. D schedule three meals and one midafternoon snack a day.

B Fluids should be taken between (not with) meals to provide for maximum nutrient uptake in the small intestine. A bedtime snack of slowly digested protein is especially important to prevent the occurrence of hypoglycemia during the night that would contribute to nausea. Dry carbohydrates such as plain toast or crackers are recommended before getting out of bed. Eating small, frequent meals (about five or six each day) with snacks helps to avoid a distended or empty stomach, both of which contribute to the development of nausea and vomiting.

A woman is evaluated to be using an effective bearing down effort if she: a. begins pushing as soon as she is told that her cervix is fully dilated and effaced b. takes two deep, cleansing breaths at the onset of a uterine contraction and at the end of the contraction c. uses the valsalva maneuver by holding her breath and pushing vigorously for a count of 12 d. continues to push for short periods between uterine contractions throughout the second stage of labor

B. Cleansing breaths at the onset of a contraction allow it to build to a peak before pushing begins. They also enhance gas exchange in the alveoli and help the woman relax after the uterine contraction subsides.

An amniocentesis is ordered for an 8-month-pregnant client. The nurse is aware that the most common reason for this test at this time is to test for: a. Fetal demise. b. Fetal chromosome abnormalities. c. Fetal lung maturity. d. Amniotic fluid amount

C. Fetal lung maturity. Correct - A common indication for amniocentesis during the third trimester is to determine fetal lung maturity. Amniocentesis may be done midtrimester to determine chromosomal abnormalities. Amniocentesis is not done to determine fetal demise or amniotic fluid amount.

A 26 year old woman is considering Depo-Provera as the form of contraception that is best for her since she does not like to worry about taking a pill every day. To assist this woman with decision making concerning this method of contraception, the nurse would tell her that Depo-Provera: a. is a combination of progesterone and estrogen b. is a small adhesive hormonal birth control patch that is applied weekly c. thickens and decreases cervial mucus, thereby inhibiting sperm penetration and ovulation d. has an effectiveness rate in preventing pregnancy of 99% when used correctly

C. In addition to the changes in the cervical mucous, some but not all ovulatory cycles are suppressed, and formation of an endometrium capable of supporting implantation is inhibited.

A client comes to the clinic requesting an ultrasound to confirm a pregnancy. Her last menstrual period was 15 days ago. The nurse's best response would be to: a. Make an appointment. b. Ask the reason for needing a confirmation this soon in the pregnancy. c. Inform her that an ultrasound cannot confirm pregnancy this early. d. Refer her to a specialist who is trained in confirming early pregnancies.

C. Inform her that an ultrasound cannot confirm a pregnancy this early. -The embryo is not visualized until 5 to 6 weeks after the last menstrual period.

Following rupture of membranes, a prolapse of the cord was noted on vaginal examination. A recommended action to prevent cord compression would be to: a. place the woman in a supine position and elevate legs from the hips b. insert a foley catheter to keep the bladder empty c. keep the protruding cord moist with warm sterile normal saline compresses d. attempt to reinsert the cord

C. It is advised to keep the protruding cord moist with sterile saline until further help arrives.

The nurse should include questions regarding sexuality when gathering data for a reproductive health history of a female patient. Which principle should guide the nurse when interviewing the patient? A. An in depth exploration of specific sexual practices should be included for every patient. B. Sexual histories are optional if the patient is not currently sexually active. C. Misconceptions and inaccurate information expressed by the patient should be corrected promptly. D. Questions regarding the patient's sexual relationship are unnecessary if she is monogamous.

C. Misconceptions and inaccurate information expressed by the patient should be corrected promptly

Newborns are at high risk for injury if appropriate safety precautions are not implemented. Parents should be taught to: a. place newborn on abdomen (prone) after feeding and for sleep b. avoid use of pacifiers c. use a rear facing car seat until the infant weights at least 20 lbs d. use a crib with side rail slats that are no more than 3 inches apart

C. The APA recommends using a rear facing car seat until a baby weighs 20 lbs

Following the birth of her baby, a woman expresses concern about the weight she gained during pregnancy and how quickly she can lose it now that the baby is born. The nurse, in describing the expected pattern of weight loss, should begin by telling this woman that: a. return to prepregnant weight is usually achieved by the end of the postpartum period b. fluid loss from diuresis, diaphoresis, and bleeding accounts for about a 3 pound weight loss c. the expected weight loss immediately after birth averages about 11 to 13 pounds. d. lactation will inhibit weight loss since caloric intake must increase to support milk production

C. The expected weight loss immediately following deliver is 11 to 13 pounds, followed by a gradual decrease and a return to prepregnancy weight in 2-3 months.

At 1 minute following birth, the newborn exhibited the following: heart rate of 155; loud, vigorous crying with active movement of all extremities; sneezing when nose is stimulated with a catheter; hands and feet bluish and cool to the touch. The Apgar score of this newborn should be recorded as: a. 5 b. 7 c. 9 d. 10

C. The newborn receives 2 points for each of heart rate over 100 BPM, a vigorous cry, active movement, and sneezing as a response to nasal stimulation. The newborn receives 1 point for color since he exhibits acrocyanosis.

When the nurse uses a standard nursing care plan as a guide in planning care for a hospitalized child, what should be eliminated? a. expected outcome/goal b. dependent nursing functions c. problems not pertinent to the child or family d. potential health problems of the child or family

C. These are general problems/interventions that may occur. To create an individualized plan of care, the nurse eliminates the non relevant material and concentrates on specific information pertinent to the child and family in question.

A 65 year old woman, G6 P6006 is complaining of increasing stress incontinence and pelvic pressure and fulness. Pelvic examination reveals a bulging in the anterior vaginal wall. The woman is most likely experiencing: a. Uterine prolapse b. Rectocele c. Cystocele d. Vesicovaginal fistula

C. This is the classic clinical manifestations of cystocele.

An expectant father confides in the nurse that his pregnant wife, 10 weeks of gestation, is driving him crazy. "one minute she seems happy, and the next minute she is crying over nothing at all. Is there something wrong with her?" The nurse's best response would be: a. "this is normal and should subside by the second trimester." b. "She may be having difficulty adjusting to pregnancy; I will refer her to a counselor that I know. c. "This is called emotional liability and is related to hormone changes and anxiety during pregnancy. The mood swings will eventually subside as she adjusts to being pregnant." d. "You seem impatient with her. Perhaps this is precipitating her behavior."

C. This is the most appropriate response since it gives an explanation and a time frame for when the mood swings may stop.

A 36-year-old primigravida is in the clinic for her first prenatal appointment. The nurse can anticipate that the multiple-marker screening may be done on this client to screen for: a. Gestational diabetes. b. Hypertensive disease of pregnancy. c. Trisomy disorders. d. Placenta previa.

C. Trisomy disorders. Correct - The multiple-marker screening test (MSAFP, hCG, unconjugated estriol, inhibin A) is used to screen for trisomies in the fetus. Older primigravidas are at higher risk for these defects.

A woman is in the second stage of labor and has a spinal block in place for pain management. The nurse obtains the woman's blood pressure and notes that it is 20% lower than the baseline level. Which action should the nurse take? a. encourage her to empty her bladder. b. decrease her IV rate to keep vein open rate. c. turn the woman to the left lateral position or place a pillow under her hip d. no action is necessary since a decrease int eh woman's blood pressure is expected.

C. Turning the woman to her left side is the best action to take in this situation since this will increase placental perfusion to the infant while waiting for the doctor's or nurse midwife's instruction.

A nurse providing care for the antepartum woman should understand that the contraction stress test (CST): A. sometimes uses vibroacoustic stimulation. B. is an invasive test; however, contractions are stimulated. C. is considered negative if no late decelerations are observed with the contractions. D. is more effective than nonstress test (NST) if the membranes have already been ruptured.

C. is considered negative if no late decelerations are observed with the contractions. Vibroacoustic stimulation is sometimes used with NST. CST is invasive if stimulation is by IV oxytocin but not if by nipple stimulation. No late decelerations indicate a positive CST. CST is contraindicated if the membranes have ruptured.

On examining a woman who gave birth 5 hours ago, the nurse finds that the woman has saturated her pad within 15 minutes. What should be the nurse's first action?

Check the fundus and massage if boggy

During her gynecologic checkup, a 17-year-old girl states that recently she has been experiencing cramping and pain during her menstrual periods. The nurse would document this complaint as:

Dysmenorrhea.

When caring for a pregnant woman with cardiac problems the nurse must be alert for s/sx of cardiac decompensation, which are

Dyspnea, crackles, an irregular weak pulse

When teaching an antepartal client about the passage of the fetus through the birth canal during labor, the nurse describes the cardinal mechanisms of labor. Place these events in the proper sequence in which they occur: 1. Flexion 2. External rotation 3. Descent 4. Expulsion 5. Internal Rotation 6. Extension

1. Descent 2. Flexion 3. Internal rotation 4. Extension 5. External rotation 6. Expulsion DFI EEE

The diagnosis of pregnancy is based on which positive signs of pregnancy?

1. Identification of fetal heartbeat 2. Visualization of the fetus 3. Verification of fetal movements

A woman arrives at the clinic seeking confirmation that she is pregnant. The following info is obtained; 24 years old, BMI-17.5, she admits to having used cocaine several times during the past year and drinks alcohol occasionally. BP 108/70, pulse 72 bpm, resp. 16 bpm, family history diabetes mellitus and cancer. her sister recently gave birth to an infant with neural tube defect NTD Which characteristics place her at a high risk category

Family history of NTD, low BMI, and substance abuse are all high risk factors

A woman has a thick, white, lumpy, cottage cheese-like discharge, with patches on her labia and in her vagina. She complains of intense pruritus. The nurse practitioner would order which preparation for treatment?

Fluconazole

A woman has thick, white lumpy cottage cheese like discharge, with patches on her labia and in her vagina. She complains of intense pruritus the nurse practitioner would order which preparation for treatment

Fluconazole

Evidence base practice, a current helath care trend, is best described as: a. gathering evidence of mortality and morbidity in children b. meeting physical and psychosocial needs of the child and family in all areas of practice c. using a professional code of ethics as a means for professional self regulation d. questioning why something is effective and whether there is a better approach

D. Evidence based practice helps to focus on measurable outcomes and the use of demonstrated, effective interventions and questions whether there is a better approach.

When obtaining a reproductive health history from a female patient, the nurse should: A. Limit the time spent on exploration of intimate topics. B. Avoid asking questions that may embarrass the patient. C. Use only accepted medical terminology when referring to body parts and functions. D. Explain the purpose for the questions asked and how the information will be used.

D. Explanation of the purpose for the questions asked while obtaining a reproductive health history will help to gather honest and relevant data.

Two hours after giving birth, a primiparous woman becomes anxious and complains of intense perineal pain with a strong urge to have a bowel movement. Her fundus is firm, at the umbilicus, and midline. Her lochia is moderate rubra with no clots. The nurse would suspect: a. bladder distention b. uterine atony c. constipation d. hematoma formation

D. Increasing perineal pressure along with a firm fundus and moderate lochial flow are characteristic of hematoma formation

A woman who is at 36 weeks of gestation is having a nonstress test. Which statement by the woman would indicate a correct understanding of the test? a. I will need to have a full bladder for the test to be done accurately. b. I should have my husband drive me home after b/c I may be nauseous. c. this test will help to determine if the baby has Down syndrome or a NTD. d. This test will observe for fetal activity and an acceleration of the fetal heart rate to determine the well being of the baby.

D. the nonstress test is one of the most widely used techniques to determine fetal well being and is accomplished by monitoring fetal heart rate in conjunction with fetal activity and movements.

Metabolic changes throughout pregnancy that affect glucose and insulin in the mother and the fetus are complicated. Nurses should know which of the following statements is accurate?

During the second and third trimester, pregnancy exerts a diabetogenic effect that ensures an abundant supply of glucose for the fetus

A 40-year-old woman with a high body mass index (BMI) is 10 weeks pregnant. Which diagnostic tool is appropriate to suggest to her at this time? A. Biophysical profile B. Amniocentesis C. Maternal serum alpha-fetoprotein (MSAFP) D. Transvaginal ultrasound

D. Transvaginal ultrasound A biophysical profile is a method of biophysical assessment of fetal well-being in the third trimester. An amniocentesis is performed after the fourteenth week of pregnancy. A MSAFP test is performed from week 15 to week 22 of the gestation (weeks 16 to 18 are ideal). An ultrasound is the method of biophysical assessment of the infant that is performed at this gestational age. Transvaginal ultrasound is especially useful for obese women whose thick abdominal layers cannot be penetrated adequately with the abdominal approach.

A nurse is assessing a newborn infant following circumcision and notes that the circumcised area is red with a small amount of bloody drainage. Which of the following nursing actions would be most appropriate? 1. Document the findings 2. Contact the physician 3. Circle the amount of bloody drainage on the dressing and reassess in 30 minutes 4. Reinforce the dressing

1. The penis is normally red during the healing process. A yellow exudate may be noted in 24 hours, and this is a part of normal healing. The nurse would expect that the area would be red with a small amount of bloody drainage. If the bleeding is excessive, the nurse would apply gentle pressure with sterile gauze. If bleeding is not controlled, then the blood vessel may need to be ligated, and the nurse would contact the physician. Because the findings identified in the question are normal, the nurse would document the assessment.

Which of the following fetal positions is most favorable for birth? 1. Vertex 2. Transverse lie 3. Frank breech presentation 4. Posterior position of the head

1. Vertex presentation flexion of the fetal head is the optimal presentation for passage thru the birth canal. Transverse lie is an unacceptable fetal position for vaginal birth and requires c-section frank breech presentation, in which the buttocks presents first, can be difficult vaginal delivery. posterior positioning of the fetal head can make it difficult for the fetal head to pass under the maternal symphysis pubis.

When performing an assessment on a neonate, which assessment finding is MOST SUGGESTIVE of hypothermia? 1. bradycardia 2. hyperglycemia 3. metabolic alkalosis 4. shivering

1. bradycardia hypothermic neonates become bradycardic proportional to the degree of core temp HYPOGLYCEMIA is seen in HYPOTHERMIC NEONATES Shivering is rarely observed in neonates metabolic ACIDOSIS, not alkalosis is seen due to slowed resp

Clients with gestational diabetes are usually managed by which of the following therapies? 1. diet 2. long acting insulin 3. oral hypoglycemic drugs 4. oral hypoglycemic drugs/insulin

1. diet oral hypoglycemics are contraindicated in preg. long acting insulin usually inst needed for blood glucose control in the client with GDM

Which of the following changes in resp functioning during pregnancy is considered normal? 1. increased tidal volume 2. increases expiratory volume 3. decreased inspiratory capacity 4. decreased oxygen consumption.

1. increased tidal volume a pregnant client breathes deeper, which increases the tidal volume of gas moved in and out of the respiratory tract w/ each breath. The expiratory volume and residual volume DECREASE as the preg. progresses. The inspiratory capacity INCREASES during preg. The increases oxygen consumption in the preg client is 15-20% greater than in the nonpreg state

Which statement best describes lochia rubra? 1. it contains a mixture of mucus, tissue debris and blood 2. it contains placental fragments, and blood 3. it contains mucus, placental fragments and blood. 4. it contains tissue debris and blood

1. lochia rubra contains a mixture of mucus tissue debris blood normal lochia rubra contains NO PLACENTAL FRAGMENTS

Two days after circumcision, the nurse notes a yellow - white exudate around the head of the neonates penis. What would be the most appropriate nursing intervention? 1. Leave the area alone as this is a normal finding 2. report findings to physician and document it 3. Take the neonate's temperature bc an infection is suspected 4. Try to remove the exudate with a warm washcloth

1. the yellow white exudate is part of the granulation process and a normal finding for a healing penis after circumcision therefore, notifying the doctor isnt necessary theres no indication of an infection that would necessitate taking the neonates temp the exudate shouldnt be removed

Which statement would indicate that the client requires additional instruction about breast self-examination?

"Yellow discharge from my nipple is normal if I'm having my period."

A woman inquires about herbal alternative methods for improving fertility. Which statement by the nurse is the most appropriate when instructing the client in which herbal preparations to avoid while trying to conceive?

"You may want to avoid licorice root, lavender, fennel, sage, and thyme while you are trying to conceive."

A woman is using the basal body temperature (BBT) method of contraception. She calls the clinic and tells the nurse, "My period is due in a few days, and my temperature has not gone up." The nurse's most appropriate response is: "This probably means you're pregnant." "Don't worry; it's probably nothing." "Have you been sick this month?" "You probably didn't ovulate during this cycle."

"You probably didn't ovulate during this cycle." Pregnancy cannot occur without ovulation (which is being measured using the BBT method). A comment such as this discredits the client's concerns. Illness would most likely cause an increase in BBT. The absence of a temperature decrease most likely is the result of lack of ovulation.

Fetal Movement

-4 or more FM in 30 min 3 times per day is very reassuring -Contact health provider if you do not feel movement for a couple of days -Count fetal movement for 30 minutes for 3 times a day -Occurs at 20 weeks

Nursing Response to Lochia Assessment

-Assess for firmness, express additional clots; begin peripad count -Assess for other signs of infection; report to physician or certified nurse midwife

Nursing Response to Breast Assessment Postpartum

-Assess further for signs of infection -If blocked duct, consider heat, massage, and position change for breast feeding. -Report mastitis to physician or certified nurse midwife -Reassess technique: recommend appropriate interventions

Nursing Response to Breast Assessment (Breast feeding)

-Assess further for signs of infection -If blocked duct, consider heat, massage, position change for breast feeding. -Report mastitis to physician or certified nurse midwife

Nursing care for the second stage of labor

-Assist woman in pushing efforts, encourage woman to assume position of comfort. -Provide encouragement and praise for efforts. -Keep couple informed of progress -Provide ice chips -Maintain privacy

Criteria and duration of the transition phase of labor

-Cervix is 8 to 10 cm dilated -Contractions every 2 to 3 minutes lasting 60 to 90 seconds with strong intensity. -Woman is tired; may exhibit increased restlessness and irritability. -May feel that she can't keep up with labor process and is out of control -Physical discomforts, fear of being left alone, may fear tearing open or splitting apart with contractions.

Signs of respiratory distress and cold stress

-Chest Retractions -Nasal Flaring -Expiratory Grunt -Increased respirations -Increased Metabolic Rate (Burn Brown Fat) -Put on a hat, wrap in a blanket, use radiant heat

Third Stage of Labor

-Defined as the period of time from the birth of the infant until the completed delivery of the placenta

Nursing Response to Elimination Assessment Postpartum

-Employ nursing interventions to promote voiding; if not successful, obtain order for catheterization -Report symptoms of UTI to physician or certified nurse midwife -Encourage fluids, ambulation, roughage in diet; sitz baths to promote healing of perineum; obtain order for stool softner

Variable decelerations

-These can occur either late or early in respect to the uterine contractions -This is usually do to cord compression

What are priority nursing goals during the first 4 hours after birth?

-To promote the physical well-being of the newborn -To support the establishment of a well0funcitoning family unit

Physiological cause of Maternal hypotension

-Vena cava obstruction -Hemorrhaging after birth.

A positive Babinski's signs is present in infants until approx. what age?

1 year of age this is normal in neonates but abnormal in adults

Which action best explains the main role of surfactant in the neonate? 1. Assists with ciliary body maturation in the upper airways 2. Helps maintain a rhythmic breathing pattern 3. Promotes clearing mucus from the respiratory tract 4. Helps the lungs remain expanded after the initiation of breathing

1. 4. Surfactant works by reducing surface tension in the lung. Surfactant allows the lung to remain slightly expanded, decreasing the amount of work required for inspiration.

A client with group AB blood whose husband has group O has just given birth. The major sign of ABO blood incompatibility in the neonate is which complication or test result? 1. Negative Coombs test 2. Bleeding from the nose and ear 3. Jaundice after the first 24 hours of life 4. Jaundice within the first 24 hours of life

1. 4. The neonate with ABO blood incompatibility with its mother will have jaundice (pathologic) within the first 24 hours of life. The neonate would have a positive Coombs test result.

When performing a newborn assessment, the nurse should measure the vital signs in the following sequence: 1. Pulse, respirations, temperature 2. Temperature, pulse, respirations 3. Respirations, temperature, pulse 4. Respirations, pulse, temperature

1. 4. This sequence is least disturbing. Touching with the stethoscope and inserting the thermometer increase anxiety and elevate vital signs.

Which of the following conditions is common in pregnant clients in the 2nd trimester of preg? 1. mastitis 2. metabolic alkalosis 3. physiologic anemia 4. respiratory acidosis

3. physiologic anemia Hgb and Hct values DECREASE during preg as the INCREASE in plasma volume exceeds the increase in RBC production. Alterations in acid-base balance during pregnancy result in a state of resp. alkalosis, compensated by mild metabolic acidosis. Mastitis is an infection in the breast characterized by a swollen tender breast and flu like Sx. this condition is most freq. seen in breast feeding clients.

After reviewing the client's maternal history of magnesium sulfate during labor, which condition would the nurse anticipate as a potential problem in the neonate? 1. hypoglycemia 2. jitteriness 3. resp depression 4. tachycardia

3. resp depression mag sulfate crosses the placenta and adverse neonatal effects are: resp depression hypotonia bradycardia the serum blood sugar isnt affected by mag sulfate the neonate wold be floppy, not jittery

Which assessment finding would be the most unlikely risk factor for RDS 1. second born of twins 2. neonate born at 34 weeks 3. neonate of diabetic mom 4. chronic maternal HTN

4. CHRONIC MATERNAL HTN unlikely factor bc chronic fetal stress tends to INCREASE LUNG MATURITY premature neonates < 35 weeks are associated with RDS Even with a mature lecithin to sphingomyelin ration, neonates of moms with diabetes still develop RDS 2nd born of twins may be prone to greater risk of asphyxia

An expectant couple asks the nurse about intercourse during pregnancy and if it is safe for the baby. The nurse should tell the couple that: a. Intercourse should be avoided if any spotting from the vagina occurs afterward b. Intercourse is only safe in the missionary position c. Safer sex practices should be used once the membranes rupture d. Intercourse and orgasm are often contraindicated if a history or signs of preterm labor are present.

D. Uterine contractions that accompany orgasm can stimulate labor and would be problematic if the woman were at risk for or had a history of preterm labor.

When weighing a newborn, the nurse should: a. leave its diaper on for comfort b. place a sterile scale paper on the scale for infection control c. keep hand on the newborn's abdomen for safety d. weight the newborn at the same time each day for accuracy

D. Weigh the newborn at the same time each day for accuracy

A woman is recovering at the gynecologists office following a late first trimester spontaneous abortion, at this time it is essential for the nurse too check which of the following:

Maternal blood type

A woman in active labor receives an analgesic, an opioid agonist. Which med relieves severe, persistent or recurrent pain, creates a sense of well-being, overcomes inhibitory factors and may even relax the cervix but is used cautiously with cardiac disease?

Meperidine (Demerol)

Which statement is true about the term contraceptive failure rate?

It varies from couple to couple, depending on the method and the users.

A woman at 39 weeks gestation expresses fear about her impending labor and how she will manage. What is the best response?

It's normal to be anxious about labor. Let's discuss what makes you afraid.

A pregnant woman informs the nurse that her last normal menstrual period was on Sep 20, 2012, using Nagele's rule the nurse calculates the clients EDD as:

June 27, 2013

The most critical nursing action in caring for the newborn immediately after birth is

Keeping the newborn's airway clear

RN examining a woman who gave birth 5 hours ago, the nurse finds that the woman has completely saturated a perineal pad within 15 minutes. The nurse's priority would be to

Massage the fundus

Why is magnesium sulfate given to women with preeclampsia and eclampsia?

Prevent convulsion

A new father wants to know what med was put into the baby's eyes and why. The nurse explains the purpose as:

Prevent gonorrheal and chlamydial infection of the infant's eye potentially acquired from the birth canal

Because of the effect of cyclic ovarian changes on the breast, the best time for breast self-examination (BSE) is:

5 to 7 days after menses ceases.

At 1 minute following birth, the newborn exhibited the following: heart rate of 95; loud, vigorous crying with some flexion of extremities; sneezing when nose is stimulated with a catheter; body is blue and pale. The Apgar score of this newborn should be recorded as________.

6

Which finding would be manifested in an infant with a myelomeningocele? A. clubbed feet and paralysis in the legs and arms B. Obstruction of bowel and impaired bladder function C. Spastic movement of upper and lower extremities D. Impaired bowel and bladder function and paralysis of the legs

Answer: D Rationale: A myelomeningocele is an external sac containing meninges, spinal fluid, and nerves that protrude through a defect in the vertebral column. The nerves of the cauda equina are involved with a myelomeningocele, which results in lower extremity paralysis. Innervation to the anal sphincter and the bladder is decreased, causing incontinence. In myelomeningocele, bowel function may be affected, but it is not associated with obstruction. The upper extremities are not affected by this disorder. In addition, clubfoot is not associated with this disorder.

During assessment the nurse understands that the Moro reflex should disappear by what age in an infant? A. 4 weeks B. 6 weeks C. 2 months D. 4 months

Answer: D Rationale: The Moro reflex is exhibited by the infant when suddenly jarred or a change in equilibrium occurs. This reflex usually disappears by 4 months. If the Moro reflex is still present after the age of 6 months, neurological maturity may be delayed or another neurological disorder may be present. The other answer options A, B, and C are incorrect.

A woman arrives at the clinic for her annual examination. She tells the nurse that she thinks she has a vaginal infection and she has been using an over-the-counter cream for the past 2 days to treat it. The nurse's initial response should be to:

Ask the woman to describe the symptoms that indicate to her that she has a vaginal infection.

Nulliparous woman telephones the hospital to report that she is in labor, the nurse should initially

Ask the women to describe why she believes she is in labor

During a prenatal intake interview, the nurse is in the process of obtaining an initial assessment of a 21-year-old Hispanic patient with limited English proficiency. It is important for the nurse to:

Assess whether the patient understands the discussion.

Which characteristic is associated with false labor contractions? a. Painless b. decrease in intensity with ambulation c. regular pattern of frequency established d. progressive in terms of intensity and duration

B. Although false labor contractions decrease with activity, true labor contractions are enhanced or stimulated with activity such as ambulation.

When assessing a fetal heart rate tracing, the nurse notes a decrease in the baseline rate from 155 to 110. The rate of 110 persists for more than 10 minutes. The nurse could attribute this decrease in baseline to: a. maternal hyperthyroidism b. initiation of epidural anesthesia that resulted in maternal hypotension c. Maternal infection accompanied by fever d. alteration in maternal position from semirecumbent to lateral.

B. Fetal bradycardia is the pattern described and results from hypoxia that would occur when uteroplacental perfusion is reduced by maternal hypotension. The woman receiving epidural anesthesia needs to be well hydrated before and during induction of anesthesia to maintain adequate cardiac output and blood pressure.

The nurse-midwife is teaching a group of women who are pregnant, including instruction on Kegel exercises. which statement by a participant would indicate a correct understanding of the instruction? a. I will only see results if I perform 100 Kegel exercises each day. b. I should hold the keel exercise contraction for 10 seconds and rest for 10 seconds between exercises c. I should only perform Kegel exercises in the sitting position d. I will perform daily kegel exercises during the last trimester of my pregnancy to achieve the best results.

B. I should hold the Kegel exercise contraction for 10 seconds and rest for 10 seconds between exercises.

Late decls patterns are noted when assessing the monitor tracing of a woman whose labor is being induced with an infusion of Pitocin. The woman is in a side lying position, and her vitals are stable and fall w/in normal range. Contractions are intense, last 90 seconds, and occur every 1 1/2 to 2 minutes. The nurse's immediate action would be to: a. change the womans position b. stop the pitocin c. elevate the woman's legs d. admin oxygen via tight mask at 8-10 L/min

B. Late decel patterns are most likely related to alteration in uteroplacental perfusion associated with the strong contractions described. The immediate action would be to stop the Pitocin infusion since Pitocin is a oxytocic that stimulates the uterus to contract.

When assessing the fetal heart rate of a woman at 30 weeks of gestation, the nurse counts a rate of 82 beats/min. Initally the nurse should: a. Recognize that the rate is within normal limits and record it. b. Assess the woman's radial pulse. c. Notify the physician. d. Allow the woman to hear the heart beat.

B. The nurse may have inadvertently counted the uterine souffle, the beatlike sound of blood flowing through the uterine blood vessels, which corresponds to the mother's heartbeat.

An important development that affects maternity nursing is integrative health care, which:

Blends complementary and alternative therapies with conventional Western treatment.

Acrocyanosis

Bluish discolorazation of the feet and hands

The body part that both protects the pelvic structures and accommodates the growing fetus during pregnancy is the:

Bony pelvis.

The most effective and least expensive treatment of puerperal infection is prevention. What is important in this strategy? A. Large doses of vitamin C during pregnancy B. Prophylactic antibiotics C. Strict aseptic technique, including handwashing, by all health care personnel D. Limited protein and fat intake

C A. Incorrect: Good nutrition to control anemia is a preventive measure. Increased iron intake would assist in preventing anemia. B. Incorrect: Antibiotics may be given to manage infections; they are not a cost-effective measure to prevent postpartum infection. C. Correct: Strict adherence by all health care personnel to aseptic techniques during childbirth and the postpartum period is very important and the least expensive measure to prevent infection. D. Incorrect: Good nutrition to control anemia is a preventive measure. Limiting protein and fat intake will not help prevent anemia or prevent infection. p. 988

A nurse caring for a newborn should be aware that the sensory system least mature at the time of birth is: A vision. B hearing. C smell. D taste.

A The visual system continues to develop for the first 6 months. As soon as the amniotic fluid drains from the ear (minutes), the infant's hearing is similar to that of an adult. Newborns have a highly developed sense of smell. The newborn can distinguish and react to various tastes.

The nurse is assessing a newborn girl who is 2 hours old. Which of the following findings would warrant a call to the physician?

A crepitant-like feeling when assessing the clavicles

The CDC recommended medication for the treatment of chlamydia would be: a. Doxycycline b. Podofilox c. Acyclovir d. Penicillin

A. Doxycycline is effective for treating chlamydia, but is should be avoided if the woman is pregnant.

The process by which people retain some of their own culture while adopting the practices of the dominant society is known as:

Acculturation.

A new mother states that her infant must be cold because the baby's hands and feet are blue, The nurse explains that this is a common and temporary condition called

Acrocyanosis

Prenatal testing for HIV is recommended for which of the following women?

All women, regardless of risk factors

Because uteroplacental circulation is compromised in clients with preeclampsia, a NST is performed to detect which conditions? 1. Anemia 2. Fetal well being 3. IUGR 4. oligohydarminos

An NST is based on the theory that a healthy fetus will have transient fetal heart rate accelerations with fetal movement. A fetus with compromised uteroplacental circulation usually wont have these accelerations, which indicate a nonreactive NST. Serial US wil detect IUGR and oligohydramnios in a fetus. An NST cant detect anemia in a fetus

A nurse is caring for a woman in active process of labor. The woman's water spontaneously ruptures. She then complains of dyspnea and appears restless and cyanotic and also is hypotensive and tachycardic. The nurse immediately suspects the presence of?

An amniotic fluid embolism (also known as Anaphylactoid Syndrome of Pregnancy)

A neonate experiences meconium aspiration at the time of delivery and develops respiratory distress syndrome (RDS). Which nursing diagnosis would be most appropriate for an infant diagnosed with this disorder? A. Risk for Infection B. Risk for Aspiration C. Impaired Gas Exchange D. Dysfunctional Ventilatory Weaning Response

Answer: C Rationale: Impaired gas exchange is the most appropriate nursing diagnosis because meconium aspiration interferes with the exchange of O2 and CO2. Risk for infection is present but is not as high a priority as impaired gas exchange. Risk for aspiration has already occurred. Dysfunctional ventilatory weaning response may be appropriate i the newborn demonstrates difficulty with the ventilatory weaning process

When instructing a new mother about the newborn's need for sensory stimulation, the nurse should explain that the most highly developed sense is the neonate is: A. taste B. smell C. touch D. hearing

Answer: C Rationale: It is believed that the sense of touch is the most highly developed sense at birth. For this reason, neonates respond well to touch. The senses of taste, smell, and hearing are functional in the newborn; however, not the extent of touch.

A new father observes his newborn infant receiving a vitamin K injection and asks the nurse, "Why did my son need a shot?" The nurse's response should be based on the understanding that infants: A. need vitamin K to stimulate liver maturation B. cannot get enough vitamin K from their feeeings C. have a sterile intestinal tract and cannot synthesize vitamin K D. are often born with hypokalemia, which responds to vitamin K therapy

Answer: C Rationale: There seems to be some controversy as to whether vitamin K should be given, but often it is because the newborn infant cannot synthesize vitamin K due to the lack of intestinal bacteria at birth. The other answer options A, B, and D are incorrect rationales for administration of vitamin K in newborns.

Vitamin K is give to the newborn to: a. reduce billirubin levels b. increase the production of red blood cells c. enhance ability of blood to clot d. stimulate the formation of surfactant

C. Newborns have a deficiency of vitamin K until intestinal bacteria that produce vitamin K are formed. Vitamin K is required for the production of certain clotting factors

When making a visit to the home of a postpartum woman 1 week after birth, the nurse should recognize that the woman would characteristically: a. express a strong need to review events and her behavior during the process of labor and birth b. exhibit a reduced attention span, limiting readiness to learn c. vacillate between the desire to have her own nurturing needs met and the need to take charge of her own care and that of her newborn d. have reestablished her role as a spouse/partner

C. One week after birth the woman should exhibit behaviors characteristic of the taking hold stage. this stage lasts for as long as 4-5 weeks after birth.

Which hematocrity (HCT) and hemoglobin (HGB) represent(s) the lowest acceptable values for a woman in the third trimester of pregnancy? a. 38% HCT; 14 g/dl HGB b. 35% HCT; 13 g/dl HGB c. 33% HCT; 11 g/dl HGB d. 32% HCT; 10.5 g/dl HGB

C. represents the lowest acceptable value during the first and third trimesters.

A woman was treated recently for toxic shock syndrome (TSS). She has intercourse occasionally and uses over-the-counter protection. On the basis of her history, what contraceptive method should she and her partner avoid?

Cervical cap

The nurse has received a report about a woman in labor. At last vag exam she was 3 cm, 30%, and -2. What is the interpretation?

Cervix is 3 cm dilated, it is effaced 30% and the presenting part is 2 cm above the ischial spines

While evaluating an external monitor tracing during active labor, the nurse notes the the FHR for 5 contractions begins to decelerate late in the contraction, with nadir (lowest point) of the decel at the peak of the contraction. What is the nurse's first priority?

Change the woman's position

The nurse providing care in a women's health care setting must be aware regarding which sexually transmitted infection that can be successfully treated and cured?

Chlamydia

When evaluating women for STIs, the nurse should be aware the most common bacterial sexually transmitted infection is

Chlamydia

When evaluating a patient for sexually transmitted infections (STIs), the nurse should be aware that the most common bacterial STI is:

Chlamydia.

Hand hygiene:

Comply with CDC guidelines.

When the nurse is unsure about how to perform a patient care procedure, the best action would be to:

Consult the agency procedure manual and follow the guidelines for the procedure.

With regard to the functioning of the renal system in newborns, nurses should be aware that

The pediatrician should be notified if the newborn has not voided in 24 hours

A 5-month-pregnant woman has been diagnosed with iron deficiency anemia. The nurse evaluates the patient teaching on diet to be effective when the woman selects which of the following meals to increase her iron intake? a. Chicken with wild rice, steamed broccoli, sliced tomatoes, a green salad, and orange juice b. Pinto beans with cornbread and milk c. Broiled flounder, baked sweet potatoes, green beans, and iced tea d. Refried beans with corn tortillas, Spanish rice, green salad, and coffee

a. Chicken with wild rice, steamed broccoli, sliced tomatoes, a green salad, and orange juice - Primary sources of iron are meat, fish, chicken, and green leafy vegetables. Foods rich in vitamin C will enhance the absorption of iron. Milk, tea, and coffee will decrease the absorption of iron

Healthy People 2020 has established national health priorities that focus on a number of maternal-child health indicators. Nurses are assuming greater roles in assessing family health and providing care across the perinatal continuum. Therefore it is important for the nurse to be aware that significant progress has been made in:

The reduction of fetal deaths and use of prenatal care.

Regarding the estimation and interpretation of the recurrence of risks for genetic disorders, nurses should be aware that

The risk factor remains the same no matter how many affected children are in the family

Methotrexate is recommended as part of the treatment plan for which of the following obstetric complications?

Unruptured ectopic pregnancy

When caring for a postpartum woman with hemorrhagic shock, the nurse recognizes that the most objective assessment of adequate organ perfusion and oxygenation is which?

Urinary output of at least 30 ml/hr

Increased fetal surveillance should occur in the first trimester of a pregnant woman with preexisting diabetes mellitus to monitor for: a. Congenital anomalies. b. Signs of hypoxia. c. Evidence of placental failure. d. Signs of macrosomia.

a. Congenital anomalies. Correct -There is an increased risk for congenital anomalies, so surveillance should begin early for women with preexisting diabetes. Testing should be done to identify possible neural tube defects, chromosomal abnormalities, and cardiac anomalies. The goal of increased surveillance in the last trimester is to monitor for signs of worsening intrauterine environment, such as signs of fetal hypoxia and evidence of placental failure.

A pregnant woman at 36 weeks of gestation is admitted to the high-risk pregnancy unit with hypertension. Assessment findings indicate severe preeclampsia. The nurse should: a. Expect a maintenance dose of intravenous magnesium sulfate to be about 2 g/hr. b. Assign the woman to a semiprivate room next to the nurses' station. c. Offer her a diet high in complex carbohydrates and low in salt and protein. d. Encourage the woman to maintain a semi-Fowler's position when in bed.

a. Expect a maintenance dose of intravenous magnesium sulfate to be about 2 g/hr. Correct - The woman should be assigned to a private room in a quiet area because she is at risk for seizures, which can be precipitated by any sudden activity or sound. Although her diet should contain complex carbohydrates, protein content should be normal to replace protein losses in urine; low-salt diets are not recommended. While on bed rest, the woman should alternate her position from side to side to facilitate placental and renal perfusion, which will enhance cardiac output and facilitate loss of retained fluid.

The client has a biophysical profile done. The nurse notes that the NST test was reactive, the fetal breathing movements were absent, there was one gross body movement in 30 minutes, the fetal tone showed fetal extremity extension and return to flexion, and one pocket of amniotic fluid measured 3 cm. The nurse anticipates that next action by the physician will be: a. Further study because a score of 6 is not normal. Correct b. No further study because a score of 9 is normal. c. Further study because a score of 3 is not normal. d. No further study because a score of 10 is perfect.

a. Further study because a score of 6 is not normal. Correct -A score of 8 to 10 is normal for a biophysical profile. To determine the score, five categories are used. The reactive NST gives a score of 2; absent fetal breathing movements is 0; one gross body movement is 0; the fetal tone of extension and flexion is 2; and one pocket of amniotic fluid is 2 points. This gives a total of 6 points, which is not within normal limits, and further studies are indicated.

The nurse notices that during the nonstress test, the fetal heart rate accelerated 20 beats/min above baseline three times with fetal movement. Each acceleration lasted about 15 seconds before returning to baseline. This result is classified as: a. Reactive. b. Nonreactive.

a. Reactive. Correct -A reactive (reassuring) nonstress test shows at least two fetal heart accelerations, with or without fetal movement, within a 20-minute period. The accelerations peak at least 15 beats/min above baseline and last 15 seconds from baseline to baseline.

A woman in preterm labor at 30 weeks of gestation receives 2 12 mg doses of betamethasone intramuscularly, The purpose of this pharmacologic treatment is to:

antenatal glucocorticoids given as intramuscular injections to the mother to stimulates fetal lung maturation (surfactant)

Primary powers of labor

Uterine muscular contractions which cause the changes of the first stage of labor

What can maternal hypotension result in?

Uteroplacental insufficiency

Mrs. Jones is G2T1P0A0L1 who had previous C section for fetal distress. She is 16 weeks and has abdominal scar from a horizontal (lower transverse) incision. She wants to know about this delivery- you inform her?

Vaginal birth may be possible, depending on the location of her uterine incision

Which of the following signs and symptoms should a woman report immediately to her health care provider? (select all that apply)

Vaginal bleeding, vaginal fluid leakage (water breaks or rupture of membranes), epigastric discomfort accompanied by severe headache

The nurse should be aware that the criteria used to make decisions and solve problems within families are based primarily on family:

Values and beliefs.

Individual irregularities in the ovarian (menstrual) cycle are most often caused by:

Variations in the follicular (preovulatory) phase.

Postcoital contraception with Ovral:

Requires that the first dose be taken within 72 hours of unprotected intercourse.

Which of the following are the most common causes of subinvolution?

Retained placental fragments and infection

Prepidil (Prostaglandin gel) has been ordered for 43 week woman. The nurse recognizes that this med will be administered to accomplish which action?

Ripen the cervix in preparation for labor induction

The laboratory results for a postpartum woman are as follows; blood type A; Rh status positive; rubella titer, 1:8 (EIA 0.8); hematocrit, 30%. How would the nurse best interpret these data:

Rubella vaccine should be given

During the preconception phase, the nurse should teach about which infectious diseases as risk factors for maternal complications? (Select all that apply.) Diabetes Rubella Hepatitis B Anemia HIV/AIDS

Rubella, Hepatitis B, HIV/AIDS Rubella, Hepatitis B, and HIV/AIDS are all infectious diseases. Diabetes and anemia are chronic diseases.

Which personal safety precaution should guide the nurse working in home care? Do not carry personal items, such as extra car keys or a cellular phone. Avoid making a visit with another nurse. Schedule visits during daylight hours. Never wear a name tag.

Schedule visits during daylight hours. The nurse should carry keys and a cell phone in the event the keys must be used for self-defense or the cell phone is needed to call for help. Making a visit in pairs is a good personal strategy for nurses visiting families with a history of violence or substance abuse. For the nurse's personal safety, all home visits should be conducted during daylight hours. Dress should be casual but professional and should include a name tag.

A woman is having her first child. She has been in labor for 15 hours, 2 hours ago her vaginal examination revealed the cervix to be dilated to 5 cm and 100% effaced, and the presenting part was at station 0, 5 mins ago her vaginal examination indicated that there had been no change what abnormal labor pattern is associated with this description

Secondary arrest

Which of the following is an accurate statement regarding a function of the placenta?

Secretes both estrogen and progesterone

A fully matured endometrium that has reached the thickness of heavy, soft velvet describes the _____ phase of the endometrial cycle.

Secretory

As a pregnant woman lies o the examining table, she grows very short of breath and dizzy. When you take her BP you note she is hypotensive. Why does this phenomenon probably happen?

The blood is trapped in the vena cava in a supine position

What important, immediate postoperative care practice should the nurse remember when caring for a woman who has had a mastectomy?

The blood pressure (BP) cuff should not be applied to the affected arm.

Four house after delivery, the newborn has: Resp 44, breathing is uneven with periods of apnea lasting less than 5 seconds. Resp decrease to 42 when crying. The nurse should conclude?

The infant is demonstrating normal respiratory effort

With regard to the assessment of female, male, and couple infertility, nurses should be aware that:

The investigation takes 3 to 4 months and a significant financial investment.

The perinatal nurse is giving instructions to a woman, stay she must take two post-suction curettage secondary to a hydatidiform mole, the woman asks why she must take oral contraceptives for 12 months the best response

The major risk to you after a molar pregnancy is a type of cancer that can be diagnosed only by measuring the same hormone that your body produced during pregnancy, if you were to get pregnant it would make the diagnosis of this cancer more difficult

The nurse's care of a Hispanic family includes teaching about infant care. When developing a plan of care, the nurse bases interventions on the knowledge that in traditional Hispanic families:

The maternal grandmother participates in the care of the mother and her infant.

Which statement made by the nurse would indicate that he or she is practicing appropriate family-centered care techniques? The nurse encourages the mother and father to make choices whenever possible. The nurse updates the family about what is going to happen but instructs the client's sister that she cannot be present in the room during the birth. The nurse believes that he or she is acting in the best interest of the client and commands her what to do throughout labor. The father is discouraged from accompanying his wife during a cesarean birth.

The nurse encourages the mother and father to make choices whenever possible With family-centered maternity care it is important to allow for choices for the couple and to include the partner in the care process. Unless there is an institutional policy prohibiting the number of attendees at a birth, the client should be allowed to have whomever she desires with her (unless of course the birth is emergent and guests may be requested to leave). FCMC involves collaboration between the health care team and the client. In a family-centered care model, the partner, or even a grandparent may be present for a cesarean birth.

Which statement made by the nurse would indicate that she or he is practicing appropriate family-centered care techniques? (Select all that apply.) The nurse allows the mother and father to make choices when possible. The nurse informs the family about what is going to happen. The nurse instructs the patients sister, who is a nurse, that she cannot be in the room during the birth. The nurse commands the mother what to do. The nurse provides time for the partner to ask questions.

The nurse provides time for the partner to ask questions, The nurse allows the mother and father to make choices when possible It is important to allow for choices for the couple and to include the partner in the care process. Unless there is an institutional policy prohibiting the number of attendees at a birth, the patient should be allowed to have whomever she desires with her (unless the birth is emergent and the guests are requested to leave). Family-centered care involves collaboration between the health care team and the patient.

Which conditions are associated with elevated levels of serum alpha-fetoprotein? (Select all that apply). a. Open neural tube defects b. Threatened abortion c. Chromosomal trisomies d. Gestational trophoblastic disease e. Fetal demise

a, b, e

A nurse is aware that more teaching is necessary when a pregnant woman with gestational diabetes states: a. "I will eat only three meals a day." b. "I will decrease my complex carbohydrates to 40% of my diet." c. "I can increase my fat intake slightly during the pregnancy." d. "I will not eat any sugary snacks until after the baby is born.

a. "I will eat only three meals a day." Correct - A pregnant woman with diabetes should divide her calorie intake among three meals and at least two snacks a day to keep her blood sugar levels consistent. Carbohydrate intake should be about 40% to 45%, fat intake can increase to 40%, and refined sugars should be eliminated from the diet

Assessment of a couple experiencing difficulty conceiving usually begins with: a. A complete history and physical assessment of both partners. b. Semen analysis. c. Testing of cervical mucus for LH surge. d. Postcoital (Sims-Huhner) test.

a. A complete history and physical assessment of both partners. Correct - Assessment for the origin of infertility always begins with the least costly, noninvasive testing first. A complete history and physical assessment of both the male and female recognizes that infertility can have a female or male origin, or both. This assessment will provide clues about which types of tests, if any, would be most appropriate for this couple.

In preparing a pregnant client for a nonstress test, the nurse should: a. Ask the client if she smokes and when she had her last cigarette. b. Have the client drink four glasses of water. c. Ask the client when was the last time she had anything to eat or drink. d. Ask the client for a urine sample to check for glucose levels.

a. Ask the client if she smokes and when she had her last cigarette. - For the greatest accuracy of a nonstress test, the woman should not have smoked recently because smoking affects the perception of fetal movement. In addition, smoking causes vasoconstriction, which may alter the results.

A pregnant woman can reduce her risk for toxoplasmosis by: a. Avoiding the handling of cat litter and soil and not handling or eating raw or uncooked meat. b. Practicing good genital hygiene. c. Using safer sex methods, such as condoms, during intercourse. d. Maintaining a nutritious diet during pregnancy.

a. Avoiding the handling of cat litter and soil and not handling or eating raw or uncooked meat. Correct -Toxoplasmosis is a protozoal infection transmitted by handling or eating anything contaminated with infected cat feces or eating or handling raw or uncooked meat. Options B, C, and D are unrelated to the mode of transmission for toxoplasmosis. Toxoplasmosis is one of several infections that can cause severe congenital anomalies in the developing fetus. It is not sexually transmitted.

During the nursing assessment at a routine prenatal visit, the woman's blood pressure was significantly higher than the last visit. Because of this finding, the nurse should: a. Check the urine for presence of protein. b. Check the pulse for significant decrease. c. Assess for pedal edema. d. Check the urine for presence of glucose.

a. Check the urine for presence of protein. -The two classic signs of preeclampsia are hypertension and proteinuria. Edema in the feet is common in most pregnancies. Edema with preeclampsia may continue up the legs to the hands and face.

The term used to describe a situation in which a cultural group loses its identity and becomes part of the dominant culture is called: assimilation. cultural relativism. acculturation. ethnocentrism.

assimilation Assimilation is the process by which groups "melt" into the mainstream. Cultural relativism refers to learning about and applying the standards of another person's culture to activities within that culture. Acculturation refers to changes that occur within one group or among several groups when people from different cultures come in contact with one another. Ethnocentrism is a belief in the rightness of one's culture's way of doing things.

A woman is admitted to the maternity unit with preeclampsia. She is started on magnesium sulfate IV, a urinary catheter is inserted, and she is put on bed rest. The nurse understands it is important to monitor urinary output hourly. It is important that the client have an output of at least: a. 15 mL. b. 30 mL. c. 60 mL. d. 100 mL.

b. 30 mL. Correct - Normal renal profusion should be an hourly output of at least 30 mL. Decreased urinary output indicates poor perfusion of the kidneys and may precede acute renal failure.

A pregnant woman with cardiac disease is informed about signs of cardiac decompensation. She should be told that the earliest sign of decompensation is most often: a. Orthopnea. b. Decreasing energy level. c. Moist, frequent cough and frothy sputum. d. Crackles at bases of lungs upon auscultation.

b. Decreasing energy level. Correct - Decreasing energy level (fatigue) is an early finding of heart failure, and care must be taken to recognize it as a warning rather than a typical change of the third trimester. Options A, C, and D are manifestations that appear later when a failing heart reduces renal perfusion and fluid accumulates in the pulmonary interstitial space, leading to pulmonary edema. Cardiac decompensation is most likely to occur early in the third trimester, during childbirth, and during the first 48 hours following birth.

A pregnant woman has requested a tubal ligation for contraception. The nurse is aware that this surgery can occur: a. During pregnancy. b. During the postpartum period prior to being discharged from the hospital. c. 6 weeks postpartum. d. 6 months postpartum.

b. During the postpartum period prior to being discharged from the hospital. Correct - A tubal ligation can be performed soon after birth. It can be done at the same time as a cesarean birth or planned before discharge.

An 8-month-pregnant woman with gestational diabetes has been admitted to the antepartal unit of the hospital for fetal surveillance. The woman's blood sugar at 2 PM was 70 mg/dL. The nurse should: a. Record this reassuring blood sugar reading. b. Offer the woman 4 ounces of apple juice. c. Administer the appropriate amount of regular insulin needed for this blood sugar level according to the sliding scale ordered. d. Reassess the blood sugar reading in 30 minutes.

b. Offer the woman 4 ounces of apple juice. Correct - Hypoglycemia should be treated at once to prevent damage to the brain. The woman should take 15 g of carbohydrate, which is about ½ cup of fruit juice.

After 2 years of infertility treatment, a couple decides that they cannot financially go through any more treatments. When discussing this decision with the couple, the nurse can discuss with them: a. The possibility that accepting the fact they may not have a child will cause them to relax and perhaps conceive. b. How to accept the fact they may never have a child. c. The possibility of adoption. d. The possibility of accepting a niece or nephew as a surrogate child.

c. The possibility of adoption. Correct - Not every couple who seeks treatment for infertility achieves a "take home" baby. Adoption may become an option for these couples. The nurse can assist them to explore their personal feelings about adoption, availability of newborns compared with older children, and the pros and cons of adoption.

A couple is in for fertility counseling. They have achieved pregnancy six times, but have lost each pregnancy before the 20th week of gestation. The nurse is aware that the most common cause of repeated pregnancy loss is: a. Lack of FSH hormone. b. Tubal obstruction. c. Dysfunction in the pituitary gland. d. Fetal chromosome defects.

d. Fetal chromosome defects. Correct - Errors in the fetal chromosomes may result in spontaneous abortion, usually in the first trimester. Lack of FSH and dysfunction of the pituitary gland are causes of disorders in ovulation; tubal obstruction prevents implantation

Which of the following new pregnant clients should the nurse monitor more closely for signs of gestational diabetes mellitus? a. Gravida 2 with a body mass index of 22 b. Gravida 1 who is 24 years old c. Gravida 3 whose previous children weighed 6 lb, 4 oz and 7 lb, 5 oz at birth d. Gravida 2 who is pregnant with triplets

d. Gravida 2 who is pregnant with triplets Correct - A woman with a multifetal pregnancy is at risk for developing gestational diabetes. Greatly increased circulating HPL (human placental lactogen) levels as a result of extra placental tissue require a greater maternal insulin production. Women with a body mass index greater than 25, older than the age of 25, or have given birth to infants weighing more than 4000 g are also at risk.

When teaching a diabetic pregnant woman to give herself insulin injections, the nurse should emphasize that: a. A meal should be eaten before insulin injections. b. The angle of the subcutaneous injection should be 45 degrees. c. Once the needle is injected, the woman should aspirate before injecting the medication. d. The medication should be injected slowly.

d. The medication should be injected slowly. Correct - Insulin should be injected slowly (over 2 to 4 seconds) to allow tissue expansion and minimize pressure, which can cause insulin leakage. A meal should be eaten within 30 minutes after insulin is injected. The angle of the injection should be 90 degrees unless the woman is very thin. It is not necessary to aspirate when injecting into subcutaneous tissues.

An 8-month-pregnant client comes to the clinic complaining that she has not felt the baby move for the past 48 hours. Other than checking the fetal heart tones, the nurse can also anticipate which fetal study? a. Amniocentesis b. Multiple-marker study c. Blood flow assessment d. Ultrasonography

d. Ultrasonography Correct - Ultrasonography is used in the last two trimesters to confirm fetal viability.

An ultrasound is ordered for a client who is 8½ months pregnant. It is important for the nurse to: a. Instruct the client to drink several glasses of clear fluid 1 hour before the examination and not to void. b. Place a wedge under one hip when placing her on the examination table. c. Place the client in the lithotomy position for the examination. d. Instruct the client to bring someone with her to the examination to drive her home.

b. Place a wedge under one hip when placing her on the examination table. Correct -When lying supine, the client is at risk for supine hypotension. Placing a wedge or rolled blanket under one hip will move the gravid uterus over to the side away from the vena cava and aorta. Drinking several glasses of clear fluid is needed during the second trimester prior to an ultrasound. A vaginal ultrasound is done during the first trimester.

During a contraction stress test, the nurse notices late decelerations with three of the six contractions. These results are classified as: a. Negative. b. Positive. c. Equivocal. d. Unsatisfactory.

b. Positive - Positive results are when late decelerations accompany at least 50% of the contractions.

A lactating woman asks the nurse about contraception. She states that she has always used a combination pill and would like to continue with that method. The nurse should advise the woman that: a. Oral contraceptives are contraindicated during the lactation period. b. Progestin-only contraceptives may be started 6 weeks after birth. c. Combination contraceptive pills may be started 3 to 4 weeks after birth. d. Only barrier methods are recommended during the lactation period.

b. Progestin-only contraceptives may be started 6 weeks after birth. Correct -Combination oral contraceptives reduce milk production in lactating women, and very small amounts may be transferred to the milk. Progestin-only contraceptives may be a better choice if a woman wishes to use a hormonal contraceptive because they do not affect milk production. They are often started 6 weeks after birth.

When placing a newborn under a radiant heat warmer to stabilize the temperature after birth, the nurse should: A place the thermistor probe on the left side of the chest. B cover the probe with a nonreflective material. C recheck the temperature by periodically taking a rectal temperature. D prewarm the radiant heat warmer and place the undressed newborn under it.

D The thermistor probe should be placed on the upper abdomen away from the ribs. It should be covered with reflective material. Rectal temperatures should be avoided since rectal thermometers can perforate the intestine, and the temperature may remain normal until cold stress is advanced. The radiant warmer should be prewarmed so the infant does not experience more cold stress.

When monitoring a woman in labor who has just received spinal analgesia, the nurse should report which assessment findings to the health care provider? (Select all that apply.) A Maternal blood pressure of 108/79 B Maternal heart rate of 98 C Respiratory rate of 14 breaths/min D Fetal heart rate of 100 beats/min E Minimal variability on a fetal heart monitor

D, E After induction of the anesthetic, maternal blood pressure, pulse, and respirations and fetal heart rate and pattern must be checked and documented every 5 to 10 minutes. If signs of serious maternal hypotension (e.g., the systolic blood pressure drops to 100 mm Hg or less or the blood pressure falls 20% or more below the baseline) or fetal distress (e.g., bradycardia, minimal or absent variability, late decelerations) develop, emergency care must be given.

Perineal care is an important infection control measure. When evaluating a postpartum woman's perineal care technique, the nurse would recognize the need for further instruction if the woman: a. uses soap and warm water to wash the vulva and perineum b. washes from symphysis pubis back to episotomy c. changes her perineal pad every 2-3 hours d. uses the peribottle to rinse upward into her vagina

D. The peribottle should be used in a backward direction over the perineum. The flow should never be directed upward into the vagina since debris would be forced upward into the uterus through the still open cervix.

When placing a newborn under a radiant heat warmer to stabilize temperature after birth, the nurse should: a. place a thermistor probe on the left side of the chest b. cover probe with a nonreflective material c. recheck temperature by periodically taking a rectal temperature d. prewarm the radiant heat warmer and place the undressed newborn under it.

D. The radiant warmer should be prewarmed so the infant does not experience more cold stress

Which finding would be a source of concern if noted during the assessment of a woman who is 12 hours postpartum? a. postural hypotension b. tem of 100.4 F c. bradycardia - pulse 55 BPM d. pain in left calf with dorsiflexion of left foot

D. These findings indicate a positive Homan's sign and are suggestive of thrombophlepitis and should be investigated.

The etiology component of the nursing diagnosis describes: a. projected changes in an individual's health status, clinical conditions, or behavior b. an individual's response to health pattern deficits in the child, family, or community c. a cluster of cues and/or defining characteristics that are derived from patient assessment and indicate actual health problems d. physiologic, situational, and maturational factors that cause the problem or influence its development

D. This is a definition of etiology, the second component of the nursing diagnosis

A woman with severe preeclampsia is being treated with an IV infusion of magnesium sulfate. This treatment will be evaluated as successful if: a. Blood pressure is reduced to prepregnant baseline. b. Seizures do not occur. c. Deep tendon reflexes become hypotonic. d. Diuresis reduces fluid retention.

b. Seizures do not occur. Correct - Magnesium sulfate is a CNS depressant given primarily to prevent seizures. A temporary decrease in blood pressure can occur, but is not the purpose for giving the medication. Hypotonia is a sign of an excessive serum level of magnesium, as is depression of respiratory activity. If is critical that calcium gluconate be on hand to counteract the depressant effects of magnesium toxicity. Diuresis is not a priority outcome of magnesium sulfate administration.

A woman has just been admitted to the maternity unit with a diagnosis of incomplete abortion. The physician has written the following orders: (1) NPO (2) Type and crossmatch for two units of blood. (3) Start intravenous line and run Ringer's lactate at 150 mL/hr. (4) Administer Pitocin, 10 units intramuscular. (5) Acetaminophen and codeine (Tylenol with Codeine #3), every 3 to 4 hours as needed for pain (6) Bed rest with bathroom privileges Which order should the nurse carry out first for this patient? a. Inform her of the NPO and bed rest order. b. Start the IV and draw blood to send for the type and crossmatch. c. Administer the pain medication. d. Take time to listen to the client about her feelings concerning the abortion.

b. Start the IV and draw blood to send for the type and crossmatch. Correct -Initial treatment of an incomplete abortion should focus on stabilizing the woman cardiovascularly. She may have lost blood or is at high risk for blood loss, so it is important to have her typed and crossmatched for replacement blood. The IV will help with fluid replacement.

A 26-year-old woman is considering medroxyprogesterone acetate (Depo-Provera) as the form of contraception that is best for her. To assist this woman with making a decision concerning this method of contraception, the nurse would tell her that Depo-Provera: a. Is inserted subcutaneously into the upper inner arm. b. Would require that she return to the clinic every 3 months. c. Should not be used if she has a history of estrogen-sensitive cancer. d. Is effective for 3 years.

b. Would require that she return to the clinic every 3 months. Correct - Depo-Provera is an injectable progestin that prevents ovulation for 12 weeks. This requires the woman to return to the clinic every 3 months for the injection.

A woman who's 36 weeks pregnant comes into the labor & delivery unit with mild contracts. Which of the following complications should the nurse watch out for when the client informs her that she has placenta prevue? a. sudden rupture of membranes b. vaginal bleeding c. emesis d. fever

b. vaginal bleeding contractions may disrupt the microvascular network in the placenta of a client with placenta prevue and result in bleeding. If the separation of the placenta occurs at the margin of the placenta, the blood will escape vaginally. Sudden rupture of the membranes isn't related to placenta prevue. Fever would indicate an infectious process, and emesis isn't related to placenta previa

Several noted health risks are associated with menopause. These risks include all except: osteoporosis. coronary heart disease. breast cancer. obesity.

breast cancer Osteoporosis is a major health problem in the United States. It is associated with an increase in hip and vertebral fractures in postmenopausal women. A woman's risk of developing and dying of cardiovascular disease increases significantly after menopause. Breast cancer may be associated with the use of hormone replacement therapy for women who have a family history of breast cancer. Women tend to become more sedentary in midlife. The metabolic rate decreases after menopause, which may require an adjustment in lifestyle and eating patterns.

When providing health education to the client, the nurse understands that an example of the secondary level of prevention is: approved infant car seats. breast self-examination (BSE). immunizations. support groups for parents of children with Down syndrome.

breast self-examination (BSE) Infant car seats are an example of primary prevention. BSE is an example of secondary prevention, which includes health screening measures for early detection of health problems. Immunizations are an example of primary prevention. Support groups are an example of tertiary prevention, which follows the occurrence of a defect or disability (e.g., Down syndrome).

A woman has just been diagnosed as being 2 months pregnant. She is upset and states, "This is not possible, my husband had a vasectomy 3 months ago, and he should be sterile." The nurse should respond to this woman with the knowledge that a vasectomy: a. Is not effective in all men. b. Can undo itself within the first 3 months after surgery. c. Does not render a man sterile for about 3 months. d. Does result in sterility, so there should be another explanation concerning the pregnancy.

c. Does not render a man sterile for about 3 months. Correct -Following a vasectomy, complete sterilization does not occur until sperm are no longer present in the semen. This may be 3 months or longer.

When doing a chart review of a client with preeclampsia, the nurse noted that the client was assessed with +3 edema. This indicates: a. Minimal edema of the lower extremities. b. Marked edema of lower extremities. c. Edema of lower extremities, face, hands, and sacral area. d. Generalized massive edema that includes ascites.

c. Edema of lower extremities, face, hands, and sacral area. Correct - The +3 assessment of edema indicates edema of the lower extremities, face, hands, and sacral area. Option A indicates A +1 grade, option B indicates a +2 grade, and option D indicates a +4 grade.

A woman has come to the clinic for a preconception visit. It is important to teach this woman to increase her intake of which vitamin prior to conception? a. Vitamin C b. Vitamin B12 c. Folic acid d. Niacin

c. Folic acid Correct - There is an association between folic acid deficiency and neural tube defects. Therefore, it is recommended that all women of childbearing age take 400 mcg of folic acid (pregnancy amounts) daily to reduce this risk.

A woman is born without a functioning uterus but has functioning ovaries. She and her husband come to the infertility clinic requesting information on the possibility of having a baby. One possibility for this couple to have a biological child is: a. In vitro fertilization. b. Therapeutic insemination. c. Gestational surrogate. d. Gamete intrafallopian transfer.

c. Gestational surrogate. Correct - A surrogate mother in a gestational surrogate pregnancy supplies her uterus only, with the infertile couple supplying the sperm and ovum. The other choices all require a functioning uterus in the biologic mother.

During pregnancy a woman has an indirect Coombs test done. The nurse can teach her that this test will show: a. Her Rh factor. b. Her previous exposure to certain viral infections. c. Her previous exposure to Rh-positive blood. d. Her blood type.

c. Her previous exposure to Rh-positive blood. Correct -Rh-negative women should have an indirect Coombs test to determine whether they are sensitized (have developed antibodies) as a result of previous exposure to Rh-positive blood.

Self-care instructions for a woman following a modified radical mastectomy would include that she: wears clothing with snug sleeves to support her affected arm. use depilatory creams instead of shaving the axilla of her affected arm. expect a decrease in sensation or tingling in her affected arm as her body heals. empty surgical drains once a day or every other day.

expect a decrease in sensation or tingling in her affected arm as her body heals Loose clothing should be worn since tight clothing could impede circulation in the affected arm. The axilla of the affected arm should not be shaved nor should depilatory creams or strong deodorants be used. A decrease in sensation and tingling in the affected arm and in the incision are expected for weeks to months after the surgery. Drains should be emptied at least twice a day and more often if necessary.

Fundal height at 16 weeks

halfway between the umbilicus and the symphysis pubis

The two most frequently reported maternal medical risk factors are: hypertension associated with pregnancy and diabetes. drug use and alcohol abuse. homelessness and lack of insurance. behaviors and lifestyles.

hypertension associated with pregnancy and diabetes These are the most frequently reported maternal risk factors. Both are associated with obesity. Approximately 20% of U.S. women who give birth are obese. Obesity in pregnancy is associated with the use of increased health care services and longer hospital stays. Both drug and alcohol use continue to increase in the maternal population. These are associated with low-birth-weight infants, mental retardation, and birth defects. The number of these clients are increasing; however, these are not the most common risks. Behavior and lifestyle choices do contribute to the health of the mother and fetus.

Human placental lactogen

increases amount of circulating free fatty acids of maternal metabolic needs and decreases maternal metabolism of glucose to favor fetal growth

Relaxin

inhibits uterine activity, diminishes strength of uterine contraction, aids in softening of the cervix and has long term effect of remodeling collagen

An infertile woman is about to begin pharmacologic treatment. As part of the regimen, she will take purified follicle-stimulating hormone (FSH) (urofollitropin [Metrodin]). The nurse instructs her that this medication is administered in the form of a/an: intranasal spray. vaginal suppository. intramuscular injection. tablet.

intramuscular injection. Intranasal spray is not the appropriate route for urofollitropin. Vaginal suppository is not the correct route for urofollitropin. Urofollitropin is given by IM injection; the dosage may vary. Urofollitropin cannot be given by tablet; it is given only by IM injection.

Conduction heat loss

is the loss of heat from the BODY SURFACE to COOLER SURFACES in direct contact

RADIATION HEAT LOSS

is the loss of heat from the body surface to COOLER SOLID SURFACES NOT in direct contact but in relative proximity

With regard to dysfunctional uterine bleeding (DUB), the nurse should be aware that: it is most commonly caused by anovulation. it most often occurs in middle age. the diagnosis of DUB should be the first considered for abnormal menstrual bleeding. the most effective medical treatment involves steroids.

it is most commonly caused by anovulation. Anovulation may occur because of hypothalamic dysfunction or polycystic ovary syndrome. DUB most often occurs when the menstrual cycle is being established or when it draws to a close at menopause. A diagnosis of DUB is made only after all other causes of abnormal menstrual bleeding have been ruled out. The most effective medical treatment is oral or intravenous estrogen.

With regard to the diagnosis and management of amenorrhea, nurses should be aware that: it probably is the result of a hormone deficiency that can be treated with medication. it may be caused by stress or excessive exercise or both. it likely will require the client to eat less and exercise more. it often goes away on its own.

it may be caused by stress or excessive exercise or both. Amenorrhea may be the result of a decrease in follicle-stimulating hormone (FSH) and luteinizing hormone (LH). This is usually caused by stress, body fat to lean ratio, and in rare occurrences a pituitary tumor. It cannot be treated by medication. Amenorrhea usually is the result of stress and/or an inappropriate ratio of body fat to lean tissue, possibly as a result of excessive exercise. Management includes counseling and education about the causes and possible lifestyle changes. In most cases a client will need to decrease her amount of exercise and increase her body weight in order to resume menstruation. Management of stress and eating disorders is usually necessary to manage this condition.

Progesterone

maintains the endometrium and inhibits spontaneous uterine contractility (prevents abortion due to uterine activity)

The normal term infant has little difficulty clearing its airway at birth. Most secretions are brought up to the oropharynx by the cough reflex. However, if the infant has excess secretion, the mouth and nasal passages can be cleared easily with a bulb syringe. When instructing the parents on the correct use of the piece of equipment it is important to teach the parents to suction

mouth first

Discharge instructions after tubal ligation should include: (Select all that apply.) being prepared for significant mood swings due to hormonal influences. expecting heavier menstrual periods. using two forms of birth control to prevent pregnancy. not expecting change in sexual functioning; may enjoy more. using condoms to prevent sexually transmitted infections.

not expecting change in sexual functioning; may enjoy more. using condoms to prevent sexually transmitted infections. Patient teaching regarding what to expect after tubal ligation includes: • You should expect no change in hormones and their influence. • Your menstrual period will be about the same as before the sterilization. • You may feel pain at ovulation. • The ovum disintegrates within the abdominal cavity. • It is highly unlikely that you will become pregnant. • You should not have a change in sexual functioning; you may enjoy sexual relations more because you will not be concerned about becoming pregnant. • Sterilization offers no protection against sexually transmitted infections. Therefore you may need to use condoms.

A newborn is jaundices and receiving phototherapy via ultraviolet bank lights. An appropriate nursing intervention when caring for an infant with hyperbilirubinemia and receiving phototherapy by this method would be to

place eye shields over the newborn's eyes

Providing treatment and rehabilitation for people who have developed disease is part of: primary preventive care. secondary preventive care. tertiary preventive care. primordial preventive care.

tertiary preventive care Primary preventive care involves promoting healthy lifestyles. Secondary preventive care involves targeting populations at risk. Tertiary preventive care is the treatment or rehabilitation of those who already have a specific disease. Primordial preventive care refers to prevention of the risk factors themselves at either the social or environmental level.

As part of standard precautions, nurses wear gloves when handling the newborn, The chief reason is:

To protect the nurse from contamination by the newborn

Which of the following use/abuse during pregancny causes vasoconstriction and decreased placental perfusion, resulting in maternal and neonatal complications?

Tobacco

A woman during her first trimester questions how much longer she will have urinary frequency. Which of the following statements would be the most accurate response?

Urinary frequency is a common complaint during the first trimester and again in the third

When the services of an interpreter are used, which of the following should the nurse do?

Use an interpreter who is certified and document the person's name in the nursing notes

A 25 year old G2T2P0A0L2 gave birth 4 hours ago to a 9 lb 7 oz boy after augmentation of labor w/ Pitocin. She calls for the nurse and says "I'm bleeding a lot." What is the most likely cause of postpartum hemorrhage?

Uterine atony

Fennel, dong quai

Uterotonic

Which symptom is considered a first-trimester warning sign and should be reported immediately by the pregnancy woman to her health care provider

Vaginal bleeding

Semen analysis is a common diagnostic procedure related to infertility. In instructing a male patient regarding this test, the nurse would tell him to: ejaculate into a sterile container. obtain the specimen after a period of abstinence from ejaculation of 2 to 5 days. transport specimen with container packed in ice. ensure that the specimen arrives at the laboratory within 30 minutes of ejaculation.

abstinence from ejaculation of 2 to 5 days. The male must ejaculate into a clean container or a plastic sheath that does not contain a spermicide. An ejaculated sample should be obtained after a period of abstinence to get the best results. He should avoid exposing the specimen to extremes of temperature, either heat or cold. The specimen should be taken to the laboratory within 2 hours of ejaculation.

Prenatal testing for the human immunodeficiency virus HIV is recommended for which women

all women regardless of risk factors

A woman is being counseled concerning the calendar method type of natural family planning. The woman states that her cycles run from 27 to 29 days. The nurse teaches the woman that ovulation will probably occur on about days: a. 10 to 12. b. 13 to 15. c. 16 to 18. d. 18 to 20.

b. 13 to 15. Correct - Ovulation occurs approximately 14 days before the onset of menses.

A woman is 16 weeks pregnant and has elected to terminate her pregnancy. The nurse knows that the most common technique used for medical termination of a pregnancy in the second trimester is:

Dilation and evacuation (D&E).

A pregnant woman has been receiving magnesium sulfate infusion for tx. of preeclampsia for 24 hr., on assessment temp 37.3 degree c, pulse rate 88 bpm respiratory rate of 10 breaths/min blood pressure of 128/90 mm hg absent deep tendon reflexes and no ankle clonus the pt. complains I'm so thirsty and warm"

Discontinue the magnesium sulfate infusion

The nurse is providing genetic counseling to an expectant couples who already gave a child with trisomy 18, the nurse should

Discuss options with the couple, including amniocentesis to determine whether the fetus is affected

Appendicitis may be difficult to diagnose in pregnancy because the appendix is

Displaced upward and laterally, high and to the right

Fibrocystic changes in the breast most often appear in women in their 20s and 30s. The etiology is unknown, but it may be an imbalance of estrogen and progesterone. The nurse who cares for this client should be aware that treatment modalities are conservative. One proven modality that may provide relief is:

Diuretic administration.

A 36-week gestation woman lies flat on her back. Which of the following maternal signs/ symptoms would not be unanticipated?

Diziness

Which test used to diagnose the basis of infertility is done during the luteal or secretory phase of the menstrual cycle?

Endometrial biopsy

Patient care information:

Ensure that information is documented in a timely manner.

In which culture is the father more likely to be expected to participate in the labor and delivery? Asian-American African-American European-American Hispanic

European-American Asian-American fathers do not actively participate in labor or birth. African-American men view pregnancy as a sign of virility; however, they may be less likely to participate actively in labor or birth.

You are preparing an injection of a narcotic to relieve pain. You note at her last vag exam (1 hour ago) she was 8 cm. As you are about to give it, she asks for a bedpan to move her bowels. What would be your first response?

Evaluate the woman's labor progress then if appropriate, give the injection

During a health history interview, a woman states that she thinks that she has "bumps" on her labia. She also states that she is not sure how to check herself. The correct response would be to:

Explain the process of vulvar self-examination to the woman and reassure her that she should become familiar with normal and abnormal findings during the examination.

With what heart conditions is pregnancy not usually contraindicated

Heart transplant

As part of their participation in the gynecologic portion of the physical examination, nurses should:

Help the woman relax through proper placement of her hands and proper breathing during the examination.

What is the most critical nursing action in caring for the newborn immediately after birth?

Keeping the newborn's airway clear

Maternity nursing care that is based on knowledge gained through research and clinical trials, clinical expertise and patient/family preferences is which of the following?

Known as evidence-based practice

Maternity nursing care that is based on knowledge gained through research and clinical trials is:

Known as evidence-based practice.

When assessing a patient for amenorrhea, the nurse should be aware that this is unlikely to be caused by:

Lack of exercise.

Many pregnant teens wait until the second or third trimester to seek prenatal care. The nurse should understand that the reasons behind this delay include:

Lack of realization that they are pregnant. Uncertainty as to where to go for care. Continuing to deny the pregnancy. Continuing to deny the pregnancy.

With regard to prenatal genetic testing, nurses should be aware

Maternal serum screening can determine whether a pregnant woman is at risk of carrying a fetus with down syndrome

A lecithin; sphignomyelin (L/S) ratio has been ordered, What will be learned from the test?

Maturation of fetal lungs

Fundal height at 20 weeks

Midline and at the umbilicus

An African-American woman noticed some bruises on her newborn's butt. The nurse explains these marks are called what?

Mongolian spots

A newborn admitted to nursery has a positive direct Coomb's test. Which of the following is an appropriate action by the nurse?

Monitor baby for jaundice

Quadruple screen/MSAFP

Performed on mother's serum between 15-21 weeks. Detect levels of AFP. If high AFP can be risk of neural tube defect, if low AFP then risk of having Down syndrome and trisomy 18

There is little consensus on the management of premenstrual dysphoric disorder (PMDD). However, nurses can advise women on several self-help modalities that often improve symptoms. The nurse knows that health teaching has been effective when the client reports that she has adopted a number of lifestyle changes, including (Select all that apply):

Regular exercise. Improved nutrition. Smoking cessation. Oil of evening primrose.

The volume of amniotic fluid is an important factor in assessing fetal well-being. Oligohydramnios is associated with what kind of fetal anomalies

Renal

Nursing Response to Lower Extremities Assessment Postpartum

Report to physician or certified nurse midwife

Which of the following nursing diagnoses would be the most appropriate for a client experiencing severe preeclampsia?

Risk for injury to mother and fetus related to CNS irritability

The 2 primary areas of risk for sexually transmitted infections (STIs) are

Risky sexual behavior and inadequate preventive health behaviors

The two primary areas of risk for sexually transmitted infections (STIs) are:

Risky sexual behaviors and inadequate preventive health behaviors.

The lab results of postpartum woman are as follows: blood type A, Rh status positive (so A+), rubella nonimmune, hematocrit 32%. How should the nurse interpret?

Rubella vaccine should be given

Paul, a first time father, is changing the diaper of his 1-day old and asks the nurse "What is the black, sticky stuff in her diaper?" What is the nurse's best response?

That's meconium, which is your baby's first stool. It's normal.

Unique muscle fibers make the uterine myometrium ideally suited for:

The birth process.

The nurse who is teaching a group of women about breast cancer would tell the women that:

The exact cause of breast cancer is unknown.

With regard to hemolytic disease of the newborn, nurses should be aware that

The indirect Coombs test is done on the mother on the cord blood after birth

The perinatal nurse is giving discharge instructions to a woman, status post suction curettage secondary to a hydatidiform mole. The woman asks why she must take oral contraceptives for 12 months. Which is the best response?

The major risk to you after a molar pregnancy is a type of cancer that can be diagnoses only by measuring the same hormone that your body produces during pregnancy. If you were to get pregnant it would make the diagnosis of this cancer more difficult

FHR baseline

The mean FHR during a 10 minute period., rounded in increments of 5 bpm

In practical terms regarding genetic health care, nurses should be aware that

The most important of all nursing function is providing support to the family during counseling

With regard to factors that effect how the fetus moves through the birth canal, nurses should be aware that which of the following is correct?

The normal attitude of the fetus is called general flexion

Which of the following statements indicate that the nurse is practicing appropriate family-centered care techniques (Select all that apply)?

The nurse allows time for the partner to ask questions The nurse allows the mother and father to make choices when possible.

A woman is to have an Essure procedure performed to produce sterilization. The nurse should include in the client teaching that: a. This procedure requires minor surgery and will be done in the outpatient surgical unit. b. The woman should use another form of birth for 3 months after the procedure c. The woman should rest for 24 hours after the procedure and should not lift heavy objects for a week. d. Narcotic analgesics will be prescribed for pain control after the procedure.

b. The woman should use another form of birth control for 3 months after the procedure. Correct - With the Essure procedure, a tiny coil is inserted into each fallopian tube. The tubes become permanently blocked during the next 3 months as tissue grows into the inserts. During this time, another contraceptive method is used. The procedure can be performed in the physician's office. It is a nonsurgical method of sterilization.

To increase the effective rate of male condoms, the female partner may: a. Douche after intercourse. b. Use a spermicide. c. Use a female condom. d. Use an oil-based lubricant.

b. Use a spermicide. Correct - Using spermicides with condoms increases lubrication, which decreases the risk of condom breakage. Effectiveness is increased when spermicides are used with condoms.

The nurse determines that teaching about contraceptives is effective when the client makes which statement? a. "The more the contraceptives cost, the more effective they will be." b. "The birth control pill has a medication in it to prevent most STDs." c. "Condoms have very few side effects." d. "I will use the contraceptive method that my husband prefers."

c. "Condoms have very few side effects." Correct - Condoms will have local side effects only, such as allergic reaction. They will not produce systemic effects, as will the hormone type of contraception. Cost does not alter the effectiveness of a method. Oral hormone contraceptives do not contain other medications. The woman usually makes the final decision about her contraceptive method, and her satisfaction with the choice is crucial.

After a year of infertility treatment, a woman has just begun another menstrual period. She tells the nurse, "I am so tired of trying. It feels like I will never have a baby. I just am not a real woman." The best response by the nurse would be: a. "Remember, not everyone receiving treatment will become pregnant. We told you that at the very beginning of the treatments." b. "Don't give up yet. You are still young and we are learning newer techniques to try every day." c. "This must be very frustrating to you." d. "Having a baby does not make you a woman."

c. "This must be very frustrating to you." Correct - Infertility treatment is stressful. The woman may experience depression and guilt. The best response by the nurse is to allow the woman time to express her concerns and feelings.

A woman is requesting information concerning contraception. She is sexually active with multiple partners and is concerned about sexually transmitted diseases. When doing client teaching, the nurse should be aware that the contraceptive method that offers the most protection against sexually transmitted diseases is: a. Oral hormones. b. An intrauterine device. c. A male condom. d. Natural birth control.

c. A male condom. Correct -The male condom offers the best protection from sexually transmitted diseases because it is a barrier.

An effective relief measure for primary dysmenorrhea would be to: reduce physical activity level until menstruation ceases. begin taking prostaglandin synthesis inhibitors on the first day of the menstrual flow. decrease intake of salt and refined sugar about 1 week before menstruation is about to occur.

decrease intake of salt and refined sugar about 1 week before menstruation is about to occur. use barrier methods rather than the oral contraceptive pill (OCP) for birth control. Staying active is helpful since it facilitates menstrual flow and increases vasodilation to reduce ischemia. Prostaglandin inhibitors should be started a few days before the onset of menstruation. Decreasing intake of salt and refined sugar can reduce fluid retention. OCPs are beneficial in relieving primary dysmenorrhea as a result of inhibition of ovulation and prostaglandin synthesis.

Leopold's Maneuvers 3rd Maneuver

determine what fetal part is lying above the inlet

Leopold's Maneuvers 2nd Maneuver

determines fetal back and if on R or L side

The CDC-recommended medication for the treatment of chlamydia would be: doxycycline. podofilox. acyclovir. penicillin.

doxycycline. Doxycycline is effective for treating chlamydia, but it should be avoided if the woman is pregnant. Podofilox is a recommended treatment for nonpregnant women diagnosed with human papilloma virus infection. Acyclovir is recommended for genital herpes simplex virus infection. Penicillin is not a CDC-recommended medication for chlamydia; it is the preferred medication for syphilis.

A first-time mother is concerned about the type of medications she will received during labor she is in a fair amount of pain

sedatives can be used to reduce the n/v that often accompany opioid use in addition some ataractics reduce anxiety and apprehension and potentiate the opioid analgesic affects

A nurse might be called on to stimulate the fetal scalp

to elicit an acceleration in FHR

The nurse providing care for the laboring woman should understand that variable fetal heart rate (FHR) decelerations are caused by

umbilical cord compression

Role of vitamin k in a neonate

vitamin K, deficient in the neonate, is needed to activate clotting factors II,IV,IX and X In the event of trauma, the neonate would be at risk for EXCESSIVE BLEEDING vitamin k doesnt assist the gut to mature but the gut PRODUCES vitamin K once maturity is achieved

When using intermittent auscultation IA for fetal heart rate nurses should be aware that:

you can order an ultrasound and look for the best place to listen for FHR

A physician prescribes clomiphene citrate (Clomid, Serophene) for a woman experiencing infertility. She is very concerned about the risk of multiple births. The nurse's most appropriate response is:

"This is a legitimate concern. Would you like to discuss this further before your treatment begins?"

Thermal Changes

- A significant decreased in ambient temperature after birth - from 98.6 to 70-75 F results in sudden chilling of the moist newborn. The cold stimulus skin sensory receptors and the newborn responds with rhythmic inspirations. -Excessive cooling may result in profound respiratory depression and evidence of cold stress

The physician has ordered a chorionic villus sampling to be done on a client. The nurse knows to schedule the test to be done between which weeks of gestation? a. 4 and 6 weeks b. 6 and 8 weeks c. 8 and 10 weeks d. 10 and 12 weeks

- CVS is usually performed between 10 and 12 weeks of gestation. At this time the chorionic villi have been formed significantly to obtain adequate samplings.

7 cardinal movements

- Descent - Flexion - Internal rotation - Extension - Restitution - External rotation - Expulsion

Physical Changes Postpartum: Vital signs

-A maternal temp of 100.4 may occur. -Immediately following delivery, many women experience a transient rise in both systolic and diastolic blood pressure. -WBC may increase. Resp increase as well as heart rate. Hemoglobin and hemoatocrit levels increase.

Presumptive signs of Pregnancy

-Amenorrhea -Breast tenderness -nausea and vomiting -Urinary frequency -Excessive fatigue

Physiological Jaundice

-Appears after first 24 hours of birth -RBC excess and shorter lifespan -Liver immaturity -Intestinal Factors - Sterile -Delayed Feedings

Mechanical Changes

-As the fetus experiences labor, there is a fetal grasp and active exhalation that initiates the removal of fluid from the lungs. -During the vaginal birth process the detail chest is compressed, increasing intrathoracic pressure and squeezing a small amount of fluid out of the lungs. At delivery the chest wall recoils, creating negative intrathoracic pressure, which is thought to produce a small, passive inspiration of air that replaces of air that replaces the fluid that was squeezed out. This is called the thoracic squeeze. -After the first inspiration, the newborn exhales and cries against a partially closed glottis, creating a positive intrathoracic pressure. This begins the FRC.

Circumcision Assessment and Care - Clamp

-Bleeding is main concern in first initial hour -Comfort/Pain -Infection -Use jelly to keep from sticking to diaper

Vital signs assessment postpartum

-Blood pressure: should remain consistent with baseline BP during pregnancy. -High BP - preeclampsia, essential hypertension, renal disease, anxiety -Drop in BP - may be normal, uterine hemorrhage -Pulse: 50-90 bpm; may be brady cardia if 50-70 bpm -Tachycardia - difficult labor and birth, hemorrhage -Respirations: 16 - 24 per minute -Marked tachypnea - respiratory disease -Diminished respirations - long action epidural narcotics -Temperature: 98.0 - 100.4 -After the first 24 hours, temperature of 100.4 or above suggests infection

Criteria and duration for the second stage of labor

-Cervical dilation is complete -Contractions every 1.5 to 2 minutes -Woman may feel out of control; helplessness, panic, or may be happy that she can take a more active role - PUSHING! -position change of fetus may occur - 7 cardinal movements

Criteria and duration of the latent phase of labor

-Cervix is 0 to 3 cm dilated -Contractions are every 10-20 minutes, lasting 15-20 seconds with mild intensity -They progress to every 5-7 minutes, lasting 30-40 seconds with moderate intensity -The woman is usually talkative and eager to be in labor -Exhibits need for independence by taking care of own bodily and seeking information

Criteria and duration of the active phase of labor

-Cervix is 4 to 7 cm dilated -Contractions every 2 to 3 minutes lasting 50 to 60 seconds with moderate to strong intensity. -Woman may experience feelings of helplessness, exhibits increased fatigue, and may begin to feel restless, and anxious as contraction become stronger -Expresses fear of abandonment, becomes more dependent and is less able to meet her needs.

Risk Factors for DVT

-Cesarean birth -Immobility -Obesity -Smoker -Previous thromboembolic disease -Trauma to extremity -Varicose veins -Diabetes mellitus -Advanced maternal age -Inherited coagulation disorders -Anemia -Exogenous hormone use -Malignancy -Inherited coagulation pathyway deficiency -Increased amounts of certain blood clotting factors -Postpartal thrombocytosis -Release of thromboplastin substances from the tissue of the decidua, placenta, and fetal membranes -Increased amounts of fibrinolysis inhibitors -Changes in the woman's coagulation system in pregnancy contribute to DVT

Probable signs of pregnancy

-Chadwick's Sign -Goodell's sign -Enlarged Uterus -Hegar's Sign -McDonald's Sign -Ladin's Sign -Enlargement of the abdomen -Uterine souffle -Changes in pigmentation of skin -Braxton Hicks contractions

Nursing care for Variable decelerations

-Change mom's position -Do a vaginal exam to make sure cord has not fallen through -STOP PITOCIN

Signs of False Labor

-Contractions are irregular -No change within contractions -Discomfort is usually in abdomen -Walking has no effect on or lessens contractions -No change within cervix -Rest and warm bath lessens contractions

Fetal Presentation

-Determined by fetal lie and refers to the body part of the fetus that enters the maternal pelvis first and leads through the birth canal during labor.

Nursing care for the latent phase of labor

-Establish rapport on admission and continue to build during care -Assess information bases and learning needs -Be available to consult regarding breathing technique if needed -Orient family to room, equipment, monitors, and procedures -Encourage woman and partner to participate in care as desired -Assist woman into position of comfort -Encourage frequent change of position -Encourage ambulation during early labor -Offer fluid and ice chips -Keep couple informed of progress -Encourage woman to void every 1-2 hours -Assess need for an interest of visualization to enhance relaxation.

Nursing Response to Vital Signs Postpartum

-Evaluate history of preexisting disorders and check for other signs of preeclampsia (edema, proteinuria). -Assess for other signs of hemorrhage (increased pulse, cool clammy skin) -Evaluate for other signs of hemorrhage -Assess for other signs of respiratory disease -Assess for other signs of infection; notify physician or certified midwife

Common Discomforts of pregnancy in the second and third trimesters

-Heartburn (pyrosis) -Ankle edema -Varicose veins -Flatulence -Hemorrhoids -Constipation -Backache -Leg cramps -Faintness -Shortness of breath -Difficulty sleeping -Restless leg syndrome -Round ligament pain -Carpal tunnel syndrome

Respiratory Adaptions of newborn

-Mechanical -Chemical -Thermal -Sensory

Common discomforts of pregnancy in the first trimester

-Nausea and vomiting -Urinary frequency -Fatigue -Breast tenderness -Increased vaginal discharge -Nasal stuffiness and epistaxis (nosebleed) -Pytalism

Lower Extremities Assessment Postpartum

-No pain with palpation -Negative Homan's sign - Positive findings - thrombophlebitis

Circumcision Assessment and Care - Plastibell

-Plastic rim cuts off circulation and eventually falls off -No bleeding involved. -Rim falls off 3-5 days removed after days if still intact

Risk Factors for hematoma

-Preeclampsia -Use of pudendal anesthesia -First full term baby -Precipitous labor -Prolonged second stage of labor -Macrosomia -Vacuum assisted delivery -History of vulvar varicosities

Fetal Position

-Refers to the relationship of the landmark on the presenting fetal part to the anterior, posterior, or sides of the maternal pelvis. -The landmark on the fetal presenting part is related to four imaginary quadrants of the maternal pelvis: • Left anterior • Right anterior • Left posterior • Right posterior -These quadrants designate where the presenting part is directed. -Three notations are used to describe the fetal position: • Right (R) or Left (L) side of the maternal pelvis. -The landmark of the fetal presenting part: occiput (O), mentum (M), sacrum (S), or acromion process (A) • Anterior (A), posterior (P), or transverse (T), depending on whether the landmark is the front, back or side of the pelvis.

Fetal Lie

-Refers to the relationship of the long axis of the fetus to the long axis of the mother ( spine to spine) -Longitudinal lie - occurs when the cephalocaudal axis is parallel to the mother's spine -Transverse lie - occurs when the cephalocaudal axis is at a right angle with the mother's spine

Pathological Jaundice

-Shows up in 1st 24 hours // ABNORMAL -Hemolytic Disease -Too many RED BLOOD CELLS -diabetic mom -Liver malfunction -Baby sleeps longer with jaundice—wake every 3-4 hours to feed

Perineum Assessment Postpartum

-Slight edema and bruising in intact perineum -Marked fullness, bruising, pain - vulvar hematoma -Episiotomy: no redness, edema, ecchymosis, or discharge; edges well approximated -Redness, edema, ecchymosis, discharge or gaping stitches - infection -Hemorrhoids: none present; if present should be small and nontender -Full, tender, inflamed hemorrhoids

Breasts: General Appearance

-Smooth, even pigmentation, changes of pregnancy still apparent; one may appear larger -Reddened area - mastitis

Assessment of Lungs Postpartum

-Sounds: clear to bases bilaterally -Diminished sounds - fluid overload, asthma, pulmonary embolus, pulmonary edema

Breasts: Nipples

-Supple, pigmented, intact; become erect when stimulated - Fissures, cracks, soreness - problems with breastfeeding -Not erectile with stimulation - inverted nipples

Vital signs for a newborn

-Temperature: 97.7-98.6 -Pulse: 120-160 bpm -Respirations: 30-60

Restitution

-The shoulders of the infant enter the pelvis obliquely and remain oblique when the head rotates to the anteroposterior diameter through internal rotation. -Because of this rotation the kneck becomes twisted. -Once the head emerges and is free of pelvic resistance the neck untwists, turning the head to one side, and aligns with the position of the back in the birth canal.

Criteria and duration of the fourth stage of labor

-The stage after the birth of the placenta, lasting 1-4 hours after or until vital signs become stable. -Maternal blood pressure should be monitored at 5-15 minutes intervals. The return of the blood pressure is due to an increased volume of blood returning to the maternal circulation from the uteroplacental shunt.

Facotrs that contribute to hemorrhage

-Uterine atony -Retained placenta -Distended bladder -Vaginal birth of larger baby -Distended uterus -Saturated pad within 15 minutes

Which of the following statements about dizygotic twins are true?

1. ... also known as fraternal twins 2. ... develop from two fertilized ova 3. ... can be different sexes

18) To help limit the development of hyperbilirubinemia in the neonate, the plan of care should include: 1. Monitoring for the passage of meconium each shift 2. Instituting phototherapy for 30 minutes every 6 hours 3. Substituting breastfeeding for formula during the 2nd day after birth 4. Supplementing breastfeeding with glucose water during the first 24 hours

1. 1. Bilirubin is excreted via the GI tract; if meconium is retained, the bilirubin is reabsorbed.

When newborns have been on formula for 36-48 hours, they should have a: 1. Screening for PKU 2. Vitamin K injection 3. Test for necrotizing enterocolitis 4. Heel stick for blood glucose level

1. 1. By now the newborn will have ingested an ample amount of the amino acid phenylalanine, which, if not metabolized because of a lack of the liver enzyme, can deposit injurious metabolites into the blood stream and brain; early detection can determine if the liver enzyme is absent.

When performing an assessment on a neonate, which assessment finding is most suggestive of hypothermia? 1. Bradycardia 2. Hyperglycemia 3. Metabolic alkalosis 4. Shivering

1. 1. Hypothermic neonates become bradycardic proportional to the degree of core temperature. Hypoglycemia is seen in hypothermic neonates.

11) A nursing instructor asks a nursing student to describe the procedure for administering erythromycin ointment into the eyes if a neonate. The instructor determines that the student needs to research this procedure further if the student states: 1. "I will cleanse the neonate's eyes before instilling ointment." 2. "I will flush the eyes after instilling the ointment" 3. "I will instill the eye ointment into each of the neonate's conjunctival sacs within one hour after birth." 4. "Administration of the eye ointment may be delayed until an hour or so after birth so that eye contact and parent-infant attachment and bonding can occur.."

1. 2. Eye prophylaxis protects the neonate against Neisseria gonorrhoeae and Chlamydia trachomatis. The eyes are not flushed after instillation of the medication because the flush will wash away the administered medication.

The nurse is aware that a neonate of a mother with diabetes is at risk for what complication? 1. Anemia 2. Hypoglycemia 3. Nitrogen loss 4. Thrombosis

1. 2. Neonates of mothers with diabetes are at risk for hypoglycemia due to increased insulin levels. During gestation, an increased amount of glucose is transferred to the fetus across the placenta. The neonate's liver cannot initially adjust to the changing glucose levels after birth. This may result in an overabundance of insulin in the neonate, resulting in hypoglycemia.

The nurse is aware that a healthy newborn's respirations are: 1. Regular, abdominal, 40-50 per minute, deep 2. Irregular, abdominal, 30-60 per minute, shallow 3. Irregular, initiated by chest wall, 30-60 per minute, deep 4. Regular, initiated by the chest wall, 40-60 per minute, shallow

1. 2. Normally the newborn's breathing is abdominal and irregular in depth and rhythm; the rate ranges from 30-60 breaths per minute.

The nurse decides on a teaching plan for a new mother and her infant. The plan should include: 1. Discussing the matter with her in a non-threatening manner 2. Showing by example and explanation how to care for the infant 3. Setting up a schedule for teaching the mother how to care for her baby 4. Supplying the emotional support to the mother and encouraging her independence

1. 2. Teaching the mother by example is a non-threatening approach that allows her to proceed at her own pace.

) A nurse in the newborn nursery is monitoring a preterm newborn infant for respiratory distress syndrome. Which assessment signs if noted in the newborn infant would alert the nurse to the possibility of this syndrome? 1. Hypotension and Bradycardia 2. Tachypnea and retractions 3. Acrocyanosis and grunting 4. The presence of a barrel chest with grunting

1. 2. The infant with respiratory distress syndrome may present with signs of cyanosis, tachypnea or apnea, nasal flaring, chest wall retractions, or audible grunts.

Which neonatal behavior is most commonly associated with fetal alcohol syndrome (FAS)? 1. Hypoactivity 2. High birth weight 3. Poor wake and sleep patterns 4. High threshold of stimulation

1. 3. Altered sleep patterns are caused by disturbances in the CNS from alcohol exposure in utero. Hyperactivity is a characteristic generally noted. Low birth weight is a physical defect seen in neonates with FAS. Neonates with FAS generally have a low threshold for stimulation

Within 3 minutes after birth the normal heart rate of the infant may range between: 1. 100 and 180 2. 130 and 170 3. 120 and 160 4. 100 and 130

1. 3. The heart rate varies with activity; crying will increase the rate, whereas deep sleep will lower it; a rate between 120 and 160 is expected.

Which of the following statements are true? (select all that apply)

1. The use of any drug should be avoided during pregnancy, unless its use is necessary 2. If meds need to be used, the lowest dose for the shortest duration should be chosen 3. Pregnant women should receive the flu vaccine if pregnant during flu season

Which blood pressure finding during 2nd trimester indicates risk for pregnancy induced hypertension

100/70 --> 130/85

While assessing the newborn he nurse should be aware that the average expected apical pulse range of a full-term quiet/alert newborn is

120-140 bpm

What is the average expected apical pulse range of a full-term, quiet, alert newborn?

120-160 beats/min

A multigravida reports she just started to feel the fetus move (quickening). This indicates she is most likely how many weeks pregnant?

16

How many vessels should be in the umbilical cord?

2 arteries and 1 vein

Fundal height at 14 weeks

2 finger-breadths above the symphysis pubis

A client is 33 weeks pregnant and has had diabetes since she was 21. When checking her fasting blood sugar level, which values indicate the clients disease was controlled. 1. 45 mg/dl 2. 85 mg/dl 3. 120 mg/dl 4. 136 mg/dl

2. 85 mg/dl recommended fasting blood sugar levels in pregnant clients w/ dm are 60-90 mg/dl a fasting blood sugar level of 45mg/dl is low and may result in hypoglyecemia a bs level below 120mg/dl is recommeded for 1hour postprandial values a bs level above 136 mg/dl in a pregnant client indicates hyperglycemia

a 21 y.o. client, 6 weeks pregnant, is diagnosed with hyperemesis gravidum. This excessive vomiting during pregnancy will often result in which of the following? 1. bowel perforation 2. electrolyte imbalance 3. miscarriage 4. PIH

2. ELECTROLYTE IMBALANCE Excessive vomiting in clients with hyperemesis grav often causes weight loss / fluid and electrolyte, acid base imbalance. PIH and bowel perforation arent r/t hyper grav the effects of hyper grav on the fetus depend on the severity of the disorder. clients w/ severe hyper grav may have low birth weight infant, but the disorder isnt life threatening to the fetus.

Which of the following is best to monitor a fetus of a client with diabetes in her 3rd trimester 1. US exam weekly 2. NST 2x/week 3. Daily contraction stress test at 32 weeks 4. monitoring fetal activity by client weekly

2. NST 2X/week NST is the preferred antepartum HR screening test for pregnant clients with diabetes. NSTs should be done at least 2x per week staring at 32 weeks gestation, as fetal deaths in clients w/ diabetes have been noted within 1 week of a reactive NST. US should be done ever 4-6 weeks to monitor fetal growth. CST wouldnt be initiated at 32 weeks Maternal fetal activity monitoring should be done daily

Rh isoimmunization in a pregnant client develops during which conditions? 1. Rh positive maternal blood crosses into fetal blood, stimulating fetal antibodies. 2. Rh positive fetal blood crosses into maternal blood, stimulating maternal antibodies. 3. Rh Negative fetal blood crosses into maternal blood, stimulating maternal antibodies. 4. Rh negative maternal blood crosses into fetal blood, stimulating fetal antibodies.

2. Rh positive fetal blood crosses into maternal blood, stimulating maternal antibodies. Rh isoimmunization occurs when Rh positive fetal blood cells cross into the maternal circulation and stimulate maternal antibody production. In subsequent pregnancies w/ Rh positive fetuses, maternal antibodies may cross back into the fetal circulation and destroy fetal blood cells.

A 17 y.o. primpigravida with severe PIH has been receiving mag sulfate IV for 3 hours. The latest assessment reveals DTR of +1, BP 150/100 mmgHg, pulse 92 bpm, respiratory rate 10bpm and urine output 20ml/hr. Which of the following actions would be most approp? 1. Continue monitoring per standards of care 2. Stop the mag sulfate infusion 3. Increase infusion by 5gtt/min 4. Decrease infusion by 5gtt/min

2. Stop the mag sulfate infusion Mag sulfate should be withheld if the clients resp rate or urine output falls or if reflexes are diminished / absent. The client also shows other signs of impending toxicity such as flushing / feeling warm. Inaction wont resolve the clients suppressed DTRs and low RR / urine output. The client is already showing CNS depression bc of excessive magsulfte so increasing the infusion is wrong. Impending toxicity indicates that the infusion should be stopped rather than just slowed down.

A client diagnosed with preterm labor at 28 weeks gestation. Later, she comes to the emergency dept saying "I think im in labor" The nurse would expect her physical exam to show which condition? 1. painful contractions with no cervical dilation 2. regular uterine contractions with cervical dilation 3. irregular uterine contractions with no cervical dilation 4. irregular uterine contractions with cervical effacement

2. regular uterine contractions (every 10 min or more) along with cervical dilation change before 36 weeks = PTL no cervical change with UC isnt' PTL

A woman of normal weight asks you what an ideal weight gain is during pregnancy. Which of the following should be your best answer?

25-35 pounds is ideal

A clients labor doesnt progress. After ruling out CPD, the doctor orders IV admin of 1,000 ml normal saline w/ Pitocin 10 units to run at 2 miliunits / min. 2 miliunits/min is equivalent to how many ml/unit 1. 0.002 2. 0.02 3. 0.2 4. 2.0

3 = 0.2 each unit of oxytocin contains 1,000 miliunits. Therefore, 1,000 ml of IV fluid contains 10,000 miliunits (10 units) of Pitocin

Which drug would the nurse choose to utilize as an antagonist for magnesium sulfate? 1. Oxytocin 2. Terbutaline 3. Calcium gluconate 4. Narcan

3. Calcium gluconate should be kept at the bedside while a client is recieivng mag sulfate infusion. If magnesium toxicity occurs, calcium gluconate is admined as an antidote oxytocin is the synthetic form of the naturally occurring pituitary hormone used to initiate or augment UCs Terbutaline is a beta2 adrenergic agonist that may be used to relax smooth muscle of the uterus, esp for PTL and uterine hyperstimulation Naloxone is an opiate antagonist admin to reverse resp distress

10) Vitamin K is prescribed for a neonate. A nurse prepares to administer the medication in which muscle site? 1. Deltoid 2. Triceps. 3. Vastus lateralis 4. Biceps

3. Use a 5/8 inch needle at a 90 degree angle: Vitamin K is given in the middle third of the vastus lateralis muscle using a 25-gauge, 5/8-inch needle. It is injected into skin that has been cleansed with alcohol and allowed to dry for 1 minute. It is administered at a 90-degree angle. The site is massaged after removing needle to increase absorption of the medication.

At what gestational age is a conceptus considered viable (able to live outside the womb)? 1. 9 weeks 2. 14 weeks 3. 24 weeks 4. 30 weeks

3. 23 / 24 weeks the lungs are developed enough to sometimes maintain extrauterine life. the lungs are the most immature system during the gestational period. medical care for premature labor begins much earlier (aggressively at 21 weeks0

Which of the following terms is used to describe the thinning and shortening of the cervix that occurs just before and during labor? 1. Ballottement 2. Dilation 3. Effacement 4. Muliparous

3. Effacement effacement is cervical shortening and thinning while dilation is widening of the cervix both facilitate opening the cervix in prep for delivery. Ballottement is the ability of another individual to move the fetus by externally manipulating the maternal abdomen. A ballotable fetus hasn't yet engaged in the maternal pelvis. Multiparous refers to a woman who has had previous live births.

A client hospitalized for premature labor tells the nurse she's having occasional contractions. Which of the following nursing interventions would be the most appropriate? 1. Teach the client the possible complications of premature birth. 2. Tell the clients to walk to see if she can get rid of the contractions. 3. Encourage her to empty her bladder and drink plenty of fluids, IV fluids 4. Notify anesthesia for immediate epidural placement to relieve the pain associated with contractions.

3. Encourage her to empty her bladder and drink plenty of fluids, IV fluids An empty bladder and adequate hydration may help decrease or stop labor contractions. Walking may encourage contractions to become stronger. Teaching the potential complications is likely to increase the clients anxiety rather than relax her It would be inappropriate to call anesthesia

Which condition or treatment best ensures lung maturity in an infant? 1. Meconium in the amniotic fluid 2. Glucocorticoid treatment just before delivery 3. Lecithin to sphingomyelin ratio more than 2:1 4. Absence of phosphatidylglycerol in amniotic fluid

3. Lecithin and sphingomyelin are phospholipids that help compose surfactant in the lungs; lecithin peaks at 36 weeks and sphingomyelin concentrations remain stable

During the last 6 weeks of gestation, which of the following tests isnt used to determine FWB? 1. BPP 2. NST 3. Maternal blood count 4. FM count

3. Maternal Blood count - evaluates maternal, not fetal well being BPP - uses US to eval fetal body movements, breathing movements, muscle tone, reactive fetal cardiac rate, amniotic fluid volume NST - evals the FHR for accels during FM FM - counts are used during the last trimester to obtain a rough index of fetal health - the number of FM are counted at diff times throughout the day and then charted to detect any change in overall activity over a number of days

A mother of a term neonate asks what the thick, white, cheesy coating is on his skin. Which correctly describes this finding? 1. Lanugo 2. Milia 3. Nevus flammeus 4. Vernix

4

With regard to fetal positioning during labor, nurses should be away that: birth is imminent at

4+ or 5+

A woman's obstetric history is that she is pregnant for the fourth time and all her children are living. One was born 39 weeks, twins were born 34 weeks, and another child was born 35 weeks. What is her GTPAL?

4-1-2-0-4

Pregnant for the 3rd time, all of her children are living, 1 child born at 39 weeks, twins born at 34 weeks, another child born at 35 weeks of gestation

4-1-2-0-4

Which patient is most at risk for fibroadenoma of the breast?

A 16-year-old girl

A primigravida is being monitored in her prenatal clinical for preeclampsia. What finding would concern her nurse?

A dipstick value of 3+ in her urine

Which statement about family systems theory is inaccurate?

A family as a whole is EQUAL to the sum of the individual members

During labor a fetus with an average heart rate of 135 beat/min over a 10 min period would be considered to have

A normal baseline heart rate

Which statement about pregnancy is accurate

A normal pregnancy lasts about 10 lunar months

A woman with severe preeclampsia is receiving magnesium sulfate infusion. The nurse becomes concerned when the woman exhibits which of the following symptoms?

A respiratory rate of 10 breaths/minute. (NOT: sleepy, sedated but responsive, DTR +2, and absent ankle clonus)

Ladin's Sign

A soft spot anteriorly in the middle of the uterus near the junction of the body of the uterus and cervix

The nurse is developing a dietary teaching plan for a patient on a vegetarian diet. The nurse should provide the patient with which examples of protein containing foods? (Select all that apply.) A Dried beans B Seeds C Peanut butter D Bagel E Eggs

A, B, C, E All of the foods listed except a bagel provide protein. A bagel is an example of a whole grain food, not protein.

Which suggestions should the nurse include when teaching about appropriate weight gain in pregnancy? (Select all that apply.) A Underweight women should gain 12.5 to 18 kg. B Obese women should gain at least 7 to 11.5 kg. C Adolescents are encouraged to strive for weight gains at the upper end of the recommended scale. D In twin gestations, the weight gain recommended for a single fetus pregnancy should simply be doubled. E Normal weight women should gain 11.5 to 16 kg.

A, B, C, E Underweight women need to gain the most. Obese women need to gain weight during pregnancy to equal the weight of the products of conception. Adolescents are still growing; therefore, their bodies naturally compete for nutrients with the fetus. Women bearing twins need to gain more weight (usually 16 to 20 kg) but not necessarily twice as much. Normal weight women should gain 11.5 to 16kg.

Medications used to manage postpartum hemorrhage include (choose all that apply): A. Pitocin B. Methergine C. Terbutaline D. Hemabate E. Magnesium sulfate

A, B, D Correct: These medications are all used to manage PPH. Incorrect: These medications are tocolytics; relaxation of the uterus causes or worsens PPH. p. 981

Which of these statements indicate the effect of breastfeeding on the family or society at large. (Select all that apply.) A Breastfeeding requires fewer supplies and less cumbersome equipment. B Breastfeeding saves families money. C Breastfeeding costs employers in terms of time lost from work. D Breastfeeding benefits the environment. E Breastfeeding results in reduced annual health care costs.

A, B, D, E Breastfeeding is convenient because it does not require cleaning or transporting bottles and other equipment. Breastfeeding saves families money because the cost of formula far exceeds the cost of extra food for the lactating mother. Less time is lost from work by breastfeeding mothers, in part because infants are healthier. Breastfeeding uses a renewable resource; it does not need fossil fuels, advertising, shipping, or disposal. Breastfeeding results in reduced annual health care costs.

The nurse is caring for a woman who is at 24 weeks of gestation with suspected severe preeclampsia. Which signs and symptoms would the nurse expect to observe? (Select all that apply.) A Decreased urinary output and irritability B Transient headache and +1 proteinuria C Ankle clonus and epigastric pain D Platelet count of less than 100,000/mm3 and visual problems E Seizure activity and hypotension

A, C, D Decreased urinary output and irritability are signs of severe eclampsia. Ankle clonus and epigastric pain are signs of severe eclampsia. Platelet count of less than 100,000/mm3 and visual problems are signs of severe preeclampsia. A transient headache and +1 proteinuria are signs of preeclampsia and should be monitored. Seizure activity and hyperreflexia are signs of eclampsia.

The nurse should include which instructions when teaching a mother about the storage of breast milk? (Select all that apply.) A Wash hands before expressing breast milk. B Store milk in 8 to 12 oz containers. C Store refrigerated milk in the door of the refrigerator. D Place frozen milk in the microwave for only a few seconds to thaw. E Milk thawed in the refrigerator can be stored for 24 hours.

A, E Breast milk storage guidelines for home use for full-term infants are: • Before expressing or pumping breast milk, wash your hands. • Containers for storing milk should be washed in hot, soapy water and rinsed thoroughly; they can also be washed in a dishwasher. If the water supply may not be clean, boil containers after washing. Plastic bags designed specifically for breast milk storage can be used for short-term storage (• Write the date of expression on container before storing milk. A waterproof label is best. • Store milk in serving sizes of 2 to 4 ounces to prevent waste. • Storing breast milk in the refrigerator or freezer with other food items is acceptable. • When storing milk in a refrigerator or freezer, place containers in the middle or back of the freezer, not on the door. • When filling a storage container that will be frozen, fill only three quarters full, allowing space at the top of the container for expansion. • To thaw frozen breast milk, place container in the refrigerator for gradual thawing or under warm, running water for quicker thawing. Never boil or microwave. • Milk thawed in the refrigerator can be stored for 24 hours. • Thawed breast milk should never be refrozen. • Shake milk container before feeding baby and test the temperature of the milk on the inner aspect of your wrist. • Any unused milk left in the bottle after feeding is discarded.

A pregnant woman is the mother of 2 children. Her first pregnancy ended in a still birth at 32 weeks of gestation, her second pregnancy with the birth of her daughter at 36 weeks, and her third pregnancy with the birth of her son at 41 weeks. Using the 5 digit system to describe the woman's current obstetric history, the nurse would record: a. 4-1-2-0-2 b. 3-1-2-0-2 c. 4-2-1-0-1 d. 3-1-1-1-3

A.

The nurse should tell a primigravida that the definitive sign indicating that labor has begun would be: a. progressive uterine contractions b. lightening c. rupture of membranes d. passage of the mucous plug

A. regular, progressive uterine contractions that increase in intensity and frequency are the definitive sign of true labor

A women's amniotic membranes have just ruptured. The immediate action of the nurse would be to: a. assess the fetal heart rate pattern b. perform a vaginal examination c. inspect the characteristics of the fluid d. assess maternal temp

A. the first nursing action after the membranes are ruptured is th check the FHR> Compression of the cord could occur after rupture leading to fetal hypoxia as reflected in an alteration in FHR pattern, characteristically variable decelerations. The same initial action should follow artificial rupture of the membranes.

What would be a warning sign of ineffective adaptation to extrauterine life if noted when assessing a 24 hour old breastfed newborn before discharge? a. apical heart rate of 90 beat/min, slightly irregular, when awake and active b. acrocyanosis c.harlequin color sign d. weight loss representing 5% of the newborn's birth weight

A. the heart rate of a newborn should range from 120-140 BPM, especially when active. The rate should be regular with sharp, strong sounds.

The most prevalent clinical manifestation of abruptio placenta as opposed to placenta previa is

Abdominal pain

In caring for the woman with disseminated intravascular coagulation (DIC) what order should the nurse anticipate

Administration of blood

A community women's health nurse knows that which groups of people are considered vulnerable populations? (Select all that apply.) Caucasian Americans Adolescent girls Women with underlying health conditions Refugee women Incarcerated women

Adolescent girls. Women with underlying health conditions, Refugee women, Incarcerated women

A woman who is 32 weeks pregnant is informed by the nurse that a danger sign of pregnancy could be which of the following?

Alteration in the pattern of fetal movements

The nurse caring for a laboring women should understand that early decelerations are caused by

Altered fetal cerebral blood flow

At 35 weeks of pregnancy a women experiences preterm labor, tocolytics administer, placed on bed rest, experiences regular uterine contractions, her cervix is beginning to dilate and efface, what would be an important test for fetal well- being at this time:

Amniocentesis for L/S ratio to assess fetal lung maturity

Fetal Tachycardia

An FHR baseline greater than 160 bpm for at least a 10 minute period

Influenza:

Annually

A nurse is assessing a premature infant. What would initially alert the nurse that the infant is having respiratory distress? A. Flaring nostrils B. Sporadic crying C. Ineffective cough D. Decreased pulse rate

Answer: A Rationale: In attempt to increase intake of oxygen, the respiratory rate increases with flaring of nostrils as a cardinal sign. It is significant to note that when a neonate is in respiratory distress, the rate of respirations will increase. Sporadic crying, ineffective cough, and decreased pulse rate may be indicative of infant distress but are not classic signs of respiratory distress.

When performing a physical assessment of an unusually small newborn infant, the nurse can determine that the infant is small for gestation age (SGA) rather than premature by which of the following characteristics? A. vigorous cry B. increased lanugo C. weaking sucking reflex D. diminished ear recoil

Answer: A Rationale: In contrast to a premature infant, an SGA baby has a vigorous cry and appears alert. Increased lanugo, weak sucking reflex, and diminished ear recoil are all present in the premature newborn that is physiologically underdeveloped.

Which statement is accurate regarding the reason premature infants develop neonatal respiratory distress syndrome? A. the alveoli lack surfactant B. the lungs lack ability to absorb oxygen C. the lungs cannot remove CO2 from the blood D. immature lungs cannots exchange CO2 and O2 effecively.

Answer: A Rationale: Surfactant is a sticky lubricant on the surface of the alveoli that essentially maintains patency of the alveoli in newborns. Premature infants lack surfactant, which causes the alveoli to collapse and requires increasing effort to reexpand the alveoli. Immature lungs are able to exchange O2 and CO2 but without surfactant, the alveoli collapse and cannot reexpland without ventilatory assistance.

A new mother expresses concern over strabismus in her infant. What would the nurse explain to the mother regarding this condition? A. it is a normal finding in newborns B. this may be a permanent defect C. it will require corrective surgery D. it will result in impaired vision

Answer: A Rationale: Muscle control of the eyes in the newborn is undeveloped, resulting in temporary strabismus (pseudostrbismus), or a cross-eyed appearance. This is considered normal in the newborn and usually corrects by the 3rd to 4th month. Strabismus is not a permanent defect, does not require corrective surgery, and does not cause impaired vision.

In caring for a pregnant women with sickle cell crisis, the nurse understands that management of this patient includes all except

Antibiotics

Ginger

Antiinflammatory

The nurse is planning care for a patient with a different cultural background. What would be an appropriate goal? Strive to keep the patient's cultural background from influencing health needs. Encourage the continuation of cultural practices in the hospital setting. In a nonjudgmental way attempt to change the patient's cultural beliefs. As necessary adapt the patient's cultural practices to her health needs.

As necessary adapt the patient's cultural practices to her health needs. The cultural background is part of the individual. It would be very difficult to eliminate the influence of the patient's background. The cultural practices need to be evaluated within the context of the health care setting to determine if they are conflicting. It is not appropriate to attempt to change someone's cultural practices. Whenever possible, the nurse should facilitate the integration of cultural practices into health needs.

With regard to the structure and function of the placenta the nurse should be aware

As one of its early functions, the placenta acts as an endocrine gland

With regard to the structure and function of the placenta, which of the following statements is accurate?

As one of its early functions, the placenta acts as an endocrine gland

Informed consent:

Ask the patient to "teach back."

As a woman enters the 2nd stage of labor, her membranes spontaneously rupture. When this occurs you do what first?

Assess fetal heart rate

The nursing care for a woman with preterm labor on Terbutaline (Brethine) would include which of the following interventions to identify side effects of the drug?

Assess for dyspnea and crackles

Nursing response to lung assessment postpartum

Assess for other signs to respiratory distress.

Nursing response to costoverterbral angle tenderness postpartum

Assess for other symptoms of UTI: obtain CC urine sample; report to physician or certified nurse midwife

An infant boy was born a few minutes ago. The nurse is conducting the assessment and APGAR score. When is the APGAR assessment performed?

At least twice, 1 minute and 5 minutes after birth

What is an early cause of postpartum hemorrhage?

Atony

When assessing a women in labor, the nurse is aware that the relationship of the fetal body parts to one another is called fetal

Attitude

A married woman has made the decision to use a diaphragm as her primary method of birth control. The clinic nurse should provide which instructions regarding care of, insertion, and removal of the diaphragm? (Select all that apply.) Remove the diaphragm by catching the rim from below the dome. Avoid using mineral oil body products. On insertion, direct the diaphragm down toward the space below cervix. Wash diaphragm monthly with mild soap and water. A dusting of cornstarch is appropriate after drying the diaphragm.

Avoid using mineral oil body products. On insertion, direct the diaphragm down toward the space below cervix. Wash diaphragm monthly with mild soap and water. A dusting of cornstarch is appropriate after drying the diaphragm. The diaphragm should not be removed by trying to catch the rim from below the dome. Oil-based products can cause the breakdown of the rubber. The diaphragm should be inserted into the vagina, directing it inward and downward as far as it will go to the space behind and below the cervix. The diaphragm should be washed after each use with mild soap and water. Cornstarch may be used. Avoid use of scented talc, body powder, and baby powder because they can weaken the rubber.

What woman is at greatest risk for early postpartum hemorrhage? A. A primiparous woman (G 2 P 1 0 0 1) being prepared for an emergency cesarean birth for fetal distress B. A woman with severe preeclampsia on magnesium sulfate whose labor is being induced C. A multiparous woman (G 3 P 2 0 0 2) with an 8-hour labor D. A primigravida in spontaneous labor with preterm twins

B A. Incorrect: Although many causes and risk factors are associated with PPH, this scenario does not pose risk factors or causes of early PPH. B. Correct: Magnesium sulfate administration during labor poses a risk for PPH. Magnesium acts as a smooth muscle relaxant, thereby contributing to uterine relaxation and atony. C. Incorrect: Although many causes and risk factors are associated with PPH, this scenario does not pose risk factors or causes of early PPH. D. Incorrect: Although many causes and risk factors are associated with PPH, this scenario does not pose risk factors or causes of early PPH. p. 976

The perinatal nurse assisting with establishing lactation is aware that acute mastitis can be minimized by: A. Washing the nipples and breasts with mild soap and water once a day B. Using proper breastfeeding techniques C. Wearing a nipple shield for the first few days of breastfeeding D. Wearing a supportive bra 24 hours a day

B A. Incorrect: Washing the nipples and breasts daily is no longer indicated. In fact, this can cause tissue dryness and irritation, which can lead to tissue breakdown and infection. B. Correct: Almost all instances of acute mastitis can be avoided by proper breastfeeding technique to prevent cracked nipples. C. Incorrect: Wearing a nipple shield does not prevent mastitis. D. Incorrect: Wearing a supportive bra 24 hours a day may contribute to mastitis, especially if an underwire bra is worn, because it may put pressure on the upper, outer area of the breast, contributing to blocked ducts and mastitis. p. 989

A pregnant woman with a body mass index (BMI) of 22 asks the nurse how she should be gaining weight during pregnancy. The nurse's BEST response would be to tell the woman that her pattern of weight gain should be approximately: A a pound a week throughout pregnancy. B 2 to 5 lbs during the first trimester, then a pound each week until the end of pregnancy. C a pound a week during the first two trimesters, then 2 lbs per week during the third trimester. D a total of 25 to 35 lbs.

B A pound a week is not the correct guideline during pregnancy. A BMI of 22 represents a normal weight. Therefore, a total weight gain for pregnancy would be about 25 to 35 lbs or about 2 to 5 lbs in the first trimester and about 1 lb/wk during the second and third trimesters. These are not accurate guidelines for weight gain during pregnancy. The total is correct, but the pattern needs to be explained.

Women with an inadequate weight gain during pregnancy are at higher risk of giving birth to an infant with: A spina bifida. B intrauterine growth restriction. C diabetes mellitus. D Down syndrome.

B Spina bifida is not associated with inadequate maternal weight gain. An adequate amount of folic acid has been shown to reduce the incidence of this condition. Both normal-weight and underweight women with inadequate weight gain have an increased risk of giving birth to an infant with intrauterine growth restriction. Diabetes mellitus is not related to inadequate weight gain. A gestational diabetic mother is more likely to give birth to a large-for-gestational age infant. Down syndrome is the result of a trisomy 21, not inadequate maternal weight gain.

The emergency department nurse is assessing a pregnant trauma victim who just arrived at the hospital. What are the nurse's MOST appropriate actions? (Select all that apply.) A Place the patient in a supine position. B Assess for point of maximal impulse at fourth intercostal space. C Collect urine for urinalysis and culture. D Frequent vital sign monitoring. E Assist with ambulation to decrease risk of thrombosis.

B, C, D Passive regurgitation may occur if patient is supine, leading to high risk for aspiration. Placental perfusion is decreased when the patient is in a supine position as well. The heart is displaced upward and to the left in pregnant patients. During pregnancy, there is dilation of the ureters and urethra, and the bladder is displaced forward placing the pregnant trauma patient at higher risk for urinary stasis, infection, and bladder trauma. The trauma patient can suffer blood loss and other complications, necessitating frequent monitoring of vital signs. While the pregnant patient is at risk for thrombus formation, the patient must be cleared by the health care provider before ambulating. The pregnant trauma patient is at higher risk for pelvic fracture, and therefore this condition must be ruled out first as well.

A pregnant woman presents to the emergency department complaining of persistent nausea and vomiting. She is diagnosed with hyperemesis gravidarum. The nurse should include which information when teaching about diet for hyperemesis? (Select all that apply.) A Eat three larger meals a day. B Eat a high-protein snack at bedtime. C Ice cream may stay down better than other foods. D Avoid ginger tea or sweet drinks. E Eat what sounds good to you even if your meals are not well-balanced.

B, C, E The diet for hyperemesis includes: • Avoid an empty stomach. Eat frequently, at least every 2 to 3 hours. Separate liquids from solids and alternate every 2 to 3 hours. • Eat a high-protein snack at bedtime. • Eat dry, bland, low-fat, and high-protein foods. Cold foods may be better tolerated than those served at a warm temperature. • In general eat what sounds good to you rather than trying to balance your meals. • Follow the salty and sweet approach; even so-called junk foods are okay. • Eat protein after sweets. • Dairy products may stay down more easily than other foods. • If you vomit even when your stomach is empty, try sucking on a Popsicle. • Try ginger tea. Peel and finely dice a knuckle-sized piece of ginger and place it in a mug of boiling water. Steep for 5 to 8 minutes and add brown sugar to taste. • Try warm ginger ale (with sugar, not artificial sweetener) or water with a slice of lemon. • Drink liquids from a cup with a lid.

When caring for a newborn, the nurse must be alert for signs of cold stress, including: a. decreased activity level b. increased respiratory rate c. hyperglycemia d. shivering

B. An increased respiratory rate is a sing of cold stress in the newborn

When providing care to a young single woman just diagnosed with acute pelvic inflammatory diseaase, the nurse should: a. point out that inappropriate sexual behavior caused the infection b. position the woman in a semi fowler position c. explain to the woman that infertility is a likely outcome of this type of infection d. tell her that antibiotics need to be taken until pelvic pain is relieved.

B. The position of comfort is the semi fowler position. In addition, the foot of the bed could be elevated to keep the uterus in a dependent position and reduce discomfort.

A laboring woman becomes anxious during the transition phase of the first stage of labor and develops a rapid and deep respiratory pattern. She complains of feeling dizzy and light-headed. The nurse's immediate response would be to: a. encourage the woman to breathe more slowly. b. help the woman breathe into a paper bag. c. turn the woman on her side. d. administer a sedative.

B. The woman is exhibiting signs of hyperventilation. This leads to decreased carbon dioxide level and respiratory alkalosis. Rebreathing her air would increase the CO2 level.

Which contraceptive method best protects against sexually transmitted infections (STIs) and human immunodeficiency virus (HIV)?

Barrier methods

What is kernicterus?

Bilirubin deposits are in the brain

Which minerals and vitamins usually are recommended to supplement a pregnant woman's diet? A Fat-soluble vitamins A and D B Water-soluble vitamins C and B6 C Iron and folate D Calcium and zinc

C Fat-soluble vitamins should be supplemented as a medical prescription, as vitamin D might be for lactose-intolerant women. Water-soluble vitamin C sometimes is consumed in excess naturally; vitamin B6 is prescribed only if the woman has a very poor diet. Iron generally should be supplemented, and folic acid supplements often are needed because folate is so important. Zinc sometimes is supplemented. Most women get enough calcium.

A mother expresses fear about changing her infant's diaper after he is circumcised. What does the woman need to be taught to take care of the infant when she gets home? A Cleanse the penis with prepackaged diaper wipes every 3 to 4 hours. B Apply constant, firm pressure by squeezing the penis with the fingers for at least 5 minutes if bleeding occurs. C Cleanse the penis gently with water and put petroleum jelly around the glans after each diaper change. D Wash off the yellow exudate that forms on the glans at least once every day to prevent infection.

C With each diaper change, the penis should be washed off with warm water to remove any urine or feces. If bleeding occurs, the nurse should apply gentle pressure to the site of the bleeding with a sterile gauze square. This action is appropriate when caring for an infant who has had a circumcision. Yellow exudate covers the glans penis in 24 hours after the circumcision. This is part of normal healing and not an infective process. The exudate should not be removed.

The student nurse is giving a presentation about milestones in embryonic development. Which information should he or she include? a. at 8 weeks of gestation primary lung and uretheral buds appear. b. at 12 weeks of gestation the vagina is open or the testes are in position for descent into the scrotum. c. at 20 weeks of age the vernix caseosa and lanugo appear d. at 24 weeks of age the skin is smooth, and subcutaneous fat is beginning to collect.

C. A. happens at 6 weeks B. happens at 16 weeks smooth skin occurs at 28 weeks, subq fat begins to collect at 20 to 31 weeks.

Self care instructions for a woman following a modified radical mastectomy would include that she: a. wear clothing with snug sleeves to support her affected arm. b. Use depilatory creams instead of shaving the axilla of her affected arm. c. Expect a decrease in sensation or tingling in her affected arm as her body heals. d. Empty surgical drains once a day or every other day.

C. A decrease in sensation and tingling in the affected arm and in the incision are expected for weeks to months after surgery.

When palpating the fundus of a woman 18 hours after birth, the nurse notes that it is firm, 2 fingerbreadths above the umbilicus, and deviated to the left ot midline. The nurse should: a. Massage the fundus b. administer methergine, 0.2 mg PO, that has been ordered prn. c. Assist the woman to empty her bladder d. recognize this as an expected finding during the first 24 hours following birth.

C. The findings indicate a full bladder, which pushes the uterus up and to the right or left of midline. The recommended action would be to empty the bladder. If the bladder remains distended, uterine atony could occur, resulting in a profuse flow.

Which measure would be least effective in preventing postpartum hemorrhage? a. admin methergine, 0.2 mg every 6 hours for four doses, as ordered b. encourage the woman to void every 2 hours c. massage the fundus every hour for the first 24 hours following birth d. teach the woman the importance of rest and nutrition to enhance healing

C. The fundus should be massaged only when boggy or soft. Massaging a firm fundus could cause it to relax.

Maternal and neonatal risks associated with gestational diabetes mellitus are: a. maternal premature rupture of membranes and neonatal sepsis b. maternal hyperemesis and neonatal low birth weight. c. maternal preeclampsia and fetal macrosomia d. Maternal placenta previa and feta prematurity.a

C. Women with gestational diabetes have twice the risk of developing hypertensive disorders such as preeclampsia, and the baby usually has macrosomia.

What is the antidote to reverse the effects of magnesium sulfate?

Calcium gluconate

Nafarelin is currently used as a treatment for mild-to-severe endometriosis. The nurse should tell a woman taking this medication that the drug:

Can cause her to experience some hot flashes and vaginal dryness.

Lochia alba

Can last from day 11 all the way up to 6 weeks -Yellow to clear discharge -Should not have color

Nurses should be aware chronic hypertension

Can occur simultaneously with gestational hypertension

The drug of choice for treatment of gonorrhea is:

Ceftriaxone.

Which of the following supports the diagnosis of pathologic jaundice?

Clinical jaundice evident within 24 hours of birth

Braxton Hicks Contractions

Contractions that are irregular and painless that occur at irregular intervals throughout pregnancy but are felt with abdominal palpitation after week 28

Perinatal nurses are legally responsible for which of the following?

Correctly interpreting FHR patterns, initiating appropriate nursing interventions and documenting the outcomes

A woman arrives for evaluation of her symptoms which include a missed period, adnexal fullness, tenderness and dark red vaginal bleeding. Upon examination, the nurse notices ecchymotic blueness around the umbilicus. The nurse recognizes this assessment finding as:

Cullen's sign associated with a ruptured ectopic pregnancy

The uterus is a muscular, pear-shaped organ that is responsible for:

Cyclic menstruation.

Which finding meets the criteria of a reassuring fetal heart rate pattern? a. FHR does not change as a result of fetal activity. b. average baseline rate ranges between 100 and 140 BPM. c. Mild late decelration patterns occur with some contractions. d. Varibility averages between 6-10 BPM

D,. Variability indicates a well oxygenated fetus with a functioning autonomic nervous system

The nurse is planning care for a patient with a different cultural background. What would be an appropriate goal? a. strive to keep the patient's cultural background from influencing health needs b. encourage the continuation of cultural practices in the hospital setting c. in a nonjudgmental way attempt to change the patient's cultural beliefs. As necessary adapt the patient's cultural practices to her health needs

D. Whenever possible, the nurse should facilitate the integration of cultural practices into health needs

A primigravida asks the nurse about signs she can look for that would indicate that the onset of labor is getting closer. The nurse should describe: a. Weight gain of 1 to 3 pounds. b. Quickening. c. Fatigue and lethargy. d. bloody show

D. passage of the mucous plug, also termed pink/blood show occurs as the cervix ripens.

Practice of the calendar rhythm method is based on the number of days in each menstrual cycle. The fertile period is determined after monitoring each cycle for 6 months. The beginning of the fertile period is estimated by subtracting 18 days from the longest cycle and 11 days from the shortest. If the woman's cycles vary in length from 24 to 30 days, what would her fertile period be? ________ to _________

Day 6 to day 19

During a prenatal visit, the nurse is explaining dietary management to a woman with pregestational diabetes. The nurse evaluates that teaching has been effective when the woman says:

Diet and insulin needs change during pregnancy

Team member for: newly diagnosed gestational diabetic

Dietician

When assessing a women in first stage of labor the nurse recognizes that the most conclusive sign that uterine contraction are effective would be

Dilation of cervix

Estimated Date of Confinement (EDC)

During pregnancy the approximate date when childbirth will occur, the "due date"

____________________ is the most common postpartum infection.

Endometritis p. 987

Black cohosh

Estrogen-like luteinizing hormone suppressant

A 23-year-old African-American woman is pregnant with her first child. Based on the statistics for infant mortality, which plan is most important for the nurse to implement?

Explain to the woman the importance of keeping her prenatal care appointments.

A mother's household consists of her husband, his mother, and another child. She is living in a(n):

Extended family.

Excessive blood loss after child birth can have several causes, the most common is:

Failure of the uterine muscle to contract firmly-atony

The exact cause of breast cancer remains undetermined. Researchers have found that there are many common risk factors that increase a woman's chance of developing a malignancy. It is essential for the nurse who provides care to women of any age to be aware of which of the following risk factors (Select all that apply)?

Family history Race Nulliparity or first pregnancy after age 30

A woman is giving birth to her third child in a setting that allows her husband and other two children to be actively involved in the process. The nurse caring for the woman must also consider the husband and family as patients and work to meet their needs. This type of setting is termed: family-centered care. emergency care. hospice care. individual care.

Family-centered care Family-centered care is any setting in which the pregnant woman and family are treated as one unit. The nurse assumes a major role in teaching, counseling, and supporting the family. In emergency care settings, the nurse deals primarily with the patient who is having difficulty. In hospice care settings, the nurse deals with patients who have terminal illnesses. Individual care deals only with the patient and does not include the family.

In the care of preterm infants, why are nasogastric and orogastric tubes used?

Feed the infant

Examples of sexual risk behaviors associated with exposure to a sexually transmitted infection (STI) include (Select all that apply):

Fellatio. Unprotected anal intercourse. Multiple sex partners.

The nurse caring for a laboring women should know that meconium is produced by

Fetal intestines

Although remarkable developments have occurred in reproductive medicine, assisted reproductive therapies are associated with numerous legal and ethical issues. Nurses can provide accurate information about the risks and benefits of treatment alternatives so couples can make informed decisions about their choice of treatment. Which issue would not need to be addressed by an infertile couple before treatment?

Financial ability to cover the cost of treatment

Which diagnostic test is used to confirm a suspected diagnosis of breast cancer?

Fine-needle aspiration (FNA)

To adequately care for a laboring woman, the nurse should know which state of labor varies the most in length

First stage

Which of the following descriptions of the four stages of labor is correct for both definition and duration?

First stage: onset of regular uterine contractions to full dilation; less than 1 hour to 20 hours

Exposure to teratogens is most likely to adversely effect the fetus during which time of development?

First trimester

A couple comes in for an infertility workup, having attempted to get pregnant for 2 years. The woman, 37, has always had irregular menstrual cycles but is otherwise healthy. The man has fathered two children from a previous marriage and had a vasectomy reversal 2 years ago. The man has had two normal semen analyses, but the sperm seem to be clumped together. What additional test is needed?

Follicle-stimulating hormone (FSH) level

A nurse is performing an assessment on four 22-week pregnant clients. The nurse reports to the OB that which of the clients may be carrying twins:

Fundal height measurement 26

Normal Fetal attitude

General flexion - the head is flexed so that the chin is on the chest with the arms crossed over the chest, and the legs are flexed at the knees with the thighs on the abdomen.

Team member for: Newly diagnosed pregnant woman who has a family history of Down's syndrome

Genetic counselor

A pictorial tool that can assist the nurse in assessing aspects of family life related to health care is the:

Genogram.

The nurse must be cognizant that an individual's genetic makeup is known as

Genotype

Which woman is most likely to experience strong after pains

Gravida 4, para 4-0-0-4

The labor of a preeclamptic woman is going to be induced. Before initiating the oxytocin infusion, the nurse reviews the woman's labs and finds a platelet count of 90,000, an elevated AST level and a falling hematocrit. The nurse should tell the Dr because it indicates?

HELLP

To detect human immunodeficiency virus (HIV), most laboratory tests focus on the:

HIV antibodies.

For which of the following reasons would breastfeeding be contraindicated?

HIV positive

When a woman is diagnosed with PPD with psychotic features, one of the major concerns is that she may

Harm her infant

When a woman is diagnosed with postpartum depression with psychotic features, one of the main concerns is that she may do which of the following?

Harm her infant or herself

Informed consent concerning contraceptive use is important because some of the methods:

Have potentially dangerous side effects

Which condition not uncommon in pregnancy is likely to require careful medical assessment during puerperium

Headaches

Which of the following statements about the various forms of hepatitis is accurate?

Hepatitis A is acquired by eating contaminated food or drinking polluted water.

You are to administer Methylergonovinc (Methergine) 0.2 mg to a postpartal woman with uterine subinvolution. Which of the following assessments would prompt you to hold the med?

Her blood pressure is 150/100

A woman who is older than 35 years may have difficulty achieving pregnancy primarily because:

Her ovaries may be affected by the aging process

What should a laboring woman be told if an opioid antagonist is administered?

Her pain will return

Which of the following is an opiate the causes euphoria, relaxation, drowsiness, and detachment from reality and has possible effects on the pregnancy, including preeclampsia, IUGR, and premature rupture of membranes. Methadone may be given as treatment

Heroin

Which viral sexually transmitted infection is characterized by a primary infection followed by recurrent episodes?

Herpes simplex virus

Which viral sexually transmitted infection is characterized by a primary infection followed by recurrent episodes?

Herpes simplex virus (HSV)-2

During which phase of the cycle of violence does the batterer become contrite and remorseful?

Honeymoon phase

A woman is 3 months pregnant. At her prenatal visit, she tells the nurse that she doesn't know what is happening: one minute she is happy that she is pregnant and the next minute she cries. Which of the following responses by the nurse is most appropriate?

Hormone changes during pregnancy commonly result in mood swings

The viral sexually transmitted infection (STI) that affects most people in the United States today is:

Human papillomavirus (HPV).

A condition in which the ventricles of the brain are enlarged as a result of imbalanced CSF. Bulging fontanels, increased head circumference, and increased ICP. What is the condition?

Hydrocephalus

The stimulated release of gonadotropin-releasing hormone and follicle-stimulating hormone is part of the:

Hypothalamic-pituitary cycle.

A baby was just born to a mother who had positive vaginal cultures for group B strep. The mother was admitted to labor 2 hours before the birth. Which of the following should the nurse closely observe for in this baby

Hypothermia

With regard to the use of intrauterine devices (IUDs), nurses should be aware that:

IUDs containing copper can provide an emergency contraception option if inserted within a few days of unprotected intercourse.

What would the nurse caring for a woman with hyperemesis gravidarum expect the initial treatment to involve?

IV therapy to correct fluid and electrolyte imbalances

What is the primary role of practicing nurses in the research process?

Identifying researchable problems

From the nurse's perspective, what measure should be the focus of the health care system to reduce the rate of infant mortality further?

Implementing programs to ensure women's early participation in ongoing prenatal care

A woman is being admitted for severe preeclampsia. When deciding on where to place her, which of the following areas would be most appropriate?

In a quiet, private room away from the hustle and bustle of the unit

According to standard professional thinking, nurses should auscultate the fetal heart rate (FHR):

In the active phase of the first stage of labor, the FHR should be auscultated every 30 min if no risk factors are involved

When managing health care for pregnant women at a prenatal clinic, the nurse should recognize that the most significant barrier to access to care is the pregnant woman's:

Inability to pay.

A gravid woman with sickle cell anemia is admitted in vaso-occlusive crisis. Which of the following is the priority intervention that the nurse must perform

Infuse IV hydration

While evaluating an external monitor of a woman in active labor that is being induced, the nurse notes FHR begins to decelerate at the onset of several contractions and returns to baseline. What should the nurse do?

Initiate oxygen therapy

The nurse assessing a newborn knows that is most critical physiologic change required immediately after birth is which of the following?

Initiation and maintenance of respirations

Because a full bladder prevents the uterus from contracting normally, nurses intervene to help empty it spontaneously ASAP. If all else fails, which of the following is the last thing the nurse might try?

Inserting a sterile catheter

4 babies are in the newborn nursery. The nurse pages the neonatologist to see that baby who exhibits what? o Intercostal retractions o Erythema toxicum o Pseudostrabismus o Vernix caseousa

Intercostal retractions

A woman has a breast mass that is not well delineated and is nonpalpable, immobile, and nontender. This is most likely:

Intraductal papilloma.

• When planning care for a laboring woman whose membranes have ruptured, the nurse recognizes that the woman's risk for ___ has increased

Intrauterine Infection

The most dangerous effect on the fetus of a mother who smokes cigarettes while pregnant is:

Intrauterine growth restriction

Women with an inadequate weight gain during pregnancy are at higher risk of giving birth to an infant with which of the following conditions?

Intrauterine growth restriction

When planning care for a laboring woman whose membranes have ruptured, the nurse needs to recognize that the woman is at increased risk for which of the following?

Intrauterine infection

Nurses should be aware that infertility:

Is perceived differently by women and men.

Nurses should be aware that induction of labor

Is rated for viability by a bishop score

While instructing a couple regarding birth control, the nurse should be aware that the method called natural family planning:

Is the only contraceptive practice acceptable to the Roman Catholic church.

Which of the following statements regarding postpartum depression without psychotic features is accurate?

It is distinguished by irritability, severe anxiety and lack of interest in appearance.

As relates to dysfunctional uterine bleeding (DUB), the nurse should be aware that:

It is most commonly caused by anovulation.

With regard to endometriosis, nurses should be aware that:

It may worsen with repeated cycles or remain asymptomatic and disappear after menopause.

A maternity nurse should be aware of which fact regarding the amniotic fluid?

It serves as a source of oral fluid and as a repository for waste from the fetus

Way to Increase Uterine Tone

Keagel exercises

A 62-year-old woman has not been to the clinic for an annual examination for 5 years. The recent death of her husband reminded her that she should come for a visit. Her family doctor has retired, and she is going to see the women's health nurse practitioner for her visit. To facilitate a positive health care experience, the nurse should:

Listen carefully and allow extra time for this woman's health history interview.

Which of the following positions would be least effective when gravity is desired to assist in fetal descent?

Lithotomy

A woman gave birth to a healthy infant 5 days ago. What type of lochia would the nurse expect?

Lochia serosa

Screening at 24 weeks of gestation reveals that a normal weight pregnant woman has gestational diabetes mellitus (GDM) in planning her care nurse and woman mutually agree that an expected outcome is to prevent injury to the fetus as a result of GDM the nurse identifies that the fetus is at greatest risk for

Macrosomia

The following are nursing care measures commonly offered to woman in labor. Which demonstrates gate-control theory?

Massage the woman's back

The nurse is performing a gestational age and physical assessment. The infant appears to have an excessive amount of saliva. The nurse recognizes this finding as what?

May indicate that the infant has tracheoesophageal fistula or esophageal atresia

A collection of blood between the skull bone and its periosteum is known as cephalohematoma. To reassure the new parents infants develops such a soft bulge it is important that the nurse be aware

May occur with spontaneous vaginal birth

A first time father is changing the diaper of his 1 day old daughter. He asks the nurse, "What is this black sticky stuff in her diaper?

Meconium

Health care functions carried out by families to meet their members' needs include:

Meeting nutritional requirements.

A 36-year-old woman has been given a diagnosis of uterine fibroids. When planning care for this patient, the nurse should know that:

Menorrhagia is a common finding.

Prostaglandins are produced in most organs of the body, including the uterus. Other source(s) of prostaglandins is/are:

Menstrual blood.

One of the alterations in cyclic bleeding that occurs between periods is called:

Metrorrhagia.

Physiologically, sexual response can be characterized by:

Myotonia and vasocongestion.

A patient has been prescribed adjuvant tamoxifen therapy. What common side effect might she experience?

Nausea, hot flashes, and vaginal bleeding

The nurse must evaluate a male patient's knowledge regarding the use of a condom. The nurse would recognize the need for further instruction if the patient states that he: lubricates the condom with a spermicide containing nonoxynol-9. leaves an empty space at the tip of the condom. leaves a small amount of air in the tip. removes his still-erect penis from the vagina while holding onto the base of the condom.

Nonoxynol-9 is no longer recommended. Recent data suggest that frequent use of nonoxynol-9 may increase human immunodeficiency virus transmission and can cause genital lesions. An empty space at the tip of the condom is the correct instruction. Leaving a small amount of air at the tip of the condom is the correct instruction. Removing the condom while holding the base is the correct instruction.

Management of primary dysmenorrhea often requires a multifaceted approach. The nurse who provides care for a client with this condition should be aware that the optimal pharmacologic therapy for pain relief is:

Nonsteroidal antiinflammatory drugs (NSAIDs).

The woman's family members are present when the home care maternal-child nurse arrives for a postpartum and newborn visit. What should the nurse do?

Observe the family members' interactions with the newborn and one another.

A woman has chosen the calendar method of conception control. During the assessment process, it is most important that the nurse:

Obtain a history of menstrual cycle lengths for the past 6 to 12 months.

Tetanus-diphtheria-pertussis (Tdap):

Once and then a booster every 10 years

Measles, mumps, rubella:

Once if born after 1956

A woman gave birth to an infant boy 10 hours ago, Where would the nurse expect to locate this woman's fundus

One centimeter above the umbilicus

Herpes Zoster:

One dose after age 65

Thyroid dysfunction or obesity

Other factors

Anorexia

Ovarian

The two primary functions of the ovary are:

Ovulation and hormone production.

With regard to dysmenorrhea, nurses should be aware that:

Pain usually occurs in the suprapubic area or lower abdomen.

Which finding should nurse expect when assessing a client with placenta previa

Painless, vaginal bleeding

While giving care to a client who has had premature rupture of membranes, the nurse should refrain from which nursing action?

Performing frequent vaginal examinations

The nurse is assessing a woman's breast self-examination (BSE) technique. Which action indicates that a woman needs further instruction regarding BSE? Performs every month on the first day of her menstrual period Uses the pads of her fingers when palpating each breast Inspects her breasts while standing before a mirror and changing arm positions Places a folded towel under right shoulder and right hand under head when palpating right breast

Performs every month on the first day of her menstrual period BSE should be performed once a month after the menstrual period has ended. These are correct actions for performing a BSE.

Which marks on a baby's skin may indicate an underlying problem that required notification of the physician?

Petechiae scattered over the infant's body

All of the following are considered normal gastrointestinal changes in pregnancy except which one? This condition can also be associated with anemia

Pica

Placenta retention

Placenta or membranes are left behind in the uterus during the third stage of labor.

A woman has just been admitted to the ER subsequent to head-on automobile accident. Her body appears to be uninjured. The nurse monitor for which complications of pregnancy

Placental Abruption

What would a breastfeeding mother who is concerned that her baby is not getting enough to eat find most helpful and most cost-effective on the day after discharge? Visiting a pediatric screening clinic at the hospital Placing a call to the hospital nursery "warm line" Calling the pediatrician for a lactation consult referral Requesting a home visit

Placing a call to the hospital nursery "warm line" This action would not necessarily be cost-effective. The first course of action should be to call a warm line for advice from a nurse. Warm lines are telephone lines offered as a community service to provide new parents with support, encouragement, and basic parenting education. This action would not necessarily be cost-effective. The first course of action should be to call a warm line for advice from a nurse. This action would not necessarily be cost-effective. The first course of action should be to call a warm line for advice from a nurse.

A pregnant woman's amniotic membranes have ruptured prolapsed cord is suspected. What intervention would be the top priority

Placing the woman in knee to chest position

Hepatitis B:

Primary series of three injections

Which of the following hormones is essential for maintaining pregnancy by relaxing smooth muscle?

Progesterone

The hormone responsible for maturation of mammary gland tissue is:

Progesterone.

Culture measurement, feedback, and intervention:

Promote interventions that will reduce patient risk.

Certain fatty acids classified as hormones that are found in many body tissues and that have roles in many reproductive functions are known as:

Prostaglandins (PGs).

What is a major nursing intervention for an infant born with myelomeningocele?

Protect the sac from injury

When attempting to communicate with a patient who speaks a different language, the nurse should:

Provide as much privacy as possible.

Which interventions would help alleviate the problems associated with access to health care for maternity patients (Select all that apply)?

Provide transportation to prenatal visits. Provide childcare so that a pregnant woman may keep prenatal visits. Provide low-cost or no-cost health care insurance.

Chadwick's Sign

Purple discoloration of the cervix

Alternative and complementary therapies:

Recognize the value of clients' input into their health care.

Pediatric imaging:

Reduce exposure to radiation.

After change of shift report, the nurse assumes care of a multiparous patient in labor. The woman is complaining of pain that radiates to her abdominal wall, lower back, and buttocks and down her thighs. Before implementing a plan of care the nurse should understand that this type of pain is

Referred

Most important adaption in newborns

Respiratory (Breathing)

Despite warnings, prenatal exposure to alcohol continues to exceed by far exposure to illicit drugs. A diagnosis of fetal alcohol syndrome (FAS) is made when there are visible markers in each of three categories. Which is category is not associated with a diagnosis of FAS?

Respiratory conditions

Infants of mothers with diabetes are at higher risk for developing which of the following conditions?

Respiratory distress syndrome

Habituation

Response to light and loud noises will soon diminish. They become ok with it and forget that it is there

The long-term treatment plan for an adolescent with an eating disorder focuses on:

Restructuring perception of body image.

Fundal height at 12 weeks

Right above the symphysis pubis

In what form do families tend to be most socially vulnerable?

Single-parent family

A woman is at 14 weeks gestation, the nurse would expect to palpate the fundus at which level

Slightly above pubic symphisis

Team member for: a patient who tests positive for cocaine

Social worker

Which contraceptive method has a failure rate of less than 25%?

Standard days

Non stress test

Start at 28 weeks if evidence of preeclampsia, IUGR, oligohyramnios or poorly controlled blood glucose levels

A woman in preterm labor at 30 weeks receives two doses of Betamethasone IM. What is the purpose of this treatment?

Stimulate fetal surfactant production

After an emergency birth (birth may be unexpected place, not in typical facility with things you would normally have to deal with the birth) , the nurse encourages the woman to breastfeed her newborn. The primary purpose of this activity is to:

Stimulate the uterus to contract

An essential component of counseling women regarding safe sex practices includes discussion regarding avoiding the exchange of body fluids. The physical barrier promoted for the prevention of sexually transmitted infections and human immunodeficiency virus is the condom. Nurses can help motivate clients to use condoms by initiating a discussion related to a number of aspects of condom use. The most important of these is:

Strategies to enhance condom use.

How to calculate Nigele's Rule

Subtract 3 months from LMP and add 7 days

A premature infant with respiratory distress syndrome receives artificial surfactant. How would the nurse explain the surfactant therapy to the parents?

Surfactant improves the ability of your baby's lungs to exchange oxygen and carbon dioxide

The National Quality Forum has issued a list of "never events" or "serious reportable events" pertaining specifically to maternal and child health. Which of the following is not included on this list?

Surgery performed on the wrong body part

The cervix of a 26 year old primigravida in labor is 5cm dilated and 75% effaced, and the fetus is at 0 station. The doctor prescribes an epidural regional block. Into which of the following positions should the nurse place the client when the epidural is admin? 1. Lithotomy 2. Supine 3. Prone 4. Lateral

THe client should be placed on her left side or sitting up right, with her shoulders parallel and legs slightly flexed. Her back shouldnt be flexed bc this position increases increases the possibility that the dura may be punctured and the anesthetic will accidentally be given as spinal, not epidural, anesthesia.

A 24 week pregnant client is being seeing the prenatal client. She states, "I've had a terrible headache for the past few days, Which is the priority assessment

Take her BP

A woman will be taking oral contraceptives using a 28-day pack. The nurse should advise this woman to protect against pregnancy by:

Taking one pill at the same time every day.

What is one of the first symptoms of puerperal infection to assess for in postpartum woman?

Temperature of 38 C (100 F) or higher on 2 successive days starting 24 hours after birth

The various systems and organs develop at different times. Which of the following statements in accurate?

The cardiovascular system is the first organ system to function in the developing human

The level of practice a reasonably prudent nurse provides is called:

The standard of care.

A 38-year-old Hispanic woman delivered a 9-pound, 6-ounce girl vaginally after being in labor for 43 hours. The baby died 3 days later from sepsis. On what grounds would the woman potentially have a legitimate legal case for negligence?

The standards of care were not met.

Effacement

The taking up (or drawing up) of the internal os and the cervical canal into the uterine side walls.

Ferguson reflex

The urge to push

With regard to afterbirth pains, nurses should be aware that which of the following statements is corret?

These pains are more noticeable in births in which the uterus was over distended

Which of the following statements is the most complete and accurate description of medical abortions?

They can be either elective or therapeutic.

Human papillomavirus (HPV):

Three injections for girls between the ages 9 to 26

Nurse teaches pregnancy women about characteristics of true labor contractions, the nurse evaluated the woman understands

True labor contractions will continue and get stronger even if I relax and even if I relax and take a shower

Endometriosis

Tubal/peritoneal

What statement about multifetal pregnancy is not accurate

Twin pregnancies come to term with the same frequency as single pregnancies

A 39 year old primagravida thinks she is about 8 weeks pregnant although she has had irregular MP her entire life, history of smoking one pack of cigarettes a day, she tells you that she is trying to cut down. Her lab data are within normal limits what diagnostic (should be screening) technique could be used with this pregnant woman at this time

Ultrasound

A 39 year old primagravida thinks she is about 8 weeks pregnant although she has had irregular menstrual periods her life history of smoking approx. one pack of cigarette a day tell you she trying to cut down, lab within normal limits

Ultrasound

26 year old women G2 P1 is 28 weeks pregnancy when she experiences bright red, painless vaginal bleeding. She has not previously had any prenatal care for this pregnancy. Upon arrival at the hospital what would be an expected diagnostic procedure:

Ultrasounds for placental location

What causes variable FHR decelerations?

Umbilical cord compression

Which of the following infant responses to cool environmental conditions is not effective to increase temp?

Unflexing from the normal position

When a nurse is counseling a woman for primary dysmenorrhea, which nonpharmacologic intervention might be recommended?

Using a heating pad on the abdomen to relieve cramping

The perinatal nurse assisting with establishing lactation is aware that acute mastitis can be minimized by

Using proper breastfeeding techniques

The perinatal nurse assisting with establishing lactation is aware that mastitis can be minimized by which of the following?

Using proper breastfeeding techniques

Endometrial or myometrial tumors

Uterine

In 1st trimesters ultrasonography can be used to gain information on

Uterine Abnormalities

The perinatal nurse is caring for a woman in the immediate post birth period. Assessment reveals that the woman is experiencing profuse bleeding. The most likely etiology for the bleeding is

Uterine Atony

Valerian, wild yam

Uterine antispasmodic

What causes late FHR decelerations

Uteroplacental insufficiency

The female reproductive organ(s) responsible for cyclic menstruation is/are the:

Uterus.

How do you assess cervical change

Vaginal Exam

A woman is 36 weeks gestation, which of the following tests should be done during her next prenatal visit

Vaginal and rectal cultures

Isoimmunization

Vaginal/cervical

A common effect of both smoking and cocaine use in the pregnant woman is:

Vasoconstriction

Through the use of social media technology, nurses can link with other nurses who may share similar interests, insights about practice, and advocate for patients. The most concerning pitfall for nurses using this technology is:

Violation of patient privacy and confidentiality.

After a mastectomy a woman should be instructed to perform all of the following except:

Wearing clothing with snug sleeves to support the tissue of the arm on the operative side.

In helping the breastfeeding mother position the baby, nurses should keep in mind that:

Whatever the position used, the infant is "belly to belly" with the mother/Don't want to have the baby's head turned, this can cause latching problems

What is descriptive of the family systems theory? The family is viewed as the sum of individual members. When the family system is disrupted, change can occur at any point in the system. Change in one family member cannot create change in other members. Individual family members are readily identified as the source of a problem.

When the family system is disrupted, change can occur at any point in the system. Although the family is the sum of the individual members, the family systems theory focuses on the number of dyad interactions that can occur. The family systems theory describes an interactional model. Any change in one member will create change in others. Change in any family member will affect other members of the family. The interactions are considered to be the problem, not the individual family members.

Orientation

When they fixate on something such as the human face like their mothers

The nurse providing care for woman with gestational diabetes understands that a lab test for glycosated hemoglobin A1C:

Would be considered evidence of good diabetes control with a result of 2.5%-5.9%

A father and a mother are carriers of phyneulketonuria (PKU). Their 2 year old daughter has PKU. The couples tell the nurse that they are planning to have a second baby. Because their daughter has PKU, they are sure that their next baby won't be affected what response by the nurse is most accurate

You are both carriers, so each baby has 25% of being affected

The recommended treatment for the prevention of human immunodeficiency virus (HIV) transmission to the fetus during pregnancy is:

Zidovudine.

A Native American woman gave birth to a baby girl 12 hours ago. The nurse notes that the woman keeps her baby in the bassinet except for feeding and states that she will wait until she gets home to begin breastfeeding. The nurse recognizes that this behavior is most likely a reflection of: delayed attachment. embarrassment. disappointment that the baby is a girl. a belief that babies should not be fed colostrum.

a belief that babies should not be fed colostrum. Delayed attachment is a cultural belief, not a delay in attachment. Embarrassment is a cultural belief, not an expression of embarrassment. This cultural belief does not indicate that there is disappointment regarding the sex of the baby. Native Americans often use cradle boards and avoid handling their newborn often; they believe that the infant should not be fed colostrum.

A nursing instructor is conducting lecture and is reviewing the functions of the female reproductive system. She asks Mark to describe the follicle-stimulating hormone (FSH) and the luteinizing hormone (LH). Mark accurately responds by stating that: a. FSH and LH are released from the anterior pituitary gland. b. FSH and LH are secreted by the corpus luteum of the ovary c. FSH and LH are secreted by the adrenal glands d. FSH and LH stimulate the formation of milk during pregnancy.

a. FSH and LH are released from the anterior pituitary gland. FSH and LH, when stimulated by gonadotropin-releasing hormone from the hypothalamus, are released from the anterior pituitary gland to stimulate follicular growth and development, growth of the graafian follicle, and production of progesterone.

A pregnant woman should be taught that the first sign of a threatened abortion is usually: a. Vaginal bleeding. b. Uterine cramping. c. Rupture of membranes. d. Backache.

a. Vaginal bleeding. Correct - The first sign of threatened abortion is vaginal bleeding, which is rather common during early pregnancy. One third of pregnant women experience bleeding in early pregnancy and up to 50% of these pregnancies end in A spontaneous abortion. The vaginal bleeding may be followed by uterine cramping and backache.

NEC inflammatory disease of the GI mucosa, the signs of NEC are nonspecific some generalized signs are:

abdominal distention, temp instability and grossly bloody stools

An infant body was just born a few minutes ago. The nurse is conducting the initial assessment. Part of the assessment includes the Apgar score. The apgar assessment is performed

at least twice, 1 minute and 5 minutes after birth

A woman in labor has a long history of uncontrolled hypertension. The hypertension has continued throughout the pregnancy and labor. The nurse is aware that the woman is at high risk for which complication? a. Placenta previa b. Abruptio placentae c. Hypotonic contractions d. DIC

b. Abruptio placentae Correct -Risk factors for abruptio placentae include maternal hypertension. Vasoconstriction is an effect of hypertension that can affect the endometrial arteries.

After a percutaneous umbilical blood sampling (PUBS) on a client, the nurse notes a fetal heart rate of 100 beats/min. The nurse is aware that this indicates a: a. Major complication, and interventions are necessary immediately. b. Common complication and is usually brief. c. Minor complication that needs to be monitored for the next 2 weeks.

b. Common complication and is usually brief. Correct - Fetal bradycardia is the most common complication after PUBS, is usually brief, and has no long-term consequences.

The prevalence of urinary incontinence increases as women age, with more than 1/3 of women in the US suffering from some form of this disorder. The symptoms of mild to moderate-to-mild UI can be successfully decreased by a number of strategies, which of these should the nurse instruct the patient to do first

bladder training and pelvic floor exercises

Premature infants who exhibit 5-10 seconds of resp. pauses followed by 10-15 seconds of compensatory rapid respiration are:

breathing in a respiratory pattern common to premature infants

A woman who is 7 months pregnant with a preexisting cardiac disease has been treated with restriction of activities and sodium intake. During a clinic visit, the woman complains of increased shortness of breath and fatigue. The nurse is aware that the next line of treatment for this woman may be: a. Anticoagulant therapy with warfarin (Coumadin). b. Complete bed rest. c. Diuretic therapy. d. Hospitalization.

c. Diuretic therapy. Correct - Diuretics may be needed when congestive heart failure is uncontrolled by restriction of activity and sodium intake.

A woman has just had a spontaneous abortion. She asks the nurse, "Why did this happen?" The nurse is aware that the most common cause of spontaneous abortion is: a. Improper maternal nutrition. b. Caffeine use in the early pregnancy. c. Severe congenital abnormalities. d. Improper implantation.

c. Severe congenital abnormalities. - Chromosomal abnormalities account for about 50% to 60% of early spontaneous abortions. Other possible causes are various types of infections and maternal disorders.

MENCONIUM

collects in the GI tract during gestation and is initially STERILE greenish black bc of occult blood and is viscous stools of breast fed neonates - are loose golden yellow after the transition to extrauterine life stools of formula fed babies are - typically soft and pale yellow after feeding's well established

The nurse should refer the patient for further testing if she noted this on inspection of the breasts of a 55 year old woman: a. left breast slightly smaller than right breast b. eversion (elevation) of both nipples c. bilateral symmetry of venous network, which is faintly visible d. small dimple located in the upper outer quadrant of the right breast

d. A small dimple is an abnormal finding and should be further evaluated

After an infertility assessment, a woman receives a prescription for clomiphene citrate (Clomid). The nurse should teach the woman that this medication is used to: a. Correct excess prolactin secretion. b.Reduce endometriosis. c.Stimulate the release of FSH and LH. d. Induce ovulation.

d. Induce ovulation. Correct - Clomiphene citrate is used to induce ovulation in women who have specific types of ovulatory dysfunction. Bromocriptine (Parlodel) corrects excessive prolactin secretion. Gonadotropin-releasing hormone (GnRH) antagonists reduce endometriosis. GnRH stimulates the release of FSH and LH.

A primigravida 40 weeks gestational is having uterine contractions every 1 ½ to 2 minutes and says that they are very painful, her cervix is dilated 2 cm and has not changes in 3 hours. The woman is crying and wants an epidural what is likely status of this woman's labor

hypertonic

Nursing care measures are commonly offered to women labor, which nursing measure reflects application of the gate-control theory

massaging the women's back

how long should normal acrocyanosis last max.

max 24 hours post birth

Leopold's Maneuvers 1st Maneuver

palpates upper abdomen with both warm hands to determine size, shape, consistency and mobility

An infant is to receive gastronomy feeding what intervention should the nurse institute to prevent bloating, GI reflux into the esophagus. Vomiting, and respiratory compromise

slow small warm bolus feedings over 20-30 mins

Leopold's Maneuvers 4th Maneuver

stand at head of bed and locate cephalic prominence or brow

A client is to have an ultrasound to do a general survey of the fetus, placenta, and amniotic fluid. This type of ultrasound level is termed ______________.

standard or basic

Convection heat loss

the flow of heat from the body surface to COOLER AIR

The nurse providing care for the laboring woman should understand that late fetal heart rate (FHR) decelerations are caused by

uteroplacental insufficiency

With regard to work and travel during pregnancy nurses should be aware that: while working or traveling in a car or on a plane

women should arranged to walk around at least every hour or so

To provide competent care to an Asian-American family, the nurse should include which of the following questions during the assessment interview?

"Do you prefer hot or cold beverages?"

While obtaining a detailed history from a woman who has recently emigrated from Somalia, the nurse realizes that the client has undergone female genital mutilation (FGM). The nurse's best response to this patient is:

"The extent of your circumcision will affect the potential for complications."

What symptom described by a woman is characteristic of premenstrual syndrome (PMS)? "I feel irritable and moody a week before my period is supposed to start." "I have lower abdominal pain beginning the third day of my menstrual period." "I have nausea and headaches after my period starts, and they last 2 to 3 days." "I have abdominal bloating and breast pain after a couple days of my period."

"I feel irritable and moody a week before my period is supposed to start." PMS is a cluster of physical, psychologic, and behavioral symptoms that begin in the luteal phase of the menstrual cycle and resolve within a couple of days of the onset of menses. PMS begins in the luteal phase and resolves as menses occurs. It does not start after menses has begun. This complaint is associated with PMS. However, the timing reflected in this statement is inaccurate. PMS begins in the luteal phase and resolves as menses occurs. It does not start after menses has begun. Abdominal bloating and breast pain are likely to occur a few days prior to menses, not after it has begun.

Which statement by the patient indicates that she understands breast self-examination?

"I will perform breast self-examination 1 week after my menstrual period starts."

Startle reflex

-How you would check -Make a loud noise -Age expected to be absent -6 months

Lochia serosa

-Lasts 4 to 10 days -Pinkish in color

Which contraceptive method protects against sexually transmitted infections and HIV

Barrier methods

Goodell's sign

Softening of the cervix caused by increased vascularization

Which statement regarding female sexual response is inaccurate?

The orgasmic phase is the final state of the sexual response cycle.

Preconception counseling is critical to the outcome of diabetic pregnancies because poor glycemic control before and during early pregnancy is associated with

congenital anomalies in the fetus

In the classification of newborns by gestational age and birth weight, the appropriate gestation age (AGA) weight would

fall between the 10th and 90th percentiles for the infant's age

Estrogen

stimulate uterine development and develops the ductal system of breasts in preparation for lactation

Human chorionic gonadotropin (hCG)

stimulates progesterone and estrogen by corpus luteum. Maintain pregnancy until placenta is developed sufficiently to assume that function

Breastfeeding preterm neonates

studies have proven that breast milk provides preterm neonates w/ better protection from infection such as NEC bc of the antibodies contained in breast milk Commercial formula doesn't provide any better nutrition than breast milk breast milk feedings can be started as soon as the neonate is stable and the neonate is more likely to develop infections when fed formula rather than breast milk

Pulmonary embolus signs

sudden dyspnea diaphoresis confusion tachycardia stationary blood clot from a varicose vein becomes an embolus (moving clot) that lodges in the pulmonary circulation chills and fever = infection

A married couple lives in a single-family house with their newborn son and the husband's daughter from a previous marriage. On the basis of the information given, what family form best describes this family?

Married-blended family

A newborn is jaundiced and is receiving phototherapy via bili lights. Which of the following is the most appropriate nursing intervention when caring for this infant?

Place eye shields over the newborn's closed eyes

A pregnant woman's amniotic membranes rupture. Prolapse cord is suspected. Which intervention is top priority?

Place the woman in a knee-chest position

A man smokes two packs of cigarettes a day. He wants to know if smoking is contributing to the difficulty he and his wife are having getting pregnant. The nurse's most appropriate response is:

"Smoking can reduce the quality of your sperm."

To ensure optimal outcomes for the patient, the contemporary maternity nurse must incorporate both teamwork and communication with clinicians into her care delivery, The SBAR technique of communication is an easy-to-remember mechanism for communication. Which of the following correctly defines this acronym?

Situation, background, assessment, recommendation

The nurse guides a woman to the examination room and asks her to remove her clothes and put on an examination gown with the front open. The woman states, "I have special undergarments that I do not remove for religious reasons." The most appropriate response from the nurse would be:

"Tell me about your undergarments. I'll explain the examination procedure, and then we can discuss how you can have your examination comfortably."

Fetal Heart Rate (FHR)

-120-160 bpm -Can be detected with a fetoscope by approximately 17 to 20 weeks -Should be counted and compared with the maternal pulse for differentiation

FHR ausculatation

-120-160 bpm -The number of times the fetal heart beats per minute -Can be detected with a fetoscope by approximately 17 to 20 weeks -Should be counted and compared with the maternal pulse for differentiation

Expected weight gain during pregnancy by trimester

-1st Trimester - 2-4 lbs. the first couple of months (3.5- 5 lbs.) -2nd Trimester - pound each week (12-15 lbs.) -3rd Trimester - pound each week (12-15 lbs.)

Sensory Changes

-As the fetus moves from a familiar, comfortable, quiet environment to one of sensory abundance, a number of physical and sensory influences help respiration being and be sustained after birth. They include the numerous tactile, auditory, visual, and painful stimuli of birth and the normal handling after delivery. -Thoroughly drying the newborn and placing him or her in skin-to-skin contact with the mother's chest and abdomen provides stimulation in a far more comforting way and also decrease heat loss.

Internal rotation

-As the occiput of the fetal head meets resistance from the levator ani muscles and their fascia, the occiput rotates from left to right and the sagittal suture aligns in the anteroposterior pelvic diameter.

External rotation

-As the shoulders rotate to the anteroposterior position in the pelvis, the head is turned farther to one side.

Signs of true Labor

-Contractions are at regular intervals -Intervals between contractions gradually shortens -Contractions increase in duration and intensity -Discomfort begins in back and radiates around the abdomen -Intensity usually increases with walking -Cervical dilatation and effacement and progressive -Contractions do not decrease with rest or warm bath

Voidings and Stools

-Day 1 -1 stool and 1 void -Day 2 -2 stools and 2 voids -Day 3 -3 stools and 3 voids -3-4 stools can be expected each day by 3-5 days of age, and 3-6 stools per day on days 5-7 -At least 3-5 voids can be expected on days 3-5 and 4-6 on days 5-7. By the 5th day on life, expect to have 5 or more wet diapers a day. -After day 4, ALL have 6-8 voids and 3 stools a day.

Late decelerations

-Decels are delayed after the uterine contraction -Deduction of blood flow through the maternal vessels -Can be caused by hypotension -Uterinplacental insufficiency -Uterus may be contractions too much due to too much pitocin

Cardiovascular Adaptions of A Newborn

-Decreases pulmonary vascular resistance and increased blood flow -Increased systemic pressure and closure of ductus venous -Increased left atrium and decreased right atrium pressure -Closure of foramen ovale -Reversal of blood flow through ductus arteriosis and increased PO2 -Closure of ductus arteriosus

Client Education Postpartum

-Demonstrates basic understanding of self-care activities and infant care needs; can identify signs of complications that should be reported -Unable to demonstrate basic self-care and infant care activities - knowledge deficit; PP blues; following prescribed cultural behavior and will be cared for my grandmother or other family member

Breasts: Palpation

-Depending on postpartal day, may be soft, filling, full, or engorged -Palpable mass - caked breast, mastitis -Engorgement - venous stasis -Tenderness, heat, edema - engorgement, caked breasts, mastitis

Timing and Purpose of Newborn screen

-Detects inborn errors of metabolism that, if left untreated, cause mental retardation and physical handicaps -In order to test for PKU the infant must be AT LEAST 24 hours old -The current goal for screening infants for hearing loss is to screen them by 1 month of age, confirm hearing loss with audiologic examination by 3 months of age, and treat with comprehensive early intervention services before 6 months of age.

Cultural Assessment Postpartum

-Determine customs and practices regarding postpartum care -Ask the mother whether she would like fluids and ask what temperature she prefers -Room-temperature or warmed fluids rather than iced drinks -Ask the other what food or fluids she would like -Special foods or fluids to hasten healing after childbirth -Ask the mother whether she would prefer to be alone during breastfeeding -Some women may be hesitant to have someone with them when their breast is exposed

Costovertebral Angle Tenderness Assessment Postpartum

-None -If present - kidney infection

Expected Weight Gain during Pregnancy

-Normal - 25-35 lbs. -Underweight - ideal weight + 25-35 lbs. -Overweight - 15-25 lbs.

Flexion

-Occurs as the fetal head descends and meets resistance from the soft tissues of the pelvis, the musculature of the pelvic floor and the cervix -As a result of the resistance the fetal chin flexes downward onto the chest.

Descent

-Occurs because of 4 forces: -Pressure of the amniotic fluid -Direct pressure of the fundus of the uterus on the breech of the fetus - Contraction of the abdominal ,isc;es -Extension and straightening of the fetal body -Head enters the inlet -The sagittal suture is an equal distance from the maternal symphysis pubis and sacral promontory.

A male client asks the nurse why it is better to purchase condoms that are not lubricated with nonoxynol-9 (a common spermicide). The nurse's most appropriate response is:

"Nonoxynol-9 does not provide protection against sexually transmitted infections, as originally thought; it has also been linked to an increase in the transmission of human immunodeficiency virus and can cause genital lesions."

A couple is trying to cope with an infertility problem. They want to know what they can do to preserve their emotional equilibrium. The nurse's most appropriate response is:

"Get involved with a support group. I'll give you some names."

A 25-year-old single woman comes to the gynecologist's office for a follow-up visit related to her abnormal Papanicolaou (Pap) smear. The test revealed that the patient has human papillomavirus (HPV). The client asks, "What is that? Can you get rid of it?" Your best response is:

"HPV stands for 'human papillomavirus.' It is a sexually transmitted infection (STI) that may lead to cervical cancer."

Which symptom described by a patient is characteristic of premenstrual syndrome (PMS)?

"I feel irritable and moody a week before my period is supposed to start."

As a girl progresses through development, she may be at risk for a number of age-related conditions. While preparing a 21-year-old client for her first adult physical examination and Papanicolaou (Pap) test, the nurse is aware of excessiveness shyness. The young woman states that she will not remove her bra because, "There is something wrong with my breasts; one is way bigger." What is the best response by the nurse in this situation?

"I will explain normal growth and breast development to you."

In assessing the knowledge of pregestational woman with type 1 diabetes concerning changing insulin needs during pregnancy, the nurse recognizes that further teaching is warrented when the client says:

"I will need to increase my insulin dosage during the first 3 months of pregnancy" (Correct: "insulin dosage will likely need to be increased during 2nd and 3rd", "episodes of hypoglycemia are more likely in first 3 months", "insulin needs should return to normal within 7-10 days after birth if I am bottle feeding"

In vitro fertilization-embryo transfer (IVF-ET) is a common approach for women with blocked fallopian tubes or unexplained infertility and for men with very low sperm counts. A husband and wife have arrived for their preprocedural interview. The husband asks the nurse to explain what the procedure entails. The nurse's most appropriate response is:

"IVF-ET is a type of assisted reproductive therapy that involves collecting eggs from your wife's ovaries, fertilizing them in the laboratory with your sperm, and transferring the embryo to her uterus."

You (the nurse) are reviewing the educational packet provided to a client about tubal ligation. What is an important fact you should point out (Select all that apply)?

"It is highly unlikely that you will become pregnant after the procedure." "Sterilization offers no protection against STIs."

A 20-year-old patient calls the clinic to report that she has found a lump in her breast. The nurse's best response is:

"Many women have benign lumps and bumps in their breasts. However, to make sure that it's benign, you should come in for an examination by your physician."

When the nurse is alone with a battered patient, the patient seems extremely anxious and says, "It was all my fault. The house was so messy when he got home and I know he hates that." The best response by the nurse is:

"No one deserves to be hurt. It's not your fault. How can I help you?"

A male client asks the nurse why it is better to purchase condoms that are not lubricated with nonoxynol-9 (a common spermicide). The nurse's most appropriate response is: "The lubricant prevents vaginal irritation." "Nonoxynol-9 does not provide protection against sexually transmitted infections, as originally thought; also it has been linked to an increase in the transmission of human immunodeficiency virus (HIV) and can cause genital lesions." "The additional lubrication improves sex." "Nonoxynol-9 improves penile sensitivity."

"Nonoxynol-9 does not provide protection against sexually transmitted infections, as originally thought; also it has been linked to an increase in the transmission of human immunodeficiency virus (HIV) and can cause genital lesions." Nonoxynol-9 may cause vaginal irritation. This is a true statement. Nonoxynol-9 has no effect on the quality of sexual activity. Nonoxynol-9 has no effect on penile sensitivity.

An unmarried young woman describes her sex life as "active" and involving "many" partners. She wants a contraceptive method that is reliable and does not interfere with sex. She requests an intrauterine device (IUD). The nurse's most appropriate response is:

"The risk of pelvic inflammatory disease (PID) will be higher for you."

A married couple is discussing alternatives for pregnancy prevention and has asked about fertility awareness methods (FAMs). The nurse's most appropriate reply is:

"They can be effective for many couples, but they require motivation."

A healthy 60-year-old African-American woman regularly receives her health care at the clinic in her neighborhood. She is due for a mammogram. At her previous clinic visit, her physician, concerned about the 3-week wait at the neighborhood clinic, made an appointment for her to have a mammogram at a teaching hospital across town. She did not keep her appointment and returned to the clinic today to have the nurse check her blood pressure. What would be the most appropriate statement for the nurse to make to this patient?

"Would you like me to make an appointment for you to have your mammogram here?"

A woman is using the basal body temperature (BBT) method of contraception. She calls the clinic and tells the nurse, "My period is due in a few days, and my temperature has not gone up." The nurse's most appropriate response is:

"You probably didn't ovulate during this cycle."

A woman who has a seizure disorder and takes barbiturates and phenytoin sodium daily asks the nurse about the pill as a contraceptive choice. The nurse's most appropriate response would be:

"Your current medications will reduce the effectiveness of the pill."

Signs of Placental Separation

(1) a globular-shaped uterus, (2) a rise of the fundus in the abdomen, (3) a sudden gush or trickle of blood, and (4) further protrusion of the umbilical cord out of the vagina.

Nursing care for hospital admission

-Establish a rapport with the patient and significant others -explain all procedures or routines, which will be carried out prior to performing them. -Orient the patient to the surroundings, initiate the patients labor chart, review info obitained, evaluate emotional status. -Evaluate the patient's preparation -evaluate possible danger signs (increased pulse, excessive bleeding, presence of meconium in amniotic fluid, alteration in fetal heart tones, obsvious change in the character of uterine contractions) -perform the admission physicians orders (administer IVs, draw lab work, send uterine activity to lab). -Shaving of pubic hair to prevent infection. -Cleansing enema. -Continue monitoring uterine contractions. -Fetal Monitoring. -Monitor vital signs every hour during labor. -Give the patient an opportunity to void. -Patient is NPO during labor the patient mind have ice chips to prevent chapping. -Assist the patient from side to side. -Try to keep the patient off her back. Best position is left side lying.

Nursing Response to Abdomen Assessment

-Evaluate size of diastasis: teach appropriate exercises for decreasing the separation. -Massage until firm; assess bladder and have woman void if needed; attempt to express clots when firm -If bogginess remains or recurs, report to physician or certified nurse midwife -Assess for evidence of endometritis -Assess for hemorrhage. Reinforce dressing and notify healthcare provider

Physiologiccal cause of fetal tachycardia

-FHR greater than 160. -Causes can be idiopathic, maternal, fetal, or a combination of maternal and fetal. -Maternal causes can be: fever, dehydratio, anxiety, betasympathomimetic drugs, maternal hyperthyroidism, and supraventricular tachycardia. -Fetal causes are early fetal hypoxia, asphyxia, fetal anemia, infection, prematurity, and prolonged fetal stimulation.

Physiological cause of fetal bradycardia

-FHR less than 110 BPM for at least a ten minute period causes: -stimulation o the vagus nerve (prolonged head compression as in early decles, application of the forceps or vacuum extractor, or prolonged scalp stimulation). -Drugs that stimulate the parasympathetic nervous system or block the sympathetic. Maternal hypotension (results in decreased blood flow to the fetus). -Prolonged umbilical cord compression. -Fetal Dysrhymia, hypoxemia, and accidental maternal pulse.

Early decelerations

-FHR mirrors uterine contractions -Squeezing of baby's head -Pushing the baby down into the position for delivery -4-7 cm dilated -This is normal

Changes in pigmentation of skin

-Facial chloasma occurs in varying degrees after week 16. -Pigmentation of nipple and areola darken. -Linea nigra - the skin in midline of the abdomen may develop a pigmented line -Striae appear on the abdomen and buttocks.

Positive signs of Pregnancy

-Fetal Heart Rate (FHR) -Fetal movement -Visualization with fetus with an ultrasound

Normal Progression of Lochia

-First 1-3 days: rubra -Failure to progress normally or return to rubra from serosa - subinvolution -Following rubra: days 3-10 : serosa (alba seldom seen in the hospital)

Quickening

-Fluttering sensation in the abdomen that gradually increases with intensity and frequency -The mother's perception of fetal movement that usually occurs at 18 to 20 weeks after the LMP. -It can occur early in a woman who has already had a pregnancy (16 weeks).

Breasts Assessment Postpartum

-General Appearance: smooth, even pigmentation, changes of pregnancy still apparent; one may appear larger -Reddened area - mastitis -Palpation: depending on postpartal day, may be soft, filling, full, or engorged -Palpable mass - caked breast, mastitis -Engorgement - venous stasis -Tenderness, heat, edema - engorgement, caked breasts, mastitis -Nipples: supple, pigmented, intact; become erect when stimulated -Fissures, cracks, soreness - problems with breastfeeding -Not erectile with stimulation - inverted nipples

Babiski reflex

-How you would check -Gently stroke the sole of each of the infant's feet -Age expected to be absent -About 12 months

Rooting Reflex

-How you would check -Lightly touch finger on cheek of infant close to their mouth -Age expected to be absent -4 to 7 months

Plantar Grasp reflex

-How you would check -Apply pressure with the finger against the balls of the infant's feet -Age expected to be absent -Lessens by 8 months

Stepping Reflex

-How you would check -Hold the infant in an upright position with one foot touching a flat surface -Age expected to be absent -4 to 8 weeks

Moro Reflex

-How you would check -Infant lying on back and suddenly releasing the infant's slightly raised head -Infant held horizontally and lowered quickly about 6 inches then stopped abruptly -Age expected to be absent -6 months

Palmar grasp reflex

-How you would check -Stimulate the palm of the infant with finger -Age expected to be absent -Lessens at 3 to 4 months

Tonic neck reflex

-How you would check -Turn infant's head to one side -Age expected to be absent -3 to 4 moths

Physical changes postpartum: GI

-Hunger following birth is common and the mother may enjoy a light meal. -Slowed during labor

Comfort Measures for Episiotomy

-Ice -Sirz bath

Nursing Response to Client Education Postpartum

-Identify predominant learning style. Determine whether woman understands English and provide reinforcement of information through conversation and through written material. -Provide information regarding infant care skills that are culturally consistent -Give woman opportunity to express her feelings and demonstrate skills taught. -Consider social service home referral for women who have no family or other support, are unable to take in information about self-care and infant care, and demonstrate no care-activities.

Physical Changes Postpartum: Reproductive system

-Involution ( the rapid reduction in the size of the uterus) -The decidua of the uterus is irregular, jagged, and varied in thickness following seperation of the placenta. -The spongy layer of the decidua is cast off as lochia, and the basal layer of the decidua remains in the uterus to become differentiated into two layers. -The outermost layer comes off as lochia as well. -After expolusion of the placenta the uterus contracts firmly to the size of a large grapefruit. -The fundus is situatied midline of the abdomen, one half to 2/3 of the way between symphysis pubis and umbilicus. -Woman are unaware of a full bladder -The top of the fundus remains at the level of the umbilicus for about half a day. -Lochia rubra is present. -The cervix will appear spongy, flabby, and formless and may appear bruised. -The vagina will appear edematous and gaping and may be bruised. -Small surperficial lacerations may be evident and the rugae have been obliterated. -Progesterone and estrogen levels drop off.

1st period of reactivity

-Lasts from birth to approximately 30 minutes after birth. -The newborn is awake and active and may appear hungry and have a strong sucking reflex. -This is an optimal period for parent-infant bonding as well as a natural opportunity to initiate breastfeeding if the mother has chosen it -Bursts of random \, diffuse movements alternating with relative immobility may occur. -Respirations are rapid, as high as 80 breaths per minute, and there may be retraction of the chest, transient flaring of the nares, and grunting. -The heart rate is rapid, and rhythm may be irregular -Bowel sounds are usually absent

Nursing care for Maternal hypotension

-Left side lying -Constant monitoring -Feeling fundus -Noting bleeding, vitals signs, boggy uterus (massage it), uterine atony (hemabate). -Check for bladder distention.

Signs and symptoms of Sepsis

-Lethargy or irritability -Hypotonia -Hypotension -Pallor, duskiness, or cyanosis -Cool and clammy skin -Temperature instability -Feeding intolerance -Hyperbilirubinemia -Tachycardia followed by apnea/bradycardia

Symptoms of hypoglycemia

-Lethargy or jitteriness -Poor feeding and sucking -Vomiting -Hypothermia and pallor -Hypotonia, tremors -Seizure activity, high pitched cry, exaggerated moro reflex

Labor signs

-Lightening -Braxton Hicks contractions -Bloody show -ROM - rupture of membranes -Fluid clear without odor -Sudden burst of energy -Weight loss -Backache -Nausea and vomiting -Diarrhea -Uterine contractions -Frequency of not less than 2 minutes -Duration of less than 75 seconds -Uterine relaxation between contraction -Most intense discomfort only with contractions -Fetal heart rate -Rate 120-160 with average variability -Absence of late decelerations -Maternal vital signs - BP below 140/90 or less than +30/+15 above -Prepregnancy readings - Pulse 60-100 - Temperature between 97.8°F and 99.6°F

Nursing care for cold stress

-Maintain NTE -Warm baby slowly -Frequent monitoring of skin temperature -Warming IV fluids -Treat accompanying hypoglycemia

Nursing care for Jaundice

-Make sure the infant is eating on a regular basis -Monitor voids and stools -Bilirubin is excreted through the urine and feces

Physical changes postpartum: Cardiovascular

-Maternal hypervolemia typically occurs because the maternal circulation has an increase in blood volume that no longer travels through the placenta. -Helps to protect from excessive blood loss. -Blood volume increase -Lose up to 500 ml during labor -Cardiac output increased -H&H increased after labor

Uterine soufflé

-May be heard when auscultation the abdomen over the uterus -A soft blowing sound at the same rate as the maternal pulse -Due to the increased uterine vascularization and the blood pulsating through the placenta -At the same rate as the FHR

Abdomen Assessment Postpartum

-Musculature: abdomen may be soft, have a "doughy" texture; rectus muscle intact -Separation in musculature - diastasis recti abdominis -Fundus: firm, midline; following expected process of involution -Boggy - full bladder, uterine bleeding, deviation from midline -May be tender when palpated -Constant tenderness - infection -Cesarean section incision dressing: dry and intact -Moderate to large amount of blood or serosanguinous drainage on dressing

During pregnancy, physiologic changes can alter the pharmacokinetics of medications. What contributes to the decreased predictability of medications? (Select all that apply)

1, Increased elimination by the kidneys 2. Plasma volume expansion 3. Increased metabolism by the liver

Extension

-The resistance of the pelvic floor and the mechanical movement of the vulva opening anteriorly and forward assist with extension of the fetal head as it passes under the symphysis pubis. - With this positional change the occiput, then brow, and face emerge from the vagina.

Placental Separation

-The uterus contracts firmly, diminishing its capacity and the surface area of placental attachment -The separation is accompanied by bleeding, leading to the formation of a hematoma between the placental tissue and the remaining dicidua. -Signs of placental separation usually appear around 5 minutes after birth of the infant, but can take up to 30 minutes to manifest.

Nursing care for the fourth stage of labor

-The uterus should be palpated but not massaged unless boggy. -The nurse inspects the lock for amount, color, odor. -Bladder should not be palpable

Elimination Assessment Postpartum

-Urinary output: voiding in sufficient quantities at least every 4 to 6 hours; bladder not palpable -In ability to void - urinary retentioin -Urgency, frequency, dysuria - UTI -Bowel elimination: should have normal bowel movement by second or third day after birth -Inability to pass feces or hard, small amount of stool - constipation due to fear of pain from episiotomy, hemorrhoids, perineal trauma

Placental Delivery

-When the signs of placental separation appear, the woman may bear down to aid in placental expulsion. -The weight of the placenta as it is guided in to the placental pan aids in the removal of the membranes from the uterine wall. -A placenta is considered to be retained if more than 30 minutes have elapsed from completion of the second stage of labor. -If the placenta separates from the inside to the outer margins is expelled with the fetal side presenting, this is called Shiny Schultze. -If the placental separates from the outer margins inward, it will roll up and present sideways with the maternal surface delivering first; this is called the Dirty Duncan.

emotional support for the second stage active phase of labor

-continue to give support -offer encouragement -give explanations. -Include significant other in these procedures. -Reinforce breathing and relaxation techniques. Accelerated shallow panting may be used, and effleurage (stroking massage)

Emotional support for the third stage transition phase of labor

-encouragement is especially important. -Assist the patient to turn on her side or sit up to prevent aspiration. -Wipe her face with a cool cloth. -Don't leave the patient - support her. -Remind patient this is the shortest stage. Use more intensive breathing techniques. -Use short simple phrases.

Emotional Support of the first stage latent phase of labor

-offer support and explanations. -Instruct or reinforce breathing techniques. DO NOT PUSH DOWN during this stage, that could case cervical edema. It could also cause cerival lacerations and fetal hypoxia.

8) A nurse prepares to administer a vitamin K injection to a newborn infant. The mother asks the nurse why her newborn infant needs the injection. The best response by the nurse would be: 1. "You infant needs vitamin K to develop immunity." 2. "The vitamin K will protect your infant from being jaundiced." 3. "Newborn infants are deficient in vitamin K, and this injection prevents your infant from abnormal bleeding." 4. "Newborn infants have sterile bowels, and vitamin K promotes the growth of bacteria in the bowel."

1. 3. Vitamin K is necessary for the body to synthesize coagulation factors. Vitamin K is administered to the newborn infant to prevent abnormal bleeding. Newborn infants are vitamin K deficient because the bowel does not have the bacteria necessary for synthesizing fat-soluble vitamin K. The infant's bowel does not have support the production of vitamin K until bacteria adequately colonizes it by food ingestion.

When attempting to interact with a neonate experiencing drug withdrawal, which behavior would indicate that the neonate is willing to interact? 1. Gaze aversion 2. Hiccups 3. Quiet alert state 4. Yawning

1. 3. When caring for a neonate experiencing drug withdrawal, the nurse needs to be alert for distress signals from the neonate. Stimuli should be introduced one at a time when the neonate is in a quiet and alert state. Gaze aversion, yawning, sneezing, hiccups, and body arching are distress signals that the neonate cannot handle stimuli at that time.

) A nurse is assessing a newborn infant who was born to a mother who is addicted to drugs. Which of the following assessment findings would the nurse expect to note during the assessment of this newborn? 1. Sleepiness 2. Cuddles when being held 3. Lethargy 4. Incessant crying

1. 4. A newborn infant born to a woman using drugs is irritable. The infant is overloaded easily by sensory stimulation. The infant may cry incessantly and posture rather than cuddle when being held.

A healthy term neonate born by C-section was admitted to the transitional nursery 30 minutes ago and placed under a radiant warmer. The neonate has an axillary temperature of 99.5*F, a respiratory rate of 80 breaths/minute, and a heel stick glucose value of 60 mg/dl. Which action should the nurse take? 1. Wrap the neonate warmly and place her in an open crib 2. Administer an oral glucose feeding of 10% dextrose in water 3. Increase the temperature setting on the radiant warmer 4. Obtain an order for IV fluid administration

1. 4. Assessment findings indicate that the neonate is in respiratory distress—most likely from transient tachypnea, which is common after cesarean delivery. A neonate with a rate of 80 breaths a minute shouldn't be fed but should receive IV fluids until the respiratory rate returns to normal. To allow for close observation for worsening respiratory distress, the neonate should be kept unclothed in the radiant warmer.

) A postpartum nurse is providing instructions to the mother of a newborn infant with hyperbilirubinemia who is being breastfed. The nurse provides which most appropriate instructions to the mother? 1. Switch to bottle feeding the baby for 2 weeks 2. Stop the breast feedings and switch to bottle-feeding permanently 3. Feed the newborn infant less frequently 4. Continue to breast-feed every 2-4 hours

1. 4. Breast feeding should be initiated within 2 hours after birth and every 2-4 hours thereafter. The other options are not necessary.

A neonate has been diagnosed with caput succedaneum. Which statement is correct about this condition? 1. It usually resolves in 3-6 weeks 2. It doesn't cross the cranial suture line 3. It's a collection of blood between the skull and the periosteum 4. It involves swelling of tissue over the presenting part of the presenting head

1. 4. Caput succedaneum is the swelling of tissue over the presenting part of the fetal scalp due to sustained pressure; it resolves in 3-4 days.

A woman delivers a 3.250 g neonate at 42 weeks' gestation. Which physical finding is expected during an examination if this neonate? 1. Abundant lanugo 2. Absence of sole creases 3. Breast bud of 1-2 mm in diameter 4. Leathery, cracked, and wrinkled skin

1. 4. Neonatal skin thickens with maturity and is often peeling by post term.

Neonates of mothers with diabetes are at risk for which complication following birth? 1. Atelectasis 2. Microcephaly 3. Pneumothorax 4. Macrosomia

1. 4. Neonates of mothers with diabetes are at increased risk for macrosomia (excessive fetal growth) as a result of the combination of the increased supply of maternal glucose and an increase in fetal insulin.

A client who is 32 weeks pregnant is being monitored in the antepartum unit for PIH. She suddenly complains of continuous abdominal pain and vaginal bleeding. Which of the following nursing internventions should be included in the care of this client? Check all that apply 1. Evaluate VS 2. Prepare for vaginal delivery 3. Reassure client that she'll be able to continue pregnancy 4. Evaluate FHT 5. Monitor amt of vaginal bleed 6. Monitor I&O

1. Evaluate VS 4. Evaluate FHT 5. Monitor amt of vaginal bleed 6. Monitor I&O The clients Sx indicate that she's experiencing abruptio placenta. The nurse must immed eval the moms well being by eval VS, FWB, by auscultation of heart tones, monitoring amt of blood loss and eval the vol status by measuring I&O. After the severity of the abruption has been determined and blood and fluid have been replaced, prompt C-SECTION delivery of the fetus (not vaginal) is indicated if the fetus is in distress

) A nurse in a delivery room is assisting with the delivery of a newborn infant. After the delivery, the nurse prepares to prevent heat loss in the newborn resulting from evaporation by: 1. Warming the crib pad 2. Turning on the overhead radiant warmer 3. Closing the doors to the room 4. Drying the infant in a warm blanket

1. Evaporation of moisture from a wet body dissipates heat along with the moisture. Keeping the newborn dry by drying the wet newborn infant will prevent hypothermia via evaporation.

Which of the following nursing actions is required before a client in labor receives an epidural? 1. Give a fluid bolus of 500 ml 2. check for maternal pupil dilation 3. assess maternal reflexes 4. assess maternal gait

1. Give a fluid bolus of 500 ml one of the major adverse effects of epidural admin is hypotension therefore, a 500ml fluid bolus is usually admin to help prevent hypotension in the client who wishes to receive an epidural for pain relief. assessments of meternal reflexes, pupil response and gait arent necessary

a client is being admitted to the antepartum unit for hypovolemia secondary to hyperemesis gravidarum. WHich of the following factors predisposes a client to the development of this? 1. trophoblastic disease 2. maternal age > 35 y.o. 3. malnourished or underweight clients 4. low levels of HCG

1. Trophoblastic disease is associated w/ hyperemesis grav obesity and maternal age younger than 20 y.o. are risk factors too. High levels of estrogen HCG have also been associated with the development.

A 21 y.o. has arrives to the ER with c/o cramping abdominal pain and mild vaginal bleeding. Pelvic exam shows a left adnexal mass that's tender when palpated. Culdocentesis shows blood in the culdesac. This client probably has which of the following conditions? 1. Abruptio placentae 2. Ecoptic pregnancy 3. Hydatidiform mole 4. Pelvic Inflammatory Disease

2. Ecoptic pregnancy most ecoptic pregnancies dont appear as obvious life threatening med emergencies. THey must be considered in any sexually active woman of childbearing age who c/o menstrual irregularity, cramping abdominal pain, and mild vaginal bleeding. PID, abruptio placentae and hydatidiform moles wont show blood in the cul de sac

During a vaginal exam of a client in labor, the nurse palpates the fetus' larger, diamond shaped fontanelle toward the anterior portion of the clients pelvis. Which of the following statements best describes this situation? 1. The client can expect a brief and intense labor, which potential for lacerations. 2. The client is at risk for uterine rupture and needs constant monitoring 3. The client may need interventions to ease back pain and change fetal position 4. The fetus will be delivered using forceps or vacuum extractor

3. The client may need interventions to ease back pain and change fetal position The fetal position is occiput posterior, a position that commonly produces intense back pain during labor. Most of the time, the fetus rotates during labor to occiput anterior position. Positioning the client on her side can facilitate this rotation. An occiput posterior position would most likely result in prolonged labor POSTERIOR - PROLONGED. occiput posterior alone doesnt create a risk of uterine rupture. The fetus wont be delivered with forceps / vacuum only if its presenting part DOESNT rotate and descend spontaneously

The nurse receives an order to start an infusion for a client whos hemorrhaging due to a placenta previa. What supplies will be needed? 1. Y tubing, normal saline solution, and 20G cathether 2. Ytubing, lactated Ringers solution and 18G cath 3. Y tubing, normal saline, 18G cath 4. Y tubing, lactated RIngers, 20G cath

3. Y tubing, normal saline, 18G cath blood transfusions require Y tubing Normal Saline solution to mix with the blood product and an 18G cath to avoid lysing breaking the RBCs. A 20G cath lumen isnt large enough for a blood transfusion. Lactated RIngers solutions isnt the IV solution of choice for blood transfusions

WHich finding might be seem in a neonate suspected of having an infection? 1. flushed cheeks 2. increased temp 3. decreased temp 4. increased activity level

3. decreased temp temp instability esp when it resutls in low temp in the neonate, may be a sign of infection. the neonate's color often changes with an infection process but generally becomes ashen or mottled the neonate w/ an infection will usually show a decrease in activity level or lethargy

Which of the following doses of Rh immune globulin RhoGAM is appropriate for a pregnant client at 28 weeks gestation? 1. 50 mcg in a sensitized client 2. 50 mcg in an unsensitized client 3. 300 mcg in a sensitized client 4. 300 mcg in a unsensitized client

4. 300 mcg in a unsensitized client An Rh negative unsensitized woman should be given 300 mcg of RhoGAM at 28 weeks after an indirect Coombs test is done to verify that sensitization hasn't occurred. For a 1st trimester abortion or ectopic pregnancy, 50 mcg of RhoGAM is given.

When performing nursing care for a neonate after a birth, which intervention has the highest nursing priority? 1. Obtain a dextrostix 2. Give the initial bath 3. Give the vitamin K injection 4. Cover the neonates head with a cap

4. Covering the neonates head with a cap helps prevent cold stress due to excessive evaporative heat loss from the neonate's wet head. Vitamin K can be given up to 4 hours after birth

A client at 42 weeks gest is 3cm dilated, 30% effaced, with membranes intact and the fetus at +2 station. FHR is at 140-150 bpm. After 2 hours, the nurse notes on the EFM that, for the past 10 min, the FHR ranged from 160-190bpm. The client states that her baby has been extremely active. UCs are strong, occurring every 3-4 min. and lasting 40-60 sec. Which of the following findings would indicate fetal hypoxia? 1. Abnormally long UCs 2. Abnormally strong uterine intensity 3. Excessively frequent contractions with rapid fetal movement 4. Excessive fetal activity and fetal tachycardia

4. Excessive fetal activity and fetal tachycardia Fetal tachycardia and excessive fetal activity are the FIRST SIGNS OF FETAL HYPOXIA The duration of UCs is w/in normal limits. Uterine intensity can be mild-strong and still be w/in normal limit. The frequency of contractions is w/in normal limits for the active phase of labor.

Neonates born to women infected with hepatitis B should undergo which Tx regimen 1. Hep B vaccine at birth and 1 month 2. Hep B immune globulin at birth, no hepatitis B vaccine 3. Hepatits B immune globulin within 48 hours of birth and Hep B vaccine at 1 month 4. Hep B immune globulin within 12 hours of birth and Hep B vaccine at birth, 1 month, 6 months

4. Hep B immune globulin within 12 hours of birth and Hep B vaccine at birth, 1 month, 6 months Hep B immune globulin should be given as soon as possible after birth but within 12 hours neonates should also receive hep B vaccine at regularly scheduled intervals this sequence of care has been determined as superior to the others

In caring for an immediate postpartum client, you note petechiae and oozing from her IV site. You would monitor her closely for the clotting disorder: A. Disseminated intravascular coagulation B. Amniotic fluid embolism C. Hemorrhage D. HELLP syndrome

A A. Correct: The diagnosis of DIC is made according to clinical findings and laboratory markers. Physical examination reveals unusual bleeding. Petechiae may appear around a blood pressure cuff on the woman's arm. Excessive bleeding may occur from the site of a slight trauma, such as venipuncture sites. B. Incorrect: These symptoms are not associated with AFE, nor is AFE a bleeding disorder. C. Incorrect: Hemorrhage occurs for a variety of reasons in the PP client. These symptoms are associated with DIC. Hemorrhage would be a finding associated with DIC and is not a clotting disorder in and of itself. D. Incorrect: HELLP is not a clotting disorder, but it may contribute to the clotting disorder DIC. p. 984

The perinatal nurse is caring for a woman in the immediate postbirth period. Assessment reveals that the woman is experiencing profuse bleeding. The most likely etiology for the bleeding is: A. Uterine atony B. Uterine inversion C. Vaginal hematoma D. Vaginal laceration

A A. Correct: Uterine atony is marked hypotonia of the uterus. It is the leading cause of postpartum hemorrhage. B. Incorrect: Uterine inversion may lead to hemorrhage, but it is not the most likely source of this client's bleeding. Furthermore, if the woman was experiencing a uterine inversion, it would be evidenced by the presence of a large, red, rounded mass protruding from the introitus. C. Incorrect: A vaginal hematoma may be associated with hemorrhage. However, the most likely clinical finding would be pain, not the presence of profuse bleeding. D. Incorrect: A vaginal laceration may cause hemorrhage, but it is more likely that profuse bleeding would result from uterine atony. A vaginal laceration should be suspected if vaginal bleeding continues in the presence of a firm, contracted uterine fundus. p. 976

The maternity nurse must be cognizant that cultural practices have significant influence on infant feeding methods. Many regional and ethnic cultures can be found within the United States. One cannot assume generalized observations about any cultural group will hold for all members of the group. Which statement related to cultural practices influencing infant feeding practice is correct? A A common practice among Mexican women is known as los dos. B Muslim cultures do not encourage breastfeeding due to modesty concerns. C Latino women born in the United States are more likely to breastfeed. D East Indian and Arab women believe that cold foods are best for a new mother.

A A common practice among Mexican women is los dos. This refers to combining breastfeeding and commercial infant formula. It is based on the belief that by combining the two feeding methods, the mother and infant receive the benefits of breastfeeding along with the additional vitamins from formula. B. Among the Muslim culture, breastfeeding for 24 months is customary. Muslim women may choose to bottle-feed formula or expressed breast milk while in the hospital. C. Latino women born in the United States are less likely to breastfeed. D. East Indian and Arab women believe that hot foods, such as chicken and broccoli, are best for the new mother. The descriptor hot has nothing to do with the temperature or spiciness of the food.

A woman is experiencing back labor and complains of constant, intense pain in her lower back. An effective relief measure is to use: A counterpressure against the sacrum. B pant-blow (breaths and puffs) breathing techniques. C effleurage. D biofeedback.

A Counterpressure is steady pressure applied by a support person to the sacral area with the fist or heel of the hand. This technique helps the woman cope with the sensations of internal pressure and pain in the lower back. Pant-blow breathing techniques are usually helpful during contractions per the gate-control theory. Effleurage is light stroking, usually of the abdomen, in rhythm with breathing during contractions. It is used as a distraction from contraction pain; however, it is unlikely to be effective for back labor. Biofeedback-assisted relaxation techniques are not always successful in reducing labor pain. Using this technique effectively requires strong caregiver support.

A woman in latent labor who is positive for opiates on the urine drug screen is complaining of severe pain. Maternal vital signs are stable, and the fetal heart monitor displays a reassuring pattern. The nurse's MOST appropriate analgesic for pain control is: A fentanyl (Sublimaze). B promethazine (Phenergan). C butorphanol tartrate (Stadol). D nalbuphine (Nubain)

A Fentanyl is a commonly used opioid agonist analgesic for women in labor. It is fast and short acting. This patient may require higher than normal doses to achieve pain relief due to her opiate use. Phenergan is not an analgesic. Phenergan is an ataractic (tranquilizer) that may be used to augment the desirable effects of the opioid analgesics but has few of those drugs' undesirable effects. Stadol is an opioid agonist-antagonist analgesic. Its use may precipitate withdrawals in a patient with a history of opiate use. Nubain is an opioid agonist-antagonist analgesic. Its use may precipitate withdrawals in a patient with a history of opiate use.

With regard to protein in the diet of pregnant women, nurses should be aware that: A many protein-rich foods are also good sources of calcium, iron, and b vitamins. B many women need to increase their protein intake during pregnancy. C as with carbohydrates and fat, no specific recommendations exist for the amount of protein in the diet. D high-protein supplements can be used without risk by women on macrobiotic diets

A Good protein sources such as meat, milk, eggs, and cheese have a lot of calcium and iron. Most women already eat a high-protein diet and do not need to increase their intake. Protein is sufficiently important that specific servings of meat and dairy are recommended. High-protein supplements are not recommended because they have been associated with an increased incidence of preterm births.

A woman diagnosed with marginal placenta previa gave birth vaginally 15 minutes ago. At the present time she is at the greatest risk for: A hemorrhage. B infection. C urinary retention. D thrombophlebitis.

A Hemorrhage is the most immediate risk because the lower uterine segment has limited ability to contract to reduce blood loss. Infection is a risk because of the location of the placental attachment site; however, it is not a priority concern at this time. Placenta previa poses no greater risk for urinary retention than with a normally implanted placenta. There is no greater risk for thrombophlebitis than with a normally implanted placenta.

A woman with severe preeclampsia has been receiving magnesium sulfate by IV infusion for 8 hours. The nurse assesses the woman and documents the following findings: temperature 37.1° C, pulse rate 96 beats/min, respiratory rate 24 breaths/min, blood pressure 155/112 mm Hg, 3+ deep tendon reflexes, and no ankle clonus. The nurse calls the physician, anticipating an order for: A hydralazine. B magnesium sulfate bolus . C diazepam. D calcium gluconate.

A Hydralazine is an antihypertensive commonly used to treat hypertension in severe preeclampsia. An additional bolus of magnesium sulfate may be ordered for increasing signs of central nervous system irritability related to severe preeclampsia (e.g., clonus) or if eclampsia develops. Diazepam sometimes is used to stop or shorten eclamptic seizures. Calcium gluconate is used as the antidote for magnesium sulfate toxicity. The client is not currently displaying any signs or symptoms of magnesium toxicity.

A pregnant woman reports that she is still playing tennis at 32 weeks of gestation. The nurse would be most concerned regarding what this woman consumes during and after tennis matches. Which is the MOST important? A Several glasses of fluid B Extra protein sources, such as peanut butter C Salty foods to replace lost sodium D Easily digested sources of carbohydrate

A If no medical or obstetric problems contraindicate physical activity, pregnant women should get 30 minutes of moderate physical exercise daily. Liberal amounts of fluid should be consumed before, during, and after exercise, because dehydration can trigger premature labor. Also the woman's calorie intake should be sufficient to meet the increased needs of pregnancy and the demands of exercise. All pregnant women should consume the necessary amount of protein in their diet, regardless of level of activity. Many pregnant women of this gestation tend to retain fluid. This may contribute to hypertension and swelling. An adequate fluid intake prior to and after exercise should be sufficient. The woman's calorie and carbohydrate intake should be sufficient to meet the increased needs of pregnancy and the demands of exercise.

With regard to umbilical cord care, nurses should be aware that: A the stump can easily become infected. B a nurse noting bleeding from the vessels of the cord should immediately call for assistance. C the cord clamp is removed at cord separation. D the average cord separation time is 5 to 7 days.

A The cord stump is an excellent medium for bacterial growth. The nurse should first check the clamp (or tie) and apply a second one. If the bleeding does not stop, then the nurse calls for assistance. The cord clamp is removed after 24 hours when it is dry. The average cord separation time is 10 to 14 days.

A 3.8-kg infant was delivered vaginally at 39 weeks after a 30-minute second stage. There was a nuchal cord. After birth the infant is noted to have petechiae over the face and upper back. Information given to the infant's parents should be based on the knowledge that petechiae: A are benign if they disappear within 48 hours of birth B result from increased blood volume C should always be further investigated D usually occur with forceps delivery

A Petechiae, or pinpoint hemorrhagic areas, acquired during birth may extend over the upper portion of the trunk and face. These lesions are benign if they disappear within 2 days of birth and no new lesions appear. Petechiae may result from decreased platelet formation. In this situation the presence of petechiae is most likely a soft-tissue injury resulting from the nuchal cord at birth. Unless they do not dissipate in 2 days, there is no reason to alarm the family. Petechiae usually occur with a breech presentation vaginal birth.

In caring for the woman with disseminated intravascular coagulation (DIC), what order should the nurse anticipate? A Administration of blood B Preparation of the woman for invasive hemodynamic monitoring C Restriction of intravascular fluids D Administration of steroids

A Primary medical management in all cases of DIC involves correction of the underlying cause, volume replacement, blood component therapy, optimization of oxygenation and perfusion status, and continued reassessment of laboratory parameters. Central monitoring would not be ordered initially in a woman with DIC because this can contribute to more areas of bleeding. Management of DIC includes volume replacement, not volume restriction. Steroids are not indicated for the management of DIC.

When planning a diet with a pregnant woman, the nurse's FIRST action would be to: A review the woman's current dietary intake. B teach the woman about the food pyramid. C caution the woman to avoid large doses of vitamins, especially those that are fat-soluble. D instruct the woman to limit the intake of fatty foods.

A Reviewing the woman's dietary intake as the first step will help to establish if she has a balanced diet or if changes in the diet are required. These are correct actions on the part of the nurse, but the first action should be to assess the patient's current dietary pattern and practices since instruction should be geared to what she already knows and does.

Nurses should be aware of the difference experience can make in labor pain, such as: A sensory pain for nulliparous women often is greater than for multiparous women during early labor. B affective pain for nulliparous women usually is less than for multiparous women throughout the first stage of labor. C women with a history of substance abuse experience more pain during labor. D multiparous women have more fatigue from labor and therefore experience more pain.

A Sensory pain is greater for nulliparous women because their reproductive tract structures are less supple. Affective pain is greater for nulliparous women during the first stage but decreases for both nulliparous and multiparous during the second stage. Women with a history of substance abuse experience the same amount of pain as those without such a history. Nulliparous women have longer labors and therefore experience more fatigue.

The nurse taught new parents the guidelines to follow regarding the bottle-feeding of their newborn. They will be using formula from a can of concentrate. The parents would demonstrate an understanding of the nurse's instructions if they: A wash the top of the can and can opener with soap and water before opening the can. B adjust the amount of water added according to the weight gain pattern of the newborn. C add some honey to sweeten the formula and make it more appealing to a fussy newborn. D warm formula in a microwave oven for a couple of minutes before feeding.

A Washing the top of the can and can opener with soap and water before opening the can of formula is a good habit for a parent to get into to prevent contamination. Directions on the can for dilution should be followed exactly and not adjusted according to weight gain to prevent nutritional and fluid imbalances. Honey is not necessary and could contain botulism spores. The formula should be warmed in a container of hot water since a microwave can easily overheat it.

The birth weight of a breastfed newborn was 8 lbs, 4 oz. On the third day the newborn's weight was 7 lbs, 12 oz. On the basis of this finding, the nurse should: A encourage the mother to continue breastfeeding since it is effective in meeting the newborn's nutrient and fluid needs. B suggest that the mother switch to bottle-feeding since the breastfeeding is ineffective in meeting newborn needs for fluid and nutrients. C notify the physician since the newborn is being poorly nourished. D refer the mother to a lactation consultant to improve her breastfeeding technique.

A Weight loss of 8 oz falls within the 5% to 10% expected weight loss from birth weight during the first few days of life, which for this newborn would be 6.6 to 13.2 oz. Breastfeeding is effective at this time. Breastfeeding is effective, and bottle-feeding does not need to be initiated at this time. The infant is not undernourished, and the physician does not need to be notified. The weight loss is within normal limits; breastfeeding is effective.

Apgar score

A scoring system used to evaluate newborns at 1 minute and 5 minutes after birth. The total score is achieved by assessing five signs: heart rate, respiratory effort, muscle tone, reflex irritability, and color. Each of the signs is assigned a score of 0, 1, or 2. The highest possible score is 10.

A nurse practitioner performs a clinical breast examination on a woman diagnosed with fibroadenoma. The nurse knows that fibroadenoma is characterized by:

A single lump in one breast that can be expected to shrink as the woman ages.

What can be done is a small hematoma develops despite preventive measures

A sitz bath after the first 12 hours will aid fluid absorption once bleeding has stopped and will promote comfort.

After reporting to the unit, you are assigned the patients listed below. Which of the patients should be evaluated first?

A woman at 11 weeks gestation with suspected ectopic pregnancy complaining of should and abdominal pain

Injectable progestins (DMPA, Depo-Provera) are a good contraceptive choice for women who:

A woman currently uses a diaphragm and spermicide for contraception. She asks the nurse what the major differences are between the cervical cap and diaphragm. The nurse's most appropriate response is:

The breasts of a bottle feeding woman are engorged. The nurse should tell her to: a. wear a snug, supportive bra b. allow warm water to soothe the breasts during a shower c. express milk from breasts occasionally to relieve discomfort d. place absorbent pads with plastic liners into her bra to absorb leakage.

A. A snug, supportive brea limits milk production and reduces discomfort by supporting the tender breasts and limiting their movement.

On review of a fetal monitor tracing, the nurse notes that for several contractions the fetal heart rate decelerates as a contraction begins and returns to baseline just before it ends. The nurse should: a. describe the finding in the nurse's notes. b. reposition the woman onto her side c. call the physician for instructions d. administer O2 at 8-10 ml with a tight face mask.

A. An early decel pattern from head compression is described. No action other than documentation of the finding is required since this is an expected reaction to compression of the fetal head as it passes through the cervix.

the term used to describe a situation in which a cultural group loses its identity and becomes part of the dominant culture is called: a. assimilation b. cultural relativism c. acculturation d. ethnocentrism

A. Assimilation is the process by which groups "melt" into the mainstream.

The nurse is assessing a woman's breast self exam technique. Which action indicates that a woman needs further instruction regarding BSE? A. Performs every month on the first day of her menstrual period B. Uses the pads of her fingers when palpating each breast c. Inspects her breasts while standing before a mirror and changing arm positions d. Places a folded towel under right shoulder and right hand under head when palpating right breast.

A. BSE should be performed once a month after the menstrual period has ended.

As part of the postpartum assessment, the nurse examines the breasts of a primiparous breastfeeding woman who is 1 day postpartum. An expected finding would be: a. presence of soft, nontender colostrum b. leakage of milk at let-down c. swollen, warm and tender on palpation d. a few blisters and a bruise on each areola.

A. Breasts are essentially unchanged for the first 2-3 days after birth. Colostrum is present and may leak from the nipples.

When performing vaginal exams on laboring women, the nurse should be guided by what principle? a. cleanse the vulva and perineum before and after the exam as needed b. bigger hands are better c. wear a clean glove lubricated with tap water to reduce discomfort d. perform immediately if active bleeding is present

A. Cleansing will reduce the possibility of secretions and microorgs ascending into the vagina to the cervix. Maternal comfort will also be enhanced.

The nurse sees a woman for the first time when she is 30 weeks pregnant. The woman has smoked throughout the pregnancy, and fundal height measurements now are suggestive of growth restriction in the fetus. In addition to ultrasound to measure fetal size, what would be another tool useful in confirming the diagnosis? A. Doppler blood flow analysis B. Contraction stress test (CST) C. Amniocentesis D. Daily fetal movement counts

A. Doppler blood flow analysis Doppler blood flow analysis allows the examiner to study the blood flow noninvasively in the fetus and the placenta. It is a helpful tool in the management of high-risk pregnancies because of intrauterine growth restriction (IUGR), diabetes mellitus, multiple fetuses, or preterm labor. Because of the potential risk of inducing labor and causing fetal distress, a CST is not performed on a woman whose fetus is preterm. Indications for an amniocentesis include diagnosis of genetic disorders or congenital anomalies, assessment of pulmonary maturity, and the diagnosis of fetal hemolytic disease, not IUGR. Fetal kick count monitoring is performed to monitor the fetus in pregnancies complicated by conditions that may affect fetal oxygenation. Although this may be a useful tool at some point later in this woman's pregnancy, it is not used to diagnose IUGR.

Parents can facilitate the adjustment of their other children to a new baby by: a. Having the children choose or make a gift to the new baby on its arrival home b. emphasizing activities that keep the new baby and other children together c. having the mother carry the new baby into the home so she can show him or her to the other children d. reducing stress on the other children by limiting their involvement in the care of the new baby

A. Having the children choose or make a gift for the new baby helps to make the child feel a part of the process.

The nurse is reviewing lab values to determine Rh incompatibility between mother and fetus. The nurse should assess which specific lab result? A. Indirect Coombs test B. Hemoglobin level C. hCG level D. Maternal serum alpha-fetoprotein (MSAFP)

A. Indirect Coombs test The indirect Coombs test is a screening tool for Rh incompatibility. If the maternal titer for Rh antibodies is greater than 1:8, amniocentesis for determination of bilirubin in amniotic fluid is indicated to establish the severity of fetal hemolytic anemia. Hemoglobin reveals the oxygen carrying capacity of the blood. hCG is the hormone of pregnancy. Maternal serum alpha-fetoprotein (MSAFP) levels are used as a screening tool for NTDs in pregnancy

The nurse is caring for a pre term infant who needs to have gavage feedings started and requires the insertion of an NG tube. Place in correct order the steps for insertion of the NG tube in a preterm infant. a. lubricate the tip of the tube with sterile water b. place infant in supine position c. measure the length of the NG tube from the tip of the nose to the love of the ear to midpoint of the xyphoid process and the umbilicus d. check placement of the NG tube by aspirating gastric contents e. gently insert the NG tube through the mouth or nose

ABCDE

On completion of a vaginal exam on a laboring woman, the nurse records: 50%, 6 cm, -1. What is a correct interpretation of the data?

A. Station of -1 indicates that the fetal presenting part is above the ischial spines and has not yet passed through the pelvic inlet.

a 42-year old woman asks the nurse about mammograms, now that she is "getting older." The nurse should tell her that: a. the american cancer society recommends mammograms every 1 to 2 years for women ages 40 to 49 b. the best time to perform a mammogram is just before a menstrual period. c. regular mammograms reduce the need to perform breast self examinations d. mammograms can confirm the diagnosis for breast cancer

A. The american cancer society recommends mammograms every 1-2 years on women between ages of 40 and 49

What has the greatest impact on reducing infant mortality in the US? a. improvements in perinatal care b. decreased incidence of congenital abnormalities c. better maternal nutrition d.improved funding for health care

A. The improvements in perinatal care, in particular respiratory care and care of the mother-baby dyad before delivery, have had the greatest impact.

The nurse is caring for a woman with mitral stenosis who is the active stage. Which action should the nurse take to promote cardiac function? a. Maintain the woman in a side lying position with the head and shoulders elevated to facilitate hemodynamics b. prepare the woman for delivery by cesarean section since this is the recommended delivery method to sustain hemodynamics c. encourage the woman to avoid the use of narcotics or epidural regional analgesia since this alters cardiac function d. promote the use of the valsalva maneuver during pushing in the second stage to improve diastolic ventricular filling.

A. The side lying position with the head and shoulders elevated helps to facilitate hemodynamics during labor.

Which is correct concerning the performance of a Papanicolau (Pap) smear? a. The woman should not douce, use vaginal medications, or have intercourse for at least 24 hours before the test. b. It should be performed once a year beginning with the onset of puberty c. A lubricant such as vaseline should be used to ease speculum insertion. d. The specimen for the Pap smear should be obtained after specimens are collected for cervical infection.

A. The woman should not douce, use vaginal medications, or have intercourse for at least 24 hours before the exam so as not to alter the cytology results.

The nurse taught new parents the guidelines to follow regarding the bottle feeding of their newborn. They will be using formula from a can of concentrate. The parents would demonstrate an understanding of the nurse's instructions if they: a. wash the top of can and can opener with soap and water before opening the can b. adjust the amount of water added according to weight gain patter of teh newborn c. add some honey to sweeten the formula and make it more appealing to a fussy newborn d. warm formula in a microwave oven for a couple of minutes prior to feeding.

A. Washing teh top of the can and can opener with soap and water before opening the can of formula is a good habit for parent to get into to prevent contamination.

The birth weight of a breastfed newborn was 8 pounds, 4 ounces. On the third day the newborn's weight was 7 pounds, 12 ounces. On the basis of this finding, the nurse should: a. encourage the mother to continue to breastfeed since it is effective in meeting the newborn's nutrient and fluid needs. b. suggest that the mother switch to bottle since breast is ineffective in meeting the newborn's needs for fluid and nutrients c. notify the physician since the newborn is being poorly nourished d. refer the mother to a lactation consultant to improve her breastfeeding technique.

A. Weight loss of 8 ounces falls within the 5% to 10% expected weight loss from birth weight during the first few days of life, which for this newborn would be 6.6 to 13.2 ounces. Breastfeeding is effective at this time.

Prior to the patient undergoing amniocentesis, the most appropriate nursing intervention is to: A. administer RhoD immunoglobulin. B. administer anticoagulant. C. send the patient for a computed tomography (CT) scan before the procedure. D. assure the mother that short-term radiation exposure is not harmful to the fetus.

A. administer RhoD immunoglobulin Because of the possibility of fetomaternal hemorrhage, administering RhoD immunoglobulin to the woman who is Rh negative is standard practice after an amniocentesis. Anticoagulants are not administered before amniocentesis as this would increase the risk of bleeding when the needle is inserted transabdominally. A CT is not required before amniocentesis, because the procedure is ultrasound guided. The mother is not exposed to radiation during amniocentesis.

Which statement made by the nurse would indicate that she or he is practicing appropriate family-centered care techniques (choose all that apply) a. the nurse allows the mother and father to make choices when possible b. the nurse informs the family about what is going to happen. The nurse instructs the patient's sister, who is a nurse, that she cannot be in the room during the birth. c. the nurse commands the mother what to do. d. the nurse provides time for the partner to ask questions.

A/D. It is important to allow for choices for the couple and to include the partner in the care process. It is also important to allow for choices for the couple and to include the partner in the care process.

The nurse is caring for an infant born at 28 weeks of gestation. Which complication could the nurse expect to observe during the course of the neonate's hospitalization? select all that apply. a. polycythemia b. respiratory distress syndrom c. meconium aspiration syndrom d. periventricular hemorrhage e. persistent pulmonary hypertension f. patent ductus arteriosus

ANSWER IS B, D AND F are complications with preterm infants A, C and E are complications of postmaturity

What is the most distinguishing clinical manifestation of abuptio placenta as opposed to placenta previa?

Abdominal pain and increased uterine tone

Part of the health assessment of newborn is observing the infant's breathing pattern. A full-term newborn's breathing pattern is predominantly

Abdominal with synchronous chest

A woman at 39 weeks with a hx of preeclampsia is admitted to L&D. She suddenly experiences increased contraction frequency of every 1-2 minutes, dark red vaginal bleeding and a tense painful abdomen. What onset should the nurse suspect?

Abruptio placenta

Which statement regarding accelerations with fetal movement is true?

Accelerations are reassuring

The term used to describe legal and professional responsibility for practice for maternity nurses is:

Accountability.

While assessing the skin of a 24 hour old, the nurse notes a pink papular rash with vesicles on the thorax, back and abdomen. What should the nurse do?

Document the findings as erythema toxicum

A postpartal woman has a 4th degree perineal laceration. Which of the following orders would you question?

Administration of an enema

A pregnant woman presents at term in labor, having no prenatal care. After birth, the infant is noted as small for GA with small eyes, flat midface, and thin upper lip and microcephalic. This woman should be questioned about?

Alcohol

With regard to lab tests and diagnostic tests after birth, nurses should be aware that which of the following statements is true?

All states test for genetic disorders when the infant is 24 hours of age

McDonald's Sign

An ease in flexing the body of the uterus against the cervix

the nurse should realize that the most common and potentially harmful maternal complication of epidural anesthesia is: a. severe postpartum headache b. limited perception of bladder fullness c. increased respiratory rate d. hypotension.

D. Ep anesthesia can lead to vasodilation and a drop in BP that could interfere with placental perfusion. The woman must be well hydrated before and during epidural anesthesia to prevent this problem and maintain an adequate blood pressure.

Which of the following instructions should be included in the nurse's teaching regarding oral contraceptives? a. Weight gain should be reported to the physician. b. An alternate method of birth control is needed when taking antibiotics. c. If the client misses one or more pills, two pills should be taken per day for 1 week. d. Changes in the menstrual flow should be reported to the physician.

Answer B is correct. When the client is taking oral contraceptives and begins antibiotics, another method of birth control should be used. Antibiotics decrease the effectiveness of oral contraceptives. Approximately 5-10 pounds of weight gain is not unusual, so answer A is incorrect. If the client misses a birth control pill, she should be instructed to take the pill as soon as she remembers the pill. Answer C is incorrect. If she misses two, she should take two; if she misses more than two, she should take the missed pills but use another method of birth control for the remainder of the cycle. Answer D is incorrect because changes in menstrual flow are expected in clients using oral contraceptives. Often these clients have lighter menses.

A client telephones the emergency room stating that she thinks that she is in labor. The nurse should tell the client that labor has probably begun when: a. Her contractions are 2 minutes apart. b. She has back pain and a bloody discharge. c. She experiences abdominal pain and frequent urination. d. Her contractions are 5 minutes apart.

Answer D is correct. The client should be advised to come to the labor and delivery unit when the contractions are every 5 minutes and consistent. She should also be told to report to the hospital if she experiences rupture of membranes or extreme bleeding. She should not wait until the contractions are every 2 minutes or until she has bloody discharge, so answers A and B are incorrect. Answer C is a vague answer and can be related to a urinary tract infection

A nurse is admitting a newborn to the nursery and learns that forceps were used during delivery. What condition should the nurse assess for in the infant? A. Torticollis B. Facial paralysis C. Fractured clavicle D. Cephalohematoma

Answer: B Rationale: Facial paralysis can occur when the forceps blades compress cranial nerve VII (facial) anterior to the ears. It is usually mild and temporary, lasting only several days. Torticollis is a deformity of the neck not associated with newborns. A fracture clavicle may occur during birth as a result of dystocia, vacuum extraction, or large birth weight. Cephalohematoma is an extravasation of blood from ruptured vessels between the skull bone and its external covering known as the periosteum. The hematoma does not cross over a cranial suture line.

An infant is born to an alcoholic mother. Which assessment finding would the nurse anticipate that would contribute to the finding of fetal alcohol syndrome? A. lethargy B. irritability C. blindness D. unresponsiveness

Answer: B Rationale: Fetal alcohol syndrome (FAS) is a congenital abnormality resulting from excessive maternal alcohol intake during pregnancy. It is characterized by typical craniofacial and limb defects, cardiovascular defects, intrauterine growth restriction and developmental delays. Newborns with FAS are very irritable and difficult to calm and comfort. Seizure activity may also occur. The other answer options A, C, and D are not associated with fetal alcohol syndrome.

Which of the following signs would alert a nurse to withdrawal in the infant of a mother addicted to heroin? A. lethargy and a lack of appetite B. restlessness, irritability, and tremors C. no crying and hypoactive reflexes D. hyperactive reflexes and diaphoresis

Answer: B Rationale: Heroin does cross the placental barrier; therefore the infant is born addicted to heroin and will display signs of withdrawal such as restlessness, irritability, and tremors. The items listed in answer options A, C, and D are not associated with heroin withdrawal.

A neonate weights 8 lb, 1 oz at birth. At age 3 days, the weight has decreased to 7 lb, 12 oz. The nurse should instruct the mother to: A. increase the amount of formula to prevent further dehydration and weight loss B. continue feeding on demand because the noted weight loss is within normal limits C. give additional feedings because the weight loss indicates inadequate caloric intake D. switch to a different formula because the current one is inadequate to maintain weight

Answer: B Rationale: Neonates tend to lose 5% -- 10% of their birth weight during the first few days after birth, mostly because of decreased, but acceptable, nutrition and extracellular fluid loss. Increasing formula volumes and feedings or changing the formula is not necessary in this situation.

What is the best technique for assessing jaundice in a newborn? A. testing capillary refill B. blanching skin on the forehead C. assessing the skin on the palm of the hands D. assisng the skin on the bottom of the feet

Answer: B Rationale: The best tecnique to assess jaundice in the newborn is to apply pressure to a bony area (e.g., forehead and observe the color of the skin before blood returns. Blanched skin is yellow if the infant is jaundiced. The answers in option A, C, and D may determine jaundice; however, they are not as good as applying pressure (blanching the skin) over a bony prominence.

The parents of a newborn question the nurse regarding the blue-black discoloration in the sacral area of their infant. The nurse explains these are known as "Mongolian spots" and: A. indicate a birth defect B. usually fade over time C. result from trauma during delivery D. are indicative of an internal problem

Answer: B Rationale: Monogolian spots are discolorations that appear in the dark-skinned infants and will fade in appearance. They are often mistaken for child abuse. Mongolian spots are not considered a birth defect, do not result from a traumatic delivery, and do not indicate an internal problem.

A nurse is assessing a newborn and and recognizes which of the following as a sign of postmaturity? A. presence of vernix caseosa B. long, brittle fingernails C. fine lanugo hair on the face, shoulders, and back D. creases in the soles of the feet

Answer: B Rationale: The fingernails begin to form around 12 weeks' gestation. By 39--40 weeks, the nails have covered the nailbeds. After 40 weeks, the nails begin to extend and have a long appearance. Presence of vernix caseosa (a cheeselike whitish substance that serves as a protective covering), a fine lanugo hair, and creases in the soles of the feet are all signs of the normal term newborn.

The nurse is assisting a new mother with breast-feeding. The nurse notes that the infant is nursing well when which behaviors are observed? (SELECT ALL THAT APPLY) A. The infants makes a clicking noise B. Swallowing is audible C. The mother reports a pulling sensation on her nipple D. The infants sucks with dimpled cheeks E. The tip of the infant's nose and chin touch the breast

Answer: B, C, E Rationale: When the infant has latched on correctly and is sucking appropriately, swallowing is audible, the mother will report a pulling sensation on the nipple, and the tip of the baby's nose, cheeks, and chin are all touching the breast. If the infant makes a clicking or smacking sound and sucks with dimpled cheeks the infant may be having trouble keeping the tongue out over the lower gum ridge. The jaw may need to be stabilized. If this is not helpful, take the baby off the breast and re-attempt latching.

An infant is born with Down syndrome should be assessed for which condition? A. Clubfoot B. Cleft palate C. Cardiac defect D. Choanal atresia

Answer: C Rationale: Approximately 30% -- 40% of infants born with Down Syndrome have congenital heart defects, typically endocardial cushion defects. Clubfoot, cleft palate, and choanal atresia are all congenital defects. Clubfoot is a deformity in which portions of the foot and ankle are twisted out of normal position. A cleft palate is incomplete closure of the palate resulting in failure of the primary palate to fuse. Choanal atresia is an obstruction of the posterior nares by tissue or bone.

A nurse explains to a new mother reasons for her newborn's cranial molding and determines that the mother needs further instruction when she makes which of the following statements? A. "The molding should disappear within a few days." B. "The molding is caused by an overriding of the cranial bones." C. "The brain may be damaged if the molding doesn't resolve quickly." D. "The amount of molding is related to the amount and length of pressure on the head."

Answer: C Rationale: Brain damage is not directly associated with cranial molding. During vaginal delivery, the cranial bones tend to override when the head accommodates the size of the mother's birth canal. The amount and length of pressure influence the degree of molding, which usually disappears in a few days without any other interventions or long-lasting effects.

A 12-hour-old infant has hemolytic disease of the newborn. What is the most common complication of this disorder? A. respiratory failure B. Liver failure C. Jaundice D. Blindness

Answer: C Rationale: Hemolytic disease of the newborn is caused by incompatibility of maternal and fetal blood types. When the fetal Rh-positive antigens or anti-A or anti-B antigens cross into the maternal circulation, the mother produces anti-Rh, anti-A, or anti-B antibodies. When the maternal anti-Rh, anti-A, or anti-B antibodies cross into the fetal circulation, these antibodies attack the fetal RBCs. The RBC destruction results in release of excess bilirubin, which the fetal or newborn's immature liver cannot metabolize; the result is newborn jaundice. Respiratory failure, liver failure, and blindness are not common complications of hemolytic disease of the newborn when appropriate treatment is provided.

At the time of delivery, the nurse assigns a newborn an Apgar score at 1 and 5 minutes. THe purpose of this scoring system is to obtain: A. the infant's initial vital signs B. a survey of gross functioning C. an initial assessment of vital functions D. an assessment of mental retardation

Answer: C Rationale: The Apgar score is an initial assessment of vital functions. These include heart rate, respiratory effort, muscle tone, reflex response, and color. The other answer options A, B, and D are not primary purposes of the Apgar score. However, vital signs and gross function are taken into consideration.

The nurse should be aware that during the childbearing experience an African-American woman is most likely to:

Arrive at the hospital in advanced labor.

Nurses should first look for the most common cause of PPH, _____, by _____. A. Lacerations of the genital tract; checking for the source of blood B. Uterine atony; evaluating the contractility of the uterus C. Inversion of the uterus; feeling for a smooth mass through the dilated cervix D. Retained placenta; noting the type of bleeding

B A. Incorrect: The leading cause of PPH is uterine atony, which complicates one in 20 births. The uterus is overstretched and contracts poorly after the birth. B. Correct: The leading cause of PPH is uterine atony, which complicates one in 20 births. The uterus is overstretched and contracts poorly after the birth. C. Incorrect: The leading cause of PPH is uterine atony, which complicates one in 20 births. The uterus is overstretched and contracts poorly after the birth. D. Incorrect: The leading cause of PPH is uterine atony, which complicates one in 20 births. The uterus is overstretched and contracts poorly after the birth. p. 976

The nurse should include which information when teaching a 15-year-old about genital tract infection prevention? (Select all that apply.) Wear nylon undergarments. Avoid tight-fitting jeans. Use floral scented bath salts. Decrease sugar intake. Do not douche. Limit time spent wearing a wet bathing suit.

Avoid tight-fitting jeans. Decrease sugar intake. Do not douche. Limit time spent wearing a wet bathing suit. Correct Feedback: Patient teaching for the prevention of genital tract infections in women includes the following guidelines: • Practice genital hygiene. • Choose underwear or hosiery with a cotton crotch. • Avoid tight-fitting clothing (especially tight jeans). • Select cloth car seat covers instead of vinyl. • Limit the time spent in damp exercise clothes (especially swimsuits, leotards, and tights). • Limit exposure to bath salts or bubble bath. • Avoid colored or scented toilet tissue. • If sensitive, discontinue use of feminine hygiene deodorant sprays. • Use condoms. • Void before and after intercourse. • Decrease dietary sugar. • Drink yeast-active milk and eat yogurt (with lactobacilli). • Do not douche. Incorrect Feedback: Patient teaching for the prevention of genital tract infections in women includes the following guidelines: • Practice genital hygiene. • Choose underwear or hosiery with a cotton crotch. • Avoid tight-fitting clothing (especially tight jeans). • Select cloth car seat covers instead of vinyl. • Limit the time spent in damp exercise clothes (especially swimsuits, leotards, and tights). • Limit exposure to bath salts or bubble bath. • Avoid colored or scented toilet tissue. • If sensitive, discontinue use of feminine hygiene deodorant sprays. • Use condoms. • Void before and after intercourse. • Decrease dietary sugar. • Drink yeast-active milk and eat yogurt (with lactobacilli). • Do not douche.

A primary nursing responsibility when caring for a woman experiencing an obstetric hemorrhage associated with uterine atony is to: A. Establish venous access B. Perform fundal massage C. Prepare the woman for surgical intervention D. Catheterize the bladder

B A. Incorrect: Although this may be a necessary intervention, the initial intervention would be fundal massage. B. Correct: The initial management of excessive postpartum bleeding is firm massage of the uterine fundus. C. Incorrect: The woman may need surgical intervention to treat her postpartum hemorrhage, but the initial nursing intervention would be to assess the uterus. D. Incorrect: After uterine massage, the nurse may want to catheterize the client to eliminate any bladder distension that may be preventing the uterus from contracting properly. p. 979

What laboratory marker is indicative of disseminated intravascular coagulation (DIC)? A. Bleeding time of 10 minutes B. Presence of fibrin split products C. Thrombocytopenia D. Hyperfibrinogenemia

B A. Incorrect: Bleeding time in DIC is normal. B. Correct: Degradation of fibrin leads to the accumulation of fibrin split products in the blood. C. Incorrect: Low platelets may occur with but are not indicative of DIC, because they may result from other coagulopathies. D. Incorrect: Hypofibrinogenemia would occur with DIC. p. 985

A pregnant woman experiencing nausea and vomiting should: A drink a glass of water with a fat-free carbohydrate before getting out of bed in the morning. B eat small, frequent meals (every 2 to 3 hours). C increase her intake of high-fat foods to keep the stomach full and coated. D limit fluid intake throughout the day.

B A pregnant woman experiencing nausea and vomiting should avoid consuming fluids early in the day or when nauseated. This is a correct suggestion for a woman experiencing nausea and vomiting. A pregnant woman experiencing nausea and vomiting should reduce her intake of fried foods and other fatty foods. A pregnant woman experiencing nausea and vomiting should avoid consuming fluids early in the morning or when nauseated but should compensate by drinking fluids at other times

A woman with severe preeclampsia is being treated with an IV infusion of magnesium sulfate. This treatment is considered successful if: A blood pressure is reduced to prepregnant baseline. B seizures do not occur. C deep tendon reflexes become hypotonic. D diuresis reduces fluid retention.

B A temporary decrease in blood pressure can occur; however, this is not the purpose of administering this medication. Magnesium sulfate is a central nervous system (CNS) depressant given primarily to prevent seizures. Hypotonia is a sign of an excessive serum level of magnesium. It is critical that calcium gluconate be on hand to counteract the depressant effects of magnesium toxicity. Diuresis is not an expected outcome of magnesium sulfate administration.

With regard to the long-term consequences of infant feeding practices, the nurse should instruct the obese client that the best strategy to decrease the risk for childhood obesity for her infant is: A an on-demand feeding schedule. B breastfeeding. C lower-calorie infant formula. D smaller, more frequent feedings.

B All breastfed infants should be fed on demand. Breastfeeding is the best prevention strategy for decreasing childhood and adolescent obesity. Breastfeeding also assists the woman to return to her prepregnant weight sooner. Lower-calorie formula is an inappropriate strategy that does not meet the infant's nutritional needs. Breastfeeding is the most appropriate choice for infant feeding. Smaller feedings are not necessary. Infants should continue to be fed every 2 to 3 hours in the newborn period.

Following circumcision of a newborn, the nurse provides instructions to his parents regarding postcircumcision care. The nurse should tell the parents to: a. apply topical anesthetics with each diaper change b. expect a yellowish exudate to cover the glans after 24 hours c. change the diaper every 2 hours and cleanse the site with soap and water or baby wipes d. apply constant pressure to the site if bleeding occurs and call the physician.

B. Parents should be taught that a yellow exudate will develop over the glans and should not be removed

Which action of a breastfeeding mother indicates the need for further instruction? A Holds breast with four fingers along bottom and thumb at top. B Leans forward to bring breast toward the baby. C Stimulates the rooting reflex and then inserts nipple and areola into newborn's open mouth. D Puts her finger into newborn's mouth before removing breast.

B Holding the breast with four fingers along the bottom and the thumb at top is a correct technique. To maintain a comfortable, relaxed position, the mother should bring the baby to the breast, not the breast to the baby. The mother would need further demonstration and teaching to correct the ineffective action. Stimulating the rooting reflex is correct. Placing the finger in the mouth to remove the baby from the breast is correct.

When caring for a newborn, the nurse must be alert for signs of cold stress, including: A decreased activity level. B increased respiratory rate. C hyperglycemia. D shivering.

B Infants experiencing cold stress would have an increased activity level. An increased respiratory rate is a sign of cold stress in the newborn. Hypoglycemia would occur with cold stress. Newborns are unable to shiver as a means of increasing heat production; they increase their activity level instead.

The nurse must administer erythromycin ophthalmic ointment to a newborn after birth. The nurse should: A instill within 15 minutes of birth for maximum effectiveness. B cleanse eyes from inner to outer canthus before administration. C apply directly over the cornea. D flush eyes 10 minutes after instillation to reduce irritation.

B Instillation of the ointment can be delayed for up to 1 hour to facilitate eye-to-eye contact between the newborn and parents, an activity that fosters bonding and attachment, especially for fathers. The newborn's eyes should be cleansed from the inner to the outer canthus before the administration of erythromycin ointment. Erythromycin should be applied into the conjunctival sac to avoid accidental injury to the eye. The eyes should not be flushed after instillation of the erythromycin.

A laboring woman becomes anxious during the transition phase of the first stage of labor and develops a rapid and deep respiratory pattern. She complains of feeling dizzy and light-headed. The nurse's immediate response would be to: A. encourage the woman to breathe more slowly. B. help the woman breathe into a paper bag. C. turn the woman on her side. D. administer a sedative.

B Just telling her to breathe more slowly does not ensure a change in respirations. The woman is exhibiting signs of hyperventilation. This leads to a decreased carbon dioxide level and respiratory alkalosis. Rebreathing her air would increase the carbon dioxide level. Turning her on her side will not solve this problem. Administration of a sedative could lead to neonatal depression since this woman, being in the transition phase, is near the birth process. The side-lying position would be appropriate for supine hypotension.

With regard to systemic analgesics administered during labor, nurses should be aware that: A systemic analgesics cross the maternal blood-brain barrier as easily as they do the fetal blood-brain barrier. B effects on the fetus and newborn can include decreased alertness and delayed sucking. C IM administration is preferred over IV administration. D IV patient-controlled analgesia (PCA) results in increased use of an analgesic.

B Systemic analgesics cross the fetal blood-brain barrier more readily than the maternal blood-brain barrier. Effects depend on the specific drug given, the dosage, and the timing. IV administration is preferred over IM administration because the drug acts faster and more predictably. PCAs result in decreased use of an analgesic.

The newborn's nurse should alert the health care provider when which newborn reflex assessment findings are seen? (Select all that apply.) A Newborn turns head toward stimulus when eliciting rooting reflex. B Newborn's fingers fan out when palmar reflex checked. C Newborn forces tongue outward when tongue touched. D Newborn exhibits symmetric abduction and extension of arms, and fingers form "C" when Moro reflex elicited. E Newborn's toes hyperextend with dorsiflexion of big toe when sole of foot stroked upward along lateral aspect.

B The baby's fingers should curl around the examiner's fingers when eliciting the palmar reflex. When eliciting rooting reflex, the characteristic response is for the baby to turn head toward stimulus and open mouth. Extrusion is elicited by touching tongue, and newborn's tongue is forced outward. The newborn should elicit symmetric abduction and extension of the arms and fingers form a "C" with the Moro reflex. The Babinski reflex is elicited by stroking upward along the lateral aspect on the sole of the feet. The expected response is hyperextension of the toes with dorsiflexion of the big toe.

A newborn male, estimated to be 39 weeks of gestation, would exhibit: A extended posture when at rest. B testes descended into scrotum. C abundant lanugo over his entire body. D ability to move his elbow past his sternum.

B The newborn's good muscle tone will result in a more flexed posture when at rest. A full-term male infant will have both testes in his scrotum and rugae on his scrotum. The newborn will exhibit only a moderate amount of lanugo, usually on his shoulders and back. The newborn would have the inability to move his elbow past midline.

An examiner who discovers unequal movement or uneven gluteal skinfolds during the Ortolani maneuver: A tells the parents that one leg may be longer than the other, but they will equal out by the time the infant is walking. B alerts the physician that the infant has a dislocated hip. C informs the parents and physician that molding has not taken place. D suggests that if the condition does not change, surgery to correct vision problems might be needed.

B This is an inappropriate statement that may result in unnecessary anxiety for the new parents. The Ortolani maneuver is a technique for checking hip integrity. Unequal movement suggests that the hip is dislocated. The physician should be notified. Molding refers to movement of the cranial bones and has nothing to do with the infant's hips. The Ortolani maneuver is not a technique used to evaluate visual acuity in the newborn. This maneuver checks hip integrity.

The nurse helps a breastfeeding mother change the diaper of her 16-hour-old newborn after the first bowel movement. The mother expresses concern since the large amount of thick, sticky stool is very dark green, almost black in color. She asks the nurse if something is wrong. The nurse should respond to this mother's concern by: A telling the mother not to worry since all breastfed babies have this type of stool. B explaining to the mother that the stool is called meconium and is expected of all newborns for the first few bowel movements. C asking the mother what she ate at her last meal. D suggesting that the mother ask her pediatrician to explain newborn stool patterns to her.

B This type of stool is the first stool that all newborns, not just breastfed babies, have. At this early age this type of stool (meconium) is typical of both bottle-fed and breastfed newborns. The mother's nutritional intake is not responsible for the appearance of meconium stool. The nurse is fully capable of and responsible for teaching a new mother about the characteristics of her newborn, including expected stool patterns.

The nurse notes that, when placed on the scale, the newborn immediately abducts and extends the arms, and the fingers fan out with the thumb and forefinger forming a "C." This response is known as a: A tonic neck reflex. B Moro reflex. C cremasteric reflex. D Babinski reflex.

B Tonic neck reflex refers to the "fencing posture" a newborn assumes when supine and turns the head to the side. These actions show the Moro reflex. The cremasteric reflex refers to retraction of testes when chilled. The Babinski reflex refers to the flaring of the toes when the sole is stroked.

Following circumcision of a newborn, the nurse provides instructions to his or her parents regarding postcircumcision care. The nurse should tell the parents to: A apply topical anesthetics with each diaper change. B expect a yellowish exudate to cover the glans after the first 24 hours. C change the diaper every 2 hours and cleanse the site with soap and water or baby wipes. D apply constant pressure to the site if bleeding occurs and call the physician.

B Topical anesthetics are applied before the circumcision. Infant-comforting techniques are generally sufficient following the procedure. Parents should be taught that a yellow exudate will develop over the glans and should not be removed. The diaper is changed frequently, but the site is cleansed with warm water only since soap and baby wipes can cause pain/burning and irritation at the site. Intermittent pressure is applied if bleeding occurs.

After change of shift report, the nurse assumes care of a multiparous patient in labor. The woman is complaining of pain that radiates to her abdominal wall, lower back, buttocks, and down her thighs. Before implementing a plan of care, the nurse should understand that this type of pain is: A visceral. B referred. C somatic. D afterpain.

B Visceral pain is that which predominates the first stage of labor. This pain originates from cervical changes, distention of the lower uterine segment, and uterine ischemia. Visceral pain is located over the lower portion of the abdomen. As labor progresses the woman often experiences referred pain. This occurs when pain that originates in the uterus radiates to the abdominal wall, the lumbosacral area of the back, the gluteal area, and thighs. The woman usually has pain only during a contraction and is free from pain between contractions. Somatic pain is described as intense, sharp, burning, and well localized. This results from stretching of the perineal tissues and the pelvic floor. This occurs during the second stage of labor. Pain experienced during the third stage of labor or afterward during the early postpartum period is uterine. This pain is very similar to that experienced in the first stage of labor.

With regard to nutritional needs during lactation, a maternity nurse should be aware that: A the mother's intake of vitamin C, zinc, and protein now can be lower than during pregnancy. B caffeine consumed by the mother accumulates in the infant, who therefore may be unusually active and wakeful. C critical iron and folic acid levels must be maintained. D lactating women can go back to their prepregnant calorie intake.

B Vitamin C, zinc, and protein levels need to be moderately higher during lactation than during pregnancy. A lactating woman needs to avoid consuming too much caffeine. The recommendations for iron and folic acid are somewhat lower during lactation. Lactating women should consume about 500 kcal more than their prepregnancy intake, at least 1800 kcal daily overall.

Which lab test result would be a cause for concern if exhibited by an Rh positive newborn 12 hours after birth? a. Direct coombs: negative b. Hematocrit (Hct): 58% and hemoglobin (Hgb): 18 g/dl c. blood glucose level: 55 mg/dl d. rapid plasma reagin (RPR)/Veneral Disease Research Labs (VDRL): reactive

D. RPR/VDRL indicates exposure to syphilis while in utero

If exhibited by an expectant father, what would be a warning sign of ineffective adaptation to his partner's first pregnancy? a. views pregnancy with pride as a confirmation of his virility b. consistently changes the subject when the topic of the fetus/newborn is raised c. expresses concern that he might faint at the birth of his baby. d. experiences nausea and fatigue, along with his partner, during the first trimester.

B. Persistent refusal to talk about the fetus-newborn may be a sign of a problem and should be assessed further.

Which of these statements are helpful and accurate nursing advice concerning bathing the new baby. (Select all that apply.) A Newborns should be bathed every day, for the bonding as well as the cleaning B Tub baths may be given before the infant's umbilical cord falls off and the umbilicus is healed. C Only plain warm water can be used to preserve the skin's acid mantle. D Powders are not recommended because the infant can inhale powder. E Bathe immediately after feeding while baby is calm and relaxed.

B, D Newborns do not need a bath every day, even if the parents enjoy it. The diaper area and creases under the arms and neck need more attention. Tub baths may be given as soon as an infant's temperature has stabilized. Unscented mild soap is appropriate to use to wash the infant. Powder is not recommended due to the risk of inhalation. Should a parent elect to use baby powder, it should never be sprinkled directly onto the baby's skin. The parent can apply a small amount of powder to his or her own hand and then apply to the infant. Do not bathe immediately after a feeding period because the increased handling may cause regurgitation.

After delivering a healthy baby boy with epidural anesthesia, a woman on the postpartum unit complains of a severe headache. The nurse should anticipate which actions in the patient's plan of care? (Select all that apply.) A Keeping the head of bed elevated at all times B Administration of oral analgesics C Avoid caffeine D Assisting with a blood patch procedure E Frequent monitoring of vital signs

B, D, E The nurse should suspect the patient is suffering from a postdural puncture headache (PDPH). Characteristically, assuming an upright position triggers or intensifies the headache, whereas assuming a supine position achieves relief (Hawkins and Bucklin, 2012). Conservative management for a PDPH includes administration of oral analgesics and methylxanthines (e.g., caffeine or theophylline). Methylxanthines cause constriction of cerebral blood vessels and may provide symptomatic relief. An autologous epidural blood patch is the most rapid, reliable, and beneficial relief measure for PDPH. Close monitoring of vital signs is essential.

BUBBLEHE

B- breast U- uterus B- bladder B- bowel L-lochia E-episiotomy/laceration H-Homans'/hemorrhoids E-emotional

A pregnant woman with a BMI of 22 asks the nurse how she should be gaining weight during pregnancy. The nurses's best response: a. a pound a week throughout pregnancy. b. 2-5 pounds during the first trimester, then a pound each week until the end of pregnancy c. a pound a week during the first two trimesters, then 2 pounds per week during the third trimester d. a total of 25-35 pounds.

B. A BMI of 22 represents a normal weight. Therefore, a total weight gain for pregnancy would be about 25-35 pounds or about 2 to 5 pounds in the first trimester and about one pound per week during the second and third trimesters.

A pregnant woman at 7 weeks of gestation complains to her nurse midwife about frequent episodes of nausea during the day with occasional vomiting. She asks what she can do to feel better. The nurse midwife could suggest that the woman: a. drink warm fluids with each of her meals. b. eat a high protein snack before going to bed. c. keep crackers and peanut butter at her bedside to eat in the morning before getting out of bed. d. schedule three meals and one mid afternoon snack a day

B. A bedtime snack of slowly digested protein is especially important to prevent the occurrence of hypoglycemia during the night that would contribute to nausea.

The nurse examines a woman 1 hour after birth. The woman's fundus is boggy, midline, and 1 cm below umbilicus. Her lochial flow is profuse, with two plum sized clots. The nurse's initial action would be to: a. place her on a bedpan to empty her bladder b. massage her fundus c. call the physician d. admin methergine, 0.2 mg IM, which has been ordered prn

B. A boggy or soft fundus indicates that uterine atony is present. This is confirmed by the profuse lochia and passage of clots. The first action would be to massage the fundus until firm.

A newborn male, estimated to be 39 weeks of gestation, would exhibit: a. extended posture when at rest b. testes descended into scrotum c. abundant lango over his entire body d. ability to move his elbow past his sternum.

B. A full term male infant will have both testes in his scrotum and rugae on his scrotum

What best describes the pattern of genetic transmission known as autosomal recessive inheritance? a. disorders in which the abnormal gene for the trait is expressed even when the other member of the pair is normal b. disorders in which both genes of a pair must be abnormal for the disorder to be expressed c. disorders in which a single gene controls the particular trait d. disorders in which the abnormal gene is carried on the X chromosome

B. An autosomal recessive inheritance disorder occurs when both genes of the pair are abnormal such as phenylketonuria or sickle cell anemia

The nurse helps a breastfeeding mother change the diaper of her 16 hour old newborn after the first bowel movement. The mother expresses concern since the large amount of thick, sticky stool is very dark green, almost black in color. She asks the nurse if something is wrong. The nurse should respond to this mother's concern by: a. telling the mother not to worry since all breastfed babies have this type of stool b. explaining to the mother that the stool is called meconium and is expected of all newborns for the first few bowel movements. c. asking the mother what she ate at her last meal d. suggesting that the mother ask her pediatrician to explain newborn stool patterns to her

B. At this early age this type of stool (meconium) is typical of both bottle-fed and breastfed newborns

What is an indicator for performing a contraction stress test? A. Increased fetal movement and small for gestational age B. Maternal diabetes mellitus and postmaturity C. Adolescent pregnancy and poor prenatal care D. History of preterm labor and intrauterine growth restriction

B. Maternal diabetes mellitus and postmaturity Decreased fetal movement is an indicator for performing a contraction stress test; the size (small for gestational age) is not an indicator. Maternal diabetes mellitus and postmaturity are two indications for performing a contraction stress test. Although adolescent pregnancy and poor prenatal care are risk factors of poor fetal outcomes, they are not indicators for performing a contraction stress test. Intrauterine growth restriction is an indicator; but history of a previous stillbirth, not preterm labor, is the other indicator.

What is an indicator for performing a contraction stress test? a. increased fetal movement and small for gestational age. b. maternal diabetes mellitus and postmaturity. c. adolescent pregnancy and poor prenatal care. d. hisotry of preterm labor and intrauterine growth restriction.

B. Maternal diabetes mellitus and postmaturity are two indications for performing a contraction stress test.

What is descriptive of the family systems theory? a. the family is viewed as the sum of individual members b. when the family system is disrupted, change can occur at any point in the system c. change in one family member cannot create change in other members d. individual family members are readily identified as the source of a problem

B. The family systems theory describes an interactional model. Any change in one member will create change in others.

The nurse must administer erythromycin ophthalmic ointment to a newborn after birth. The nurse should: a. instill within 15 minutes of birth for maximum effectiveness b. cleanse eyes from inner to outer canthus before administration c. apply directly over the cornea d. flush eyes 10 minutes after installation to reduce irritation

B. The newborn's eyes should be cleansed from the inner to the outer canthus before the administration of erythromycin ointment.

The nurse notes that, when the newborn is placed on the scale, he immediately abducts and extends his arms and his fingers fan out with the thumb and forefinger forming a "C." This response is known as a: a. tonic neck reflex b. moro reflex c. cermasteric reflex d. babinski reflex

B. These actions show the Moro reflex

Which action of a breast feeding mother indicates the need for further instruction? a. holds breast with four fingers along bottom and thumb on top. b. Leans forward to bring breast toward the baby c. Stimulates the rooting reflex and tehn inserts nipple and areola into newborn's open mouth d. puts finger into the newborn's mouth before removing breast.

B. To maintain a comfortable, relaxed position, the mother should bring the baby to the breast, not the breast to the baby. The mother would need further demonstration and teaching to correct the ineffective action.

A client's maternal serum alpha-fetoprotein (MSAFP) levels are elevated. The nurse can anticipate that the next test done will be: a. Amniocentesis. b. Ultrasound. c. Biophysical profile. d. Chorionic villus sampling.

B. Ultrasound - The MSAFP is a screening test, not a diagnostic test. If the levels are elevated, an ultrasound is offered to determine whether the abnormal concentration results from inaccurate gestational age, multifetal gestation, or fetal demise.

A woman taking an oral contraceptive as her birth control method of choice should notify her health care provider immediately if she notes: a. breast tenderness and swelling b. weight gain c. swelling and pain in one of her legs d. mood swings

C. Evidence of possible DVT and should be reported immediately.

A nonstress test (NST) is ordered on a pregnant women at 37 weeks gestation. What are the most appropriate teaching points to include when explaining the procedure to the patient? (Select all that apply) A. After 20 minutes, a nonreactive reading indicates the test is complete. B. Vibroacoustic stimulation may be used during the test. C. Drinking orange juice before the test is appropriate. D. A needle biopsy may be needed to stimulate contractions. E. Two sensors are placed on the abdomen to measure contractions and fetal heart tones.

B. Vibroacoustic stimulation may be used during the test. C. Drinking orange juice before the test is appropriate. A nonreactive test requires further evaluation. The testing period is often extended, usually for an additional 20 minutes, with the expectation that the fetal sleep state will change and the test will become reactive. During this time vibroacoustic stimulation (see later discussion) may be used to stimulate fetal activity. Vibroacoustic stimulation is often used to stimulate fetal activity if the initial NST result is nonreactive and thus hopefully shortens the time required to complete the test (Greenberg, Druzin, and Gabbe, 2012). Care providers sometimes suggest that the woman drink orange juice or be given glucose to increase her blood sugar level and thereby stimulate fetal movements. Although this practice is common, there is no evidence that it increases fetal activity (Greenberg, Druzin, and Gabbe, 2012). A needle biopsy is not part of a NST. The FHR is recorded with a Doppler transducer, and a tocodynamometer is applied to detect uterine contractions or fetal movements. The tracing is observed for signs of fetal activity and a concurrent acceleration of FHR.

A primiparous woman is in the taking in stage of psychosocial recovery and adjustment following birth. Recognizing the needs of women during this stage, the nurse should: a. foster an active role in the baby's care b. provide time for the mother to reflect on the events of and her behavior during childbirth c. recognize the woman's limited attention span by giving her written materials to read when she gets home rather than doing a teaching session now. d. promote maternal independence by encouraging her to meet her own hygiene and comfort needs.

B. Women express a need to review their childbirth experience and evaluate their performance.

Semen analysis is a common diagnostic procedure related to infertility. In instructing g a male patient regarding this test, the nurse would tell him to: a. ejaculate into a sterile container. b. Obtain the specimen after a period of abstinence from ejaculation of 2 to 5 days. c. Take a victoria's secret magazine in with him to do the deed. d. ensure that the specimen arrives at the lab within 30 minutes of ejaculation

B. an ejaculated sample should be obtained after a period of abstinence to get the best results. A magazine may be helpful tho.

A woman at 25 weeks gestation tells the nurse that she dropped a pan last week and her baby jumped at the noised. Which response by the nurse is most accurate?

Babies respond to sound starting at around 24 weeks of gestation

Four 38 gestation woman have just deliver. Which of the babies should be monitored closely for respiratory distress o Baby whose mother has DM (diabetes mellitus) o Baby whose mother has lung cancer o Baby whose mother has hyperthyroidism o Baby whose mother has asthma

Baby whose mother has DM (diabetes mellitus)

On vaginal examination of 30 year old women nurse documents the following findings: profuse, thin, grayish white vaginal discharge with a "fishy" odor, complaints of pruritus, On the basis of these findings the nurse suspects that this women has

Bacterial vaginitis (BV)

On vaginal examination of a 30-year-old woman, the nurse documents the following findings: profuse, thin, grayish white vaginal discharge with a "fishy" odor; complaint of pruritus. On the basis of these findings, the nurse suspects that this woman has:

Bacterial vaginosis (BV).

Care management of a woman diagnosed with acute pelvic inflammatory disease (PID) most likely would include:

Bed rest in a semi-Fowler position.

41 week pregnant multigravida present in labor and delivery unit after a non-stress test indicated that her fetus could be experiencing some difficulties in utero, which diagnostic tool would yield more detailed info about the fetus:

Biophysical Profile

Recent trends in childbirth practices in the United States indicate that:

Births occurring in the hospital accounted for 99% of births.

The nurse caring for a pregnant client should be aware that the U.S. birth rate shows which trend?

Births to unmarried women are more likely to have less favorable outcomes.

With regard to medications, herbs, shots, and other substances normally encountered

Both prescription and OTC drugs that otherwise are harmless can be made hazardous by metabolic deficiencies of the fetus

All of these statements indicate impacts of breastfeeding on the family or society at large except

Breast feeding costs employers in terms of time lost from work

A pregnant woman demonstrates understanding of the nurse's instructions regarding relief of leg craps if she: a. wiggles and points her toes during the cramp. b. applies cold compresses to teh affected leg. c. extends her leg and dorsiflexes her foot during the cramp. d. avoids weight bearing on the affected leg during the crampl

C. Extending the leg and dorsiflexing the foot is the appropriate relief for a leg cramp

Despite popular belief, there is a rare type of hemophilia that affects women of childbearing age. Von Willebrand disease is the most common of the hereditary bleeding disorders and can affect males and females alike. It results from a factor VIII deficiency and platelet dysfunction. Although factor VIII levels increase naturally during pregnancy, there is an increased risk for postpartum hemorrhage from birth until 4 weeks postdelivery as levels of von Willebrand factor (vWf) and factor VIII decrease. The treatment that should be considered first for the client with von Willebrand disease who experiences a postpartum hemorrhage is: A. Cryoprecipitate B. Factor VIII and vWf C. Desmopressin D. Hemabate

C A. Incorrect: Cryoprecipitate may be used; however, because of the risk of possible donor viruses, other modalities are considered safer. B. Incorrect: Treatment with plasma products such as factor VIII and vWf are an acceptable option for this client. Because of the repeated exposure to donor blood products and possible viruses, this is not the initial treatment of choice. C. Correct: Desmopressin is the primary treatment of choice. This hormone can be administered orally, nasally, and intravenously. This medication promotes the release of factor VIII and vWf from storage. D. Incorrect: Although the administration of this prostaglandin is known to promote contraction of the uterus during postpartum hemorrhage, it is not effective for the client who presents with a bleeding disorder. pp. 981, 984

What infection is contracted mostly by first-time mothers who are breastfeeding? A. Endometritis B. Wound infections C. Mastitis D. Urinary tract infections

C A. Incorrect: Mastitis is infection in a breast, usually confined to a milk duct. Most women who suffer this are first-timers who are breastfeeding. B. Incorrect: Mastitis is infection in a breast, usually confined to a milk duct. Most women who suffer this are first-timers who are breastfeeding. C. Correct: Mastitis is infection in a breast, usually confined to a milk duct. Most women who suffer this are first-timers who are breastfeeding. D. Incorrect: Mastitis is infection in a breast, usually confined to a milk duct. Most women who suffer this are first-timers who are breastfeeding. p. 988

A woman who has recently given birth complains of pain and tenderness in her leg. Upon physical examination, the nurse notices warmth and redness over an enlarged, hardened area. The nurse should suspect _____ and should confirm the diagnosis by _____. A. Disseminated intravascular coagulation; asking for laboratory tests B. von Willebrand disease; noting whether bleeding times have been extended C. Thrombophlebitis; using real time and color Doppler ultrasound D. Coagulopathies; drawing blood for laboratory analysis

C A. Incorrect: Pain and tenderness in the extremities, which show warmth, redness, and hardness, is likely thrombophlebitis. A Doppler ultrasound is a common, noninvasive way to confirm the diagnosis. B. Incorrect: Pain and tenderness in the extremities, which show warmth, redness, and hardness, is likely thrombophlebitis. A Doppler ultrasound is a common, noninvasive way to confirm the diagnosis. C. Correct: Pain and tenderness in the extremities, which show warmth, redness, and hardness, is likely thrombophlebitis. A Doppler ultrasound is a common noninvasive way to confirm diagnosis. D. Incorrect: Pain and tenderness in the extremities, which show warmth, redness, and hardness, is likely thrombophlebitis. A Doppler ultrasound is a common, noninvasive way to confirm the diagnosis. p. 985

The first and most important nursing intervention when a nurse observes profuse postpartum bleeding is to: A. Call the woman's primary health care provider B. Administer the standing order for an oxytocic C. Palpate the uterus and massage it if it is boggy D. Assess maternal blood pressure and pulse for signs of hypovolemic shock

C A. Incorrect: This intervention is appropriate. However, the primary intervention should be to assess the uterus. Uterine atony is the leading cause of PPH. B. Incorrect: This intervention is appropriate. However, the primary intervention should be to assess the uterus. Uterine atony is the leading cause of PPH. C. Correct: The initial management of excessive postpartum bleeding is firm massage of the uterine fundus. D. Incorrect: This intervention is appropriate. However, the primary intervention should be to assess the uterus. Uterine atony is the leading cause of PPH. p. 979

The nurse is preparing to discharge a 30-year-old woman who has experienced a miscarriage at 10 weeks of gestation. Which statement by the woman would indicate a correct understanding of the discharge instructions? A "I will not experience mood swings since I was only at 10 weeks of gestation." B "I will avoid sexual intercourse for 6 weeks and pregnancy for 6 months." C "I should eat foods that are high in iron and protein to help my body heal." D "I should expect the bleeding to be heavy and bright red for at least 1 week."

C After a miscarriage a woman may experience mood swings and depression from the reduction of hormones and the grieving process. Sexual intercourse should be avoided for 2 weeks or until the bleeding has stopped and should avoid pregnancy for 2 months. A woman who has experienced a miscarriage should be advised to eat foods that are high in iron and protein to help replenish her body after the loss. The woman should not experience bright red, heavy, profuse bleeding; this should be reported to the health care provider.

Signs of a threatened abortion (miscarriage) are noted in a woman at 8 weeks of gestation. What is an appropriate management approach for this type of abortion? A Prepare the woman for a dilation and curettage (D&C). B Place the woman on bed rest for at least 1 week and reevaluate. C Prepare the woman for an ultrasound and blood work. D Comfort the woman by telling her that if she loses this baby, she may attempt to get pregnant again in 1 month.

C D&C is not considered until signs of the progress to an inevitable abortion are noted or the contents are expelled and incomplete. Bed rest is recommended for 48 hours initially. Repetitive transvaginal ultrasounds and measurement of human chorionic gonadotropin (hCG) and progesterone levels may be performed to determine if the fetus is alive and within the uterus. If the pregnancy is lost, the woman should be guided through the grieving process. Telling the client that she can get pregnant again soon is not a therapeutic response because it discounts the importance of this pregnancy.

A woman is in the second stage of labor and has a spinal block in place for pain management. The nurse obtains the woman's blood pressure and notes that it is 20% lower than the baseline level. Which action should the nurse take? A Encourage her to empty her bladder. B Decrease her intravenous (IV) rate to a keep vein-open rate. C Turn the woman to the left lateral position or place a pillow under her hip. D No action is necessary since a decrease in the woman's blood pressure is expected.

C Encouraging the woman to empty her bladder will not help the hypotensive state and may cause her to faint if she ambulates to the bathroom. The IV rate should be kept at the current rate or increased to maintain the appropriate perfusion. Turning the woman to her left side is the best action to take in this situation since this will increase placental perfusion to the infant while waiting for the doctor's or nurse midwife's instruction. Hypotension indicated by a 20% drop from preblock level is an emergency situation and action must be taken.

With regard to what might be called the tactile approaches to comfort management, nurses should be aware that: A either hot or cold applications may provide relief, but they should never be used together in the same treatment. B acupuncture can be performed by a skilled nurse with just a little training. C hand and foot massage may be especially relaxing in advanced labor when a woman's tolerance for touch is limited. D therapeutic touch (TT) uses handheld electronic stimulators that produce sympathetic vibrations.

C Heat and cold may be applied in an alternating fashion for greater effect. Unlike acupressure, acupuncture, which involves the insertion of thin needles, should be done only by a certified therapist. The woman and her partner should experiment with massage before labor to see what might work best. Therapeutic touch is a laying-on of hands technique that claims to redirect energy fields in the body.

While evaluating an external monitoring tracing and in active labor, the nurse notes that the fetal heart rate (FHR) for 5 sequential contractions begins to decelerate late in the contraction with nadir of the decelerations occurring after the peak of contraction, the nurse's first concern is

Change the women's position

In helping the breastfeeding mother position the baby, nurses should keep in mind that: A the cradle position is usually preferred by mothers who had a cesarean birth. B women with perineal pain and swelling prefer the modified cradle position. C whatever the position used, the infant is "belly to belly" with the mother. D while supporting the head, the mother should push gently on the occiput.

C The football position usually is preferred after cesarean birth. Women with perineal pain and swelling prefer the side-lying position because they can rest while breastfeeding. The infant inevitably faces the mother, belly to belly. The mother should never push on the back of the head. It may cause the baby to bite, hyperextend the neck, or develop an aversion to being brought near the breast.

What is the PRIORITY teaching tip the nurse should provide about bottle-feeding? A Infants may stool with each feeding in the first weeks. B Feed newborn at least every 3 to 4 hours. C Hold infant semiupright while feeding. D Some infants take longer to feed than others.

C The infant may have a stool with each feeding in the first 2 weeks, although this amount may decrease to one or two stools each day Newborns should be fed at least every 3 to 4 hours and should never go longer than 4 hours without feeding until a satisfactory pattern of weight gain is established. Infants should be held and never left alone while feeding. Never prop the bottle. The infant might inhale formula or choke on any that was spit up. Airway is priority. Taking a few sucks and then pausing briefly before continuing to suck again is normal for infants. Some infants take longer to feed than others. Be patient. Keep the baby awake; encouraging sucking may be necessary. Moving the nipple gently in the infant's mouth may stimulate sucking.

Newborns are at high risk for injury if appropriate safety precautions are not implemented. Parents should be taught to: A place the newborn on the abdomen (prone) after feeding and for sleep. B avoid use of pacifiers. C use a rear-facing car seat. D use a crib with side rail slats that are no more than 3 inches apart.

C The prone position is no longer recommended since it may interfere with chest expansion and lead to sudden infant death syndrome. Approved pacifiers are safe to use and fulfill a newborn's need to suck. If the newborn is breastfed, the use of pacifiers should be delayed until breastfeeding is well established to avoid the development of nipple confusion. Your baby should be in a rear-facing infant car safety seat from birth until age 2 years or until exceeding the car seat's limits for height and weight. Slats in a crib should be no more than 2 inches apart.

Vitamin K is given to the newborn to: A reduce bilirubin levels. B increase the production of red blood cells. C enhance ability of blood to clot. D stimulate the formation of surfactant.

C Vitamin K does not reduce bilirubin levels. Vitamin K does not increase the production of red blood cells. Newborns have a deficiency of vitamin K until intestinal bacteria that produce vitamin K are formed. Vitamin K is required for the production of certain clotting factors. Vitamin K does not stimulate the formation of surfactant.

An effective relief measure for primary dysmenorrhea would be to: a. reduce physical activity level until menstruation ceases b. begin taking prostaglandin synthesis inhibitors on the first day of the menstrual flow c. decrease intake of salt and refined sugar about 1 week before menstruation is about to occur d. use barrier methods rather than oral contraceptives for birth control

C. Decreasing intake of salt and refined sugar cana reduce fluid retention.

A pregnant client has a 2-year history of uncontrolled hypertension. The nurse can anticipate that which fetal study to be ordered? a. Amniocentesis b. Chorionic villus sampling c. Doppler ultrasound blood flow assessment

C. Doppler ultrasound blood flow assessment Correct -Pregnancies complicated by hypertension may have a Doppler ultrasound assessment of blood flow through the umbilical artery done to identify abnormalities in the diastolic flow.

During the first trimester the pregnant woman would be most motivated to learn about: a. fetal development b. impact of a new baby on family members c. measures to reduce nausea and fatigue so she can feel better d. location of childbirth prep and breastfeeding classes

C. During the first trimester a woman is egocentric and concerned about how she feels. She is working on the task of accepting her pregnancy.

Preconception counseling is critical to the outcome of diabetic pregnancies because poor glycemic control before and during early pregnancy is associated with which of the following?

Congenital abnormalities or malformation of the fetus

When the nurse is unsure how to perfor a client care procedure, which would be the best action for the nurse to take

Consult the agency procedure manual and follow the guidelines for the procedure

The ___ is/are responsible for oxygen and carbon dioxide transport to and from the maternal bloodstream

Chorionic villi

Which of the following is responsible for oxygen and carbon dioxide transport to and from the maternal bloodstream?

Chorionic villi

A mother expresses fear about changing her infant's diaper after he is circumcised. What does the woman need to be taught to take care of the infant when she gets home?

Cleanse the penis gently with water and put petroleum jelly around the glans after each diaper change

As a powerful CNS stimulant, which of these substances can lead to a miscarriage, preterm labor, abruptio placenta, and stillbirth?

Cocaine

As a powerful central nervous system stimulant, which of these substances can lead to miscarriage, preterm labor, placental separation (abruption), and stillbirth?

Cocaine

A newborn is placed under a heat warmer and the nurse evaluates temp every hour. Maintaining temp is important for preventing which?

Cold stress

Placenta previa

Complication of pregnancy in which the placenta grows in the lowest part of the womb and covers all or part of the opening to the cervix. Abnormal implantation of the placenta low in the uterus near or covering the cervical os

One of the first symptoms of puerperal infection to assess for in the postpartum woman is: A. Fatigue continuing for longer than 1 week B. Pain with voiding C. Profuse vaginal bleeding with ambulation D. Temperature of 38° C (100.4° F) or higher on 2 successive days starting 24 hours after birth

D A. Incorrect: Fatigue would be a late finding associated with infection. B. Incorrect: Pain with voiding may indicate a UTI, but it is not typically one of the earlier symptoms of infection. C. Incorrect: Profuse lochia may be associated with endometritis, but it is not the first symptom associated with infection. D. Correct: Postpartum or puerperal infection is any clinical infection of the genital canal that occurs within 28 days after miscarriage, induced abortion, or childbirth. The definition used in the United States continues to be the presence of a fever of 38° C (100.4° F) or higher on 2 successive days of the first 10 postpartum days, starting 24 hours after birth. p. 987

When caring for a postpartum woman experiencing hemorrhagic shock, the nurse recognizes that the most objective and least invasive assessment of adequate organ perfusion and oxygenation is: A. Absence of cyanosis in the buccal mucosa B. Cool, dry skin C. Diminished restlessness D. Urinary output of at least 30 ml/hr

D A. Incorrect: The assessment of the buccal mucosa for cyanosis can be subjective in nature. B. Incorrect: The presence of cool, pale, clammy skin would be an indicative finding associated with hemorrhagic shock. C. Incorrect: Hemorrhagic shock is associated with lethargy, not restlessness. D. Correct: Hemorrhage may result in hemorrhagic shock. Shock is an emergency situation in which the perfusion of body organs may become severely compromised, and death may occur. The presence of adequate urinary output indicates adequate tissue perfusion. p. 983

It is important for the perinatal nurse to be knowledgeable regarding conditions of abnormal adherence of the placenta. This occurs when the zygote implants in an area of defective endometrium and results in little to no zone separation between the placenta and decidua. Which classification of separation is not recognized as an abnormal adherence pattern? A. Placenta accreta B. Placenta increta C. Placenta percreta D. Placenta abruptio

D A. Incorrect: This is a recognized degree of attachment. With placenta accreta there is slight penetration of the trophoblast into the myometrium. B. Incorrect: This is a recognized degree of attachment that results in deep penetration of the myometrium. C. Incorrect: This is the most severe degree of placental penetration that results in deep penetration of the myometrium. Bleeding with complete placental attachment will occur only when separation of the placenta is attempted after delivery. Treatment includes blood component therapy and in extreme cases, hysterectomy may be necessary. D. Correct: Placenta abruptio is premature separation of the placenta as opposed to partial or complete adherence. This occurs between the 20th week of gestation and delivery in the area of the decidua basilis. Symptoms include localized pain and bleeding. pp. 822, 977

Nurses need to know the basic definitions and incidence data about postpartum hemorrhage. For instance: A. PPH is easy to recognize early; after all, the woman is bleeding. B. Traditionally, it takes more than 1000 ml of blood after vaginal birth and 2500 ml after cesarean birth to define the condition as PPH. C. If anything, nurses and doctors tend to overestimate the amount of blood loss. D. Traditionally, PPH has been classified as early or late with respect to birth.

D A. Incorrect: Unfortunately, PPH can occur with little warning and often is recognized only after the mother has profound symptoms. B. Incorrect: Traditionally, a 500 ml blood loss after a vaginal birth and a 1000 ml blood loss after a cesarean birth constitute PPH. C. Incorrect: Medical personnel tend to underestimate blood loss by as much as 50% in their subjective observations. D. Correct: Early PPH is also known as primary, or acute, PPH; late PPH is known as secondary PPH. p. 975

A 22-year-old woman pregnant with a single fetus has a preconception body mass index (BMI) of 24. When she was seen in the clinic at 14 weeks of gestation, she had gained 1.8 kg (4 lbs) since conception. How would the nurse interpret this? A This weight gain indicates possible gestational hypertension. B This weight gain indicates that the woman's infant is at risk for intrauterine growth restriction (IUGR). C This weight gain cannot be evaluated until the woman has been observed for several more weeks. D The woman's weight gain is appropriate for this stage of pregnancy.

D Although this is an accurate statement, it does not apply to this client. The desirable weight gain during pregnancy varies among women. The primary factor to consider in making a weight gain recommendation is the appropriateness of the prepregnancy weight for the woman's height. A commonly used method of evaluating the appropriateness of weight for height is the BMI. This woman has gained the appropriate amount of weight for her size at this point in her pregnancy. C. Weight gain should take place throughout the pregnancy. The optimal rate of weight gain depends on the stage of the pregnancy. This is an accurate statement. This woman's BMI is within the normal range. During the first trimester, the average total weight gain is only 1 to 2.5 kg.

A woman at 39 weeks of gestation with a history of preeclampsia is admitted to the labor and birth unit. She suddenly experiences increased contraction frequency of every 1 to 2 minutes; dark red vaginal bleeding; and a tense, painful abdomen. The nurse suspects the onset of: A eclamptic seizure. B rupture of the uterus. C placenta previa. D placental abruption.

D Eclamptic seizures are evidenced by the presence of generalized tonic-clonic convulsions. Uterine rupture presents as hypotonic uterine activity, signs of hypovolemia, and in many cases the absence of pain. Placenta previa presents with bright red, painless vaginal bleeding. Uterine tenderness in the presence of increasing tone may be the earliest finding of premature separation of the placenta (abruptio placentae or placental abruption). Women with hypertension are at increased risk for an abruption.

Which statement regarding infant weaning is correct? A Weaning should proceed from breast to bottle to cup. B The feeding of most interest should be eliminated first. C Abrupt weaning is easier than gradual weaning. D Weaning can be mother or infant initiated.

D Infants can be weaned directly from the breast to a cup. Bottles are usually offered to infants less than 6 months. If the infant is weaned before 1 year of age, iron-fortified formula rather than cow's milk should be offered. The feeding of least interest to the baby or the one through which the infant is likely to sleep should be eliminated first. Every few days thereafter the mother drops another feeding. Gradual weaning over a period of weeks or months is easier for both the mother and the infant than an abrupt weaning. Weaning is initiated by the mother or the infant. With infant-led weaning, the infant moves at his or her own pace in omitting feedings, which leads to a gradual decrease in the mother's milk supply. Mother-led weaning means that the mother decides which feedings to drop.

When weighing a newborn, the nurse should: A leave its diaper on for comfort. B place a sterile scale paper on the scale for infection control. C keep hand on the newborn's abdomen for safety. D weigh the newborn at the same time each day for accuracy.

D The baby should be weighed without a diaper or clothes. Clean scale paper is acceptable; it does not need to be sterile. The nurse's hand should be above, not on, the abdomen for safety. Weighing a newborn at the same time each day allows for accurate weights.

When counseling a client about getting enough iron in her diet, the maternity nurse should tell her that: A milk, coffee, and tea aid iron absorption if consumed at the same time as iron. B iron absorption is inhibited by a diet rich in vitamin C. C iron supplements are permissible for children in small doses. D constipation is common with iron supplements.

D These beverages inhibit iron absorption when consumed at the same time as iron. Vitamin C promotes iron absorption. Children who ingest iron can get very sick and even die. Constipation can be a problem.

A woman who is at 36 weeks of gestation is having a nonstress test. Which statement by the woman would indicate a correct understanding of the test? A. "I will need to have a full bladder for the test to be done accurately." B. "I should have my husband drive me home after the test because I may be nauseous." C. "This test will help to determine if the baby has Down syndrome or a neural tube defect." D. "This test will observe for fetal activity and an acceleration of the fetal heart rate to determine the well-being of the baby."

D. "This test will observe for fetal activity and an acceleration of the fetal heart rate to determine the well-being of the baby." An ultrasound is the test that requires a full bladder. An amniocentesis would be the test that a pregnant woman should be driven home afterward. A maternal alpha-fetoprotein test is used in conjunction with unconjugated estriol levels, and human chorionic gonadotropin helps to determine Down syndrome. The nonstress test is one of the most widely used techniques to determine fetal well-being and is accomplished by monitoring fetal heart rate in conjunction with fetal activity and movements.

What lab results would be a cause of concern if exhibited by a woman at her first prenatal visit during the second month of pregnancy? a. Hematocrit 38%, hemoglobin 13 g/dl b. WBC 6000/mm3 c. platelets 300,000/mm3 d. rubella titre 1:6

D. A rubella titer of less than 1:10 indicates a lack of immunity to rebella, a viral infection that has the potential to cause teratogenic effects on fetal development. Arrangements should be made to administer the rubella vaccine after birth during the postpartum period since administration of rubella, a live vaccine, would be contraindicated during pregnancy. Women receiving the vaccine during the postpartum period should be cautioned to avoid pregnancy for 3 months.

A maternal serum alpha fetoprotein test is performed at 16-18 weeks of gestation. An elevated level has been associated with: a. Down syndrome b. Sickle cell anemia c. cardiac defects d. open neural tube defects such as spina bifida

D. A triple marker test determines the levels of MsAFP along with serum levels of estriol and human chorionic gonadotropin; an elevated level is associated with open neural tube defects.

If exhibited by a pregnant woman, what represents a positive sign of pregnancy? a. Morning sickness b. quickening c. positive pregnancy test d. fetal heartbeat auscultated with doppler/fetoscope

D. Detection of a fetal heartbeat, palpation of fetal movements and parts by an examiner, and detection of the embryo/fetus with sonographic exam would be positive signs diagnostic of pregnancy.

A pregnant woman at 32 weeks of gestation complains of feeling dizzy and light-headed while her fundal height is being measured. Her skin is pale and moist. The nurse's initial response would be to:

D. During fundal height measurement the woman is placed in a supine position. This woman is experiencing supine hypotension as a result of uterine compression of the vena cava and abdominal aorta. Turning her on her side will remove the compression and restore cardiac output and blood pressure.

The health care provider has ordered a magnetic resonance imaging (MRI) study to be done on a pregnant patient to evaluate fetal structure and growth. The nurse should include which instructions when preparing the patient for this test? (Select all that apply.) A. A lead apron must be worn during the test. B. A full bladder is required prior to the test. C. An intravenous line must be inserted before the test. D. Jewelry must be removed before the test. E. Remain still throughout the test.

D. Jewelry must be removed before the test. E. Remain still throughout the test. Magnetic resonance imaging (MRI) is a noninvasive radiologic technique used for obstetric and gynecologic diagnosis. Similar to computed tomography (CT), MRI provides excellent pictures of soft tissue. Unlike CT, ionizing radiation is not used. Therefore vascular structures within the body can be visualized and evaluated without injecting an iodinated contrast medium, thus eliminating any known biologic risk. Similar to sonography, MRI is noninvasive and can provide images in multiple planes, but no interference occurs from skeletal, fatty, or gas-filled structures, and imaging of deep pelvic structures does not require a full bladder. The woman is placed on a table in the supine position and moved into the bore of the main magnet, which is similar in appearance to a CT scanner. Depending on the reason for the study, the procedure may take from 20 to 60 minutes, during which time the woman must be perfectly still except for short respites.

Postpartum women experience an increased risk for urinary tract infection. A prevention measure the nurse could teach the postpartum woman would be to: a. acidify the urine by drinking 3 glasses of OJ each day. b. maintain a fluid intake of 1-2 L/day c. Empty bladder every 4 hours throughout the day d. perform perineal care on a regular basis

D. Keeping the perineum clean will help prevent a UTI.

Nurses should be aware of the strengths and limitations of various biochemical assessments during pregnancy, including that: A. chorionic villus sampling (CVS) is becoming more popular because it provides early diagnosis. B. screening for maternal serum alpha-fetoprotein (MSAFP) levels is recommended only for women at risk for neural tube defects. C. percutaneous umbilical blood sampling (PUBS) is one of the quad-screen tests for Down syndrome. D. MSAFP is a screening tool only; it identifies candidates for more definitive procedures.

D. MSAFP is a screening tool only; it identifies candidates for more definitive procedures. CVS does provide a rapid result, but it is declining in popularity because of advances in noninvasive screening techniques. MSAFP screening is recommended for all pregnant women. MSAFP, not PUBS, is part of the quad-screen tests for Down syndrome. This is correct. MSAFP is a screening tool, not a diagnostic tool. Further diagnostic testing is indicated after an abnormal MSAFP.

A Native American woman gave birth to a baby girl 12 hours ago. The nurse notes that the woman keeps her baby in the bassinet except for feeding and states that she will wait until she gets home to begin breastfeeding. The nurse recognizes that this behavior is most likely a reflection of: a. delayed attachment b. embarrassment c. disappointment that the baby is a girl. d. a belief that babies should not be fed colostrum

D. Native Americans often use cradle boards and avoid handling their newborn often; they believe that the infant should not be fed colostrum.

The nurse is caring for an infant who is suspected to have neonatal sepsis. Which neonatal risk factor for an infant with suspected neonatal sepsis would the nurse expect to observe? a. large for gestational age (LGA) and an infant of a diabetic mother b. small for gestational age (SGA) and intrauterine growth restriction c. singleton gestation and female d. multiple gestation and low birth weight

D. Neonatal risk factors inculd multiple gestation and low birth rate

The role of the pediatric nurse is influenced by trends in health care. The greatest trend in health care is: a. primary focus on treatment of disease or disability b. national health care planning on a distributive or episodic basis c. accountability to professional codes and international standards d. shift of focus to prevention of illness and maintenance of health

D. Prevention is the current focus of health care, one in which nursing plays a major role.

A pregnant couple has formulated a birth plan and is reviewing it with the nurse at an expectant parent's class. Which aspect of their birth plan would be considered unrealistic and require further discussion with the nurse? a. My husband and I have agreed that my sister will be my coach since he becomes anxious with regard to medical procedures and blood. He will be nearby and check on me every so often to make sure everything is OK> b. We plan to use the techniques taught in the Lamaze classes to reduce the pain experienced during labor. c. We want the labor and birth to take place in a birthing room. My husband will come in the minute the baby is born. d. we do not want the fetal monitor used during labor since it will interfere with movement and doing efflurage.

D. Since monitoring is essential to assess fetal well-being, it is not a factor that can be determined by the couple. The nurse should fully explain its importance. The option for intermittent electronic monitoring could be explored if this is a low risk pregnancy and as long as the labor is progressing normally.

Four hours after a difficult labor and birth, a primiparous woman refuses to feed her baby, stating that she is too tired and just wants to sleep. The nurse should: a. tell the woman she can rest after she feeds the baby b. recognize this as a behavior of the taking hold stage c. record the behavior as ineffective maternal newborn attachment d. take the baby back to the nursery, reassuring the woman that her rest is a priority at this time

D. The behavior described is typical of this stage and not a reflection of ineffective attachment unless it persists. Mothers need to reestablish their own well being to effectively care for their baby.

Chaste tree fruit

Decreases prolactin levels

In planning the care of a 30 year old woman with pregestational diabetes, the nurse recognizes that the most important factor affecting pregnancy outcome would be which of the following?

Degree of glycemic control during pregnancy

The nurse must watch for what common complications in a patient who has undergone a transverse rectus abdominis myocutaneous (TRAM) flap?

Delayed wound healing and hematoma

Which description of the phases of the second stage of labor is accurate:

Descent phase: Significant increase in contractions, Ferguson reflex activated, average duration varies

Nurses, certified nurse-midwives, and other advanced practice nurses have the knowledge and expertise to assist women in making informed choices regarding contraception. A multidisciplinary approach should ensure that the woman's social, cultural, and interpersonal needs are met. Which action should the nurse take first when meeting with a new client to discuss contraception?

Determine the woman's level of knowledge about contraception and commitment to any particular method.

When providing care for a pregnant woman, the nurse should be aware that one of the most frequently reported maternal medical risk factors is:

Diabetes mellitus.

In their role of implementing a plan of care for infertile couples, nurses should:

Do all of the above plus be knowledgeable about potential drug and surgical remedies.

To provide culturally sensitive care to an Asian-American family, which of the following questions should the nurse include during the assessment interview?

Do you prefer hot or cold beverages?

While assessing skin of 24 h old newborn, the nurse notes a pink popular rash with vesicles superimposed on the thorax, back and abdomen, the nurse should

Document the finding as erythema toxicum

While evaluating an external monitor tracing of a woman in active labor who is being induced, the nurse notes that the fetal heart rate (FHR) begins to decelerate at the onset of several contractions and returns to baseline before each contraction ends. The nurse should:

Document the finding in the patients records

A benign breast condition that includes dilation and inflammation of the collecting ducts is called:

Ductal ectasia.

With regard to follow-up visits for women receiving prenatal care, nurses should be aware that:

During the abdominal examination, the nurse should be alert for supine hypotension

With regard to the follow-up visits for women receiving prenatal care, which of the following statements is accurate?

During the abdominal examinations, the nurse should be alert to supine hypotension

A woman in week 34 of pregnancy reports that she is very uncomfortable because of heartburn. Which of the following should the nurse suggest to the woman?

Eat five small meals daily

Which of the following statements is accurate for a pregnant woman experiencing nausea and vomiting?

Eat small, frequent meals (every 2-3 hours)

According to QSEN, which of the following is not one of the six competencies all pre-licensure nursing students should learn while in school?

Effectiveness

When managing care of a woman in second stage of labor, the nurse used various measures to enhance the progress of fetal descent. Which measure would you include?

Encouraging the woman to try various upright positions, including squatting and standing

A woman complains of severe abdominal and pelvic pain around the time of menstruation that has gotten worse over the last 5 years. She also complains of pain during intercourse and has tried unsuccessfully to get pregnant for the past 18 months. These symptoms are most likely related to:

Endometriosis.

Supine hypotension/vena cava syndrome/aortocaval compression

Enlarging uterus puts pressure on vena cava if in supine which interferes with returning blood flow and produces a marked decrease in BP with dizziness, pallor and clamminess.

A 26-week gestation woman is diagnosed with severe preeclampsia with HELLP syndrome. The nurse will assess for which of the following signs/symptoms

Epigastric pain

In which culture is the father more likely to be expected to participate in the labor and delivery?

European-American

A 23-year-old African-American woman is pregnant with her first child. Based on the statistics for infant mortality, which plan is most important for the nurse to implement? Perform a nutrition assessment. Refer the woman to a social worker. Advise the woman to see an obstetrician, not a midwife. Explain to the woman the importance of keeping her prenatal care appointments.

Explain to the woman the importance of keeping her prenatal care appointments Nutritional status is an important modifiable risk factor, but it is not the most important action a nurse should take in this situation. The client may need assistance from a social worker at some time during her pregnancy, but this is not the most important aspect the nurse should address at this time. If the woman has identifiable high risk problems, her health care may need to be provided by a physician. However, it cannot be assumed that all African-American women have high risk issues. Additionally, this is not the most important aspect on which the nurse should focus at this time, and it is not appropriate for a nurse to advise or manage the type of care a client is to receive. Consistent prenatal care is the best method of preventing or controlling risk factors associated with infant mortality.

The hospital risk management nurse is providing annual in-service training at the obstetrical unit staff meeting. The risk management nurse should discuss which conditions included on the National Quality Forum list that pertain to maternity nursing? (Select all that apply) Falls and trauma Decreased incidence of urinary tract infections with catheter use Air embolism Foreign objects retained after surgery Blood incompatibility

Falls and trauma,Air embolism, Foreign objects retained after surgery,Blood incompatibility Catheter use should be minimized to decreased urinary tract infections. Five of the conditions are also on the National Quality Forum list. Conditions that might pertain to maternity nursing include a foreign object retained after surgery, air embolism, blood incompatibility, falls and trauma, and catheter-associated urinary tract infections. Almost 1300 U.S. hospitals waive (do not bill for) costs associated with serious reportable events (O'Reilly, 2008).

Which of the following statements regarding women with hyperemesis gravidarum is accurate?

Have vomiting severe and persistent enough to cause weight loss, dehydration, and electrolyte imbalance.

The nurse is preparing for a home visit to complete a newborn wellness checkup. The neighborhood has a reputation for being dangerous. Identify which precautions the nurse should take to ensure her safety (Select all that apply).

Having access to a cell phone at all times. Carrying an extra set of car keys. Avoiding groups of strangers hanging out in doorways.

The nurse-midwife is teaching a group of women who are pregnant, including instruction on Kegel exercises. Which statement by a participant would indicate a correct understanding of the instruction? I will only see results if I perform 100 Kegel exercises each day. I should hold the Kegel exercise contraction for 10 seconds and rest for 10 seconds between exercises. I should only perform Kegel exercises in the sitting position. I will perform daily Kegel exercises during the last trimester of my pregnancy to achieve the best results.

I should hold the Kegel exercise contraction for 10 seconds and rest for 10 seconds between exercises Guidelines suggest that women perform between 30 and 80 Kegel exercises. The correct technique for Kegel exercises is to hold the contraction for at least 10 seconds and rest for 10 seconds in between so the muscles can have time to recover and each contraction can be as strong as the woman can make it. Kegel exercises are best performed in a supine position with the knees bent. Kegel exercises should be performed throughout the pregnancy to achieve the best results.

In assessing knowledge of pregestational woman with type I diabetes concerning changing insulin needs during pregnancy the nurse recognizes that further teaching is warranted when the woman states

I will need to increase my insulin dosage during the first 3 months of pregnancy

With regard to use of intrauterine devices (IUD) nurse should be aware

IUDs containing cooper can provide an emergency contraception option if inserted within a few days of unprotected intercourse

A patient has been diagnoses with gestational diabetes! I will never be able to give myself shots!" Which response by the nurse is most appropriate?

If you exercise and follow your diet your will probably not need insulin

A married couple lives in a single-family house with their newborn daughter and the husband's daughter from a previous marriage. Which of the following family forms best describes this family?

Married-blended family

• A 26 year old primigravida comes to clinic for regular prenatal visit at 12 weeks appears thin nervous, reports eats well balanced diet, although her weight is 5 pounds less than it was at her last visit, lab studies confirm she has a hyperthyroid condition, based on data from the nurse formulate a plan of care, what nursing diagnosis is most appropriate for the woman at this time

Imbalanced nutrition: less than body

From the nurse's perspective, what measure should be the focus of the health care system to reduce the rate of infant mortality further? Implementing programs to ensure women's early participation in ongoing prenatal care Increasing the length of stay in a hospital after vaginal birth from 2 to 3 days Expanding the number of neonatal intensive care units (NICUs) Mandating that all pregnant women receive care from an obstetrician

Implementing programs to ensure women's early participation in ongoing prenatal care Early prenatal care allows for early diagnosis and appropriate interventions to reduce the rate of infant mortality. An increased length of stay has been shown to foster improved self-care and parental education. However, it does not prevent the incidence of leading causes of infant mortality rates, such as low birth weight. Early prevention and diagnosis reduce the rate of infant mortality. NICUs offer care to high risk infants after they are born. Expanding the number of NICUs would offer better access for high risk care, but this is not the primary focus for further reduction of infant mortality rates. A mandate that all pregnant women receive obstetric care would be nearly impossible to enforce. Furthermore, certified nurse-midwives (CNMs) have demonstrated reliable, safe care for pregnant women.

What has had the greatest impact on reducing infant mortality in the United States? Improvements in perinatal care Decreased incidence of congenital abnormalities Better maternal nutrition Improved funding for health care

Improvements in perinatal care The improvements in perinatal care, particularly care of the mother-baby dyad before birth, have had the greatest impact. There has been a decrease in some congenital anomalies, such as spina bifida, but this has not had the greatest impact. This has had a positive influence but not the greatest overall impact. Changes in funding have not had the greatest impact.

Cardiovascular system changes in pregnancy, which finding would be normal of a woman in her second trimester

Increased pulse rate

Cardiovascular system changes occur during pregnancy. Which of the following findings would be considered normal for a woman in her second trimester?

Increased pulse rate

If constipation is a problem for a woman during pregnancy, which of the following measures would be best to recommend?

Increasing fluid intake

While working in the prenatal clinic, you care for a very diverse group of patients. When planning interventions for these families, you realize that acceptance of the interventions will be most influenced by:

Individual beliefs.

Which of the following FHR findings would concern the nurse during labor?

Late decelerations

2 days ago a woman gave birth to a full term infant, Last night she awakened several times to urinate and notes that her gown and bedding were wet from profuse diaphoresis. One mechanism for the diaphoresis and diuresis that this woman is experiencing during the early postpartum periods is

Loss of increased blood volume associated with pregnancy

The nurse providing education regarding breast care should explain to the woman that fibrocystic changes in breasts are:

Lumpiness with pain and tenderness found in varying degrees in the breast tissue of healthy women during menstrual cycles.

To ensure patient safety, the practicing nurse must have knowledge of the current Joint Commission's "Do Not Use" list of abbreviations. Which of the following is acceptable for use?

MSO4 or MgSO4

Screening at 24 weeks gestation reveals that a pregnant woman has gestational diabetes. In planning her care, the nurse and the woman mutually agree that an expected outcome is to prevent injury to the fetus. What is the fetus at greatest risk for developing?

Macrosomia

A woman has come to the clinic for preconception counseling because she wants to start trying to get pregnant in 3 months. Which of the following is the best advice to give to the woman?

Make sure you include folic acid into your diet

The nurse should include questions regarding sexuality when gathering data for a reproductive health history of a female patient. Which principle should guide the nurse when interviewing the patient? An in-depth exploration of specific sexual practices should be included for every patient. Sexual histories are optional if the patient is not currently sexually active. Misconceptions and inaccurate information expressed by the patient should be corrected promptly. Questions regarding the patient's sexual relationship are unnecessary if she is monogamous.

Misconceptions and inaccurate information expressed by the patient should be corrected promptly. More in-depth assessments are required if the patient is sexually active or if problems or concerns are raised during general questions. Sexuality should be included on every reproductive health history whether or not the patient is sexually active. To obtain the most accurate reproductive health history, the nurse needs to correct misconceptions and inaccurate information. The relationship and sexual partner should be discussed even if the patient is monogamous.

An African-American woman noticed some bruises on her newborn girl's buttocks. She asked the nurse who spanked her daughter the nurse explains that it is

Mongolian Spot

A macrosomic infant is born after a difficult, forcep-assisted delivery. After stabilization, the infant is weighed at 9 lb 6 oz. What is the nurse's most appropriate action?

Monitor blood glucose levels frequently and observe closely for signs of hypoglycemia

A women in labor has just received an epidural block. What is the most important nursing intervention?

Monitor the maternal BP for possible hypotension

A woman in labor has just received spinal anesthesia. The most important nursing intervention is to:

Monitor the maternal blood pressure for possible hypotension

In understanding and guiding a woman through her acceptance of pregnancy, which of the following statements is accurate?

Mood swings most likely are the result of worries about finances and a changed lifestyle, as well as profound hormonal changes

While evaluating reflex of newborn, nurse notes with loud noise the newborn systemically abducts and extends his arms, his fingers fan out and forma a C with the thumb and forefinger, and he has a slight tremor, the nurse would document positive

Moro reflex

While evaluating newborn reflexes, the nurse notes that with loud noise, the baby abducts his arms, his fingers fan out and form a C. Which reflex should be documented and what action should be taken?

Moro reflex/no further action necessary

A nurse suspects that a client has been physically abused. The woman refuses to report the abuse to the police. Which statement by the client suggests to the nurse that the relationship may be in the "honeymoon phase'

My partner said that he will never hurt me again

Several changes in the integumentary system that appear during pregnancy disappear after birth, although not always completely. What change is almost certain to be completely reversed?

Nail brittleness disappears

A laboring woman received Demerol, an opioid agonist, IV 90 minutes before she gave birth. Which of the following meds should be available to reduce postnatal effects of Demerol to the neonate?

Naloxone (Narcan)

Which of the following was not discussed as a general category of errors and hazards in healthcare?

Near miss

Type I diabetic is being seen for preconception counseling. The nurse should emphasize that during the first trimester the woman may experience which of the following

Need for less insulin that she normally injects

A woman inquires about herbal alternative methods for improving fertility. Which statement by the nurse is the most appropriate when instructing the woman in which herbal preparations to avoid while trying to conceive? "You should avoid nettle leaf, dong quai, and vitamin E while you are trying to get pregnant." "You may want to avoid licorice root, lavender, fennel, sage, and thyme while you are trying to conceive." "You should not take anything with vitamin E, calcium, or magnesium. They will make you infertile." "Herbs have no bearing on fertility."

Nettle leaf, dong quai, and vitamin E promote fertility. Herbs that a woman should avoid while trying to conceive include licorice root, yarrow, wormwood, ephedra, fennel, goldenseal, lavender, juniper, flaxseed, pennyroyal, passionflower, wild cherry, cascara, sage, thyme, and periwinkle. Vitamin E, calcium, and magnesium may promote fertility and conception. Although most herbal remedies have not been proven clinically to promote fertility, women should avoid the following herbs while trying to conceive: licorice root, yarrow, wormwood, ephedra, fennel, goldenseal, lavender, juniper, flaxseed, pennyroyal, passionflower, wild cherry, cascara, sage, thyme, and periwinkle.

Recent trends in childbirth practice indicate that:

Newborn infants remain with the mother and are encouraged to breastfeed.

A woman arrives at the clinic for a pregnancy test, Last Menstrual period was Feb 14, 2012, her expected date of birth would be

Nov 21

A woman arrives at the clinic for a pregnancy test. Her LMP was February 14, 2007. What is her EDB?

November 21, 2007

A traditional family structure in which male and female partners and their children live as an independent unit is known as a(n):

Nuclear family.

Which statement about cultural competence is not accurate?

Nursing care is delivered in the context of the client's culture but not in the context of the nurse's culture.

A pregnant woman, 36 weeks gestation with DM 1 has just had a BPP (biophysical profile). Which of the following results should be reported?

One fetal heart acceleration in 20 mins

Nursing care and assessments for Rupture of Membranes

Ongoing observation for signs and symptoms of infection, active labor, and nonreassuring fetal status.

Which of the following hormones would be administered for the stimulation of uterine contractions? 1. Estrogen 2. Fetal cortisol 3. Oxytocin 4. Progesterone

Oxytocin is the hormone responsible for stimulating UCs. Pitocin, the synthetic form, may be given to clients who are past their due date. Progesterone has a relaxation effect on the uterus Fetal cortisol is believed to slow the production of progesterone by the placenta although estrogen has a role in UC, it isnt given in a synthetic form to help UC

Medications used to manage postpartum hemorrhage include which of the following?

Pitocin, Methergine, Carboprost

The role of the professional nurse caring for childbearing families has evolved to emphasize:

Practice using an evidence-based approach.

With regard to preeclampsia and eclampsia, nurses should be aware which of the following statements is accurate?

Preeclampsia results in decreased function of 40-60% in such organs such as the placenta, kidneys, liver, and brain as a result of vasospasm which impedes blood flow

When evaluating a patient whose primary complaint is amenorrhea, the nurse must be aware that lack of menstruation is most often the result of:

Pregnancy.

Which statement concerning cyclic perimenstrual pain and discomfort (CPPD) is accurate?

Premenstrual syndrome is a complex, poorly understood condition that may include any of a hundred symptoms.

Screening questions for alcohol and drug abuse should be included in the overall assessment during the first prenatal visit for all women the 4 P's plus is a screening tool designed specifically or identify when there is a need of a more in depth assessment the 4 Ps include all except:

Present

The factors that affect the process of labor and birth, the five P's, include all except?

Pressure

With regard to primary and secondary powers, the maternity nurse should know that:

Primary powers are responsible for effacement and dilation of the cervix

Practices such as providing recommended immunizations, infant car seats and pre conceptual counseling are examples of what kind of care?

Primary preventive care

A PHYSICIAN ORDERED Pitocin for induction of 4 women, in which of the following situations should the nurse refuse to comply with the order

Primigravida with a transverse lie

What is a woman who has completed one pregnancy with a fetus (or fetuses) reaching the state of fetal viability called?

Primipara

Using the family stress theory as an intervention approach for working with families experiencing parenting, the nurse can help the family change internal context factors. These include:

The family's perception of the event.

A pregnant woman who abuses cocaine admits to exchanging sex for her drug habit. This behavior places her at a greater risk for:

Sexually transmitted diseases

While interviewing a 31-year-old woman before her routine gynecologic examination, the nurse collects data about the client's recent menstrual cycles. The nurse should collect additional information with which statement?

She describes her flow as very heavy.

A woman gave birth vaginally to a 9-pound, 12 ounce girl yesterday. Her primary health care provider has written orders for perineal ice packs, use of a sitz bath tid, and a stool softener. What information is most closely correlated with these orders?

She has an episiotomy

A primigravida at 40 weeks is having contractions every 1.5-2 minutes and says they are very painful. Her cervix is 2 cm and has not changed in 3 hours. She wants an epidural. What is the likely status of her labor?

She is exhibiting hypertonic uterine dysfunction

A nurse counseling a client with endometriosis understands which statements regarding the management of endometriosis is accurate? (Select all that apply) Bone loss from hypoestrogenism is not reversible. Side effects from the steroid danazol include masculinizing traits. Surgical intervention often is needed for severe or acute symptoms. Women without pain and who do not want to become pregnant need no treatment. Women with mild pain who may want a future pregnancy may take nonsteroidal antiinflammatory drugs (NSAIDs).

Side effects from the steroid danazol include masculinizing traits, Surgical intervention often is needed for severe or acute symptoms,Women without pain and who do not want to become pregnant need no treatment .Bone loss is mostly reversible within 12 to 18 months after the medication is stopped. Such masculinizing traits as hirsutism, a deepening voice, and weight gain occur with danazol but are reversible. Surgical intervention often is needed when symptoms are incapacitating. The type of surgery is influenced by the woman's age and desire to have children. Treatment is not needed for women without pain or the desire to have children. In women with mild pain who may desire a future pregnancy, treatment may be limited to use of NSAIDs during menstruation.

A woman is at 14 weeks gestation. The nurse would expect to palpate the fundus at which of the following levels?

Slightly above the symphysis pubis

A man smokes 2 packs of cigarettes a day, he wants to know if smoking is contributing to the difficulty he and his wife are having getting pregnant, the nurse's most appropriate response is

Smoking can reduce the quality of your sperm

Hegar's sign

Softening of the isthmus of the uterus, the area between the cervix and the body of the uterus, this occurs at 6 to 8 weeks

The most effective and least expensive treatment of puerperal infection is prevention. Which is important in this strategy?

Strict asceptic technique, including handwashing, by all health care providers

A woman will be taking oral contraceptive using s 28-day pack. The nurse should advice this women to protect against pregnancy by

Taking one pill at the same time every day

Which of the following time-based descriptions of a state of development in pregnancy is accurate?

Term - pregnancy from the beginning of week 38 gestation to the end of week 42

Menstruation is periodic uterine bleeding:

That is regulated by ovarian hormones.

The parents of a newborn ask the nurse how much the newborn can see the parents specifically want to know what type of visual stimuli they should provide for their newborn the nurse responds to the parents by telling them

The infant can track their parent's eyes and distinguish patterns, they prefer complex patterns

During a prenatal intake interview, the client informs the nurse that she would prefer a midwife to provide her care during pregnancy and deliver her infant. What information would be most appropriate for the nurse to share with this patient?

The client will receive fewer interventions during the birth process.

The transition phase during which ovarian function and hormone production decline is called:

The climacteric.

Which statement would the nurse recognize that a woman is in true labor?

The contractions in my uterus are getting stronger and closer together.

With regard to the care management of preterm labor, nurses should be aware

The diagnosis of preterm labor is based on gestational age, uterine activity, and progressive cervical change

With regard to the care management of preterm labor, nurses should be aware the which of the following is true?

The diagnosis of preterm labor is based on gestational age, uterine activity, and progressive cervical changes

With regard to the noncontraceptive medical effects of combined oral contraceptive pills (COCs), nurses should be aware that:

The effectiveness of COCs can be altered by some over-the-counter medications and herbal supplements.

The patient's family is important to the maternity nurse because:

The family culture and structure will influence nursing care decisions.

A patient at 24 weeks of gestation says she has a glass of wine with dinner every evening. The nurse will counsel her to eliminate all alcohol intake because:

The fetus is placed at risk for altered brain growth.

Sally comes in for her first prenatal examination. This is her first child. She asks the nurse, "How does my baby get air inside my uterus?" Which of the following is the correct response?

The placenta provides oxygen to the baby and excretes carbon dioxide into your blood stream

With regard to the newborn's developing cardiovascular system, nurses should be aware that:

The point of maximal impulse (PMI) often is visible on the chest wall

Placental abruption

The separation of the placenta (the organ that nourishes the fetus) from its attachment to the uterus wall before the baby is delivered.

Mr. and Mrs. Roberts arrive for the birth of their first child. You apply the external fetal monitor and Mr. Roberts asks you to explain the printing on the graph. He wants to know what the baby's HR should be. What is the best response?

The top line graphs the baby's heart rate. Generally, the heart rate is between 110-160. The heart rate will fluctuate in response to what is happening during labor.

Secondary powers of labor

The use of abdominal muscles to push during the second stage of labor

A woman who is 6 months pregnant has sought medical attention, saying she fell down the stairs. What scenario would cause an emergency department nurse to suspect that the woman has been a victim of intimate partner violence (IPV)?

The woman has injuries on various parts of her body that are in different stages of healing.

Which is correct concerning the performance of a Papanicolaou (Pap) smear? The woman should not douche, use vaginal medications, or have intercourse for at least 24 hours before the test. It should be performed once a year beginning with the onset of puberty. A lubricant such as Vaseline should be used to ease speculum insertion. The specimen for the Pap smear should be obtained after specimens are collected for cervical infection.

The woman should not douche, use vaginal medications, or have intercourse for at least 24 hours before the test. Women should not douche, use vaginal medications, or have sexual intercourse for 24 hours before the examination so as not to alter the cytology results. Pap smears are performed annually for sexually active women or by age 18, especially if risk factors for cervical cancer or reproductive tract infections are present. Pap smears may be performed every 3 years in low risk women after three negative results on annual examination. Only warm water should be used on the speculum so as not to alter the cytology results. The cytologic specimen should be obtained first.

A woman who is 14 weeks pregnant calls her obstetrician's office with concerns about the development of a mask-like area on her face and a dark vertical line on her lower abdomen. What should the nurse tell the woman?

These are normal changes during pregnancy

A woman's cousin gave birth to an infant with a congenital heart anomaly. The woman asks the nurse when such anomalies occur. Which response by the nurse is most accurate

They occur between the 3rd and 5th weeks of development

A woman is in her seventh month of pregnancy. She has been complaining of nasal congestion and occasional epistaxis. What should the nurse suspect?

This is normal respiratory change in pregnancy caused by elevated levels of estrogen

A woman presents to ED complaining of bleeding and cramping, the initial nursing history is significant for a last menstrual period of 6 weeks ago, on sterile speculum examination the primary care provider finds cervix is closed. The anticipated plan of care for this woman is based on probable diagnosis of which of the following types of spontaneous abortion

Threatened

• A woman who has recently given birth complains of pain and tenderness in her leg. On physical examination, the nurse notices warmth and redness

Thrombopheltbitis

Which of the following statements about multifetal pregnancy is not accurate?

Twin pregnancies come to term with the same frequency as single pregnancies

The U.S. Centers for Disease Control and Prevention (CDC) recommends that HPV be treated with client-applied:

Topical podofilox 0.5% solution or gel.

On the second day postpartum following a C section, at which of the following locations would you expect to palpate the woman's fundus?

Two fingers below the umbilicus

The women's health nurse knows which statements regarding sexual response are accurate? (Select all that apply.) Women and men are more alike than different in their physiologic response to sexual arousal and orgasm. Vasocongestion is the congestion of blood vessels. The orgasmic phase is the final state of the sexual response cycle. Facial grimaces and spasms of hands and feet are often part of arousal. Sexual difficulties should be disregarded in the postpartum period.

Women and men are more alike than different in their physiologic response to sexual arousal and orgasm, Vasocongestion is the congestion of blood vessels, Facial grimaces and spasms of hands and feet are often part of arousal. Men and women are surprisingly alike. Vasocongestion causes vaginal lubrication and engorgement of the genitals. The final state of the sexual response cycle is the resolution phase after orgasm. Arousal is characterized by increased muscular tension (myotonia). Sexual difficulties should be addressed during the postpartum period.

A woman diagnosed with marginal placenta previa gave birth vaginally 15 minutes ago. At the present time, she is at greatest risk for: a. Hemorrhage. b. Infection. c. Urinary retention. d. Thrombophlebitis.

a. Hemorrhage. Correct -Hemorrhage is the most immediate risk because the lower uterine segment has limited ability to contract to reduce blood loss. Infection is a risk because of the location of the placental attachment site but is not a priority concern at this time. There is no greater risk for urinary retention or thrombophlebitis than with a normally implanted placenta.

A couple seeking infertility counseling expresses their excitement by stating, "Now at last we can become pregnant. We are so glad to get this process started!" The response by the nurse should be based on the knowledge that: a. Infertility care does not always lead to a successful pregnancy. b. A successful pregnancy chance can be increased with intervention. c. Infertility counseling just looks at the cause of the infertility, not its treatment. d. A pregnancy can occur, but there is no guarantee about the successful completion of the pregnancy.

a. Infertility care does not always lead to a successful pregnancy. Correct -Because some factors contributing to infertility remain unknown, treatment of an identified problem does not always lead to a successful pregnancy.

A woman is admitted with a diagnosis of missed abortion. After taking her blood pressure, the nurse notices petechiae on the woman's arm where the cuff was located. The nurse's next action should be to: a. Notify the health care provider. b. Massage the arm. c. Monitor her blood pressure closely. d. Determine her temperature.

a. Notify the health care provider. Correct - One major complication of missed abortion is disseminated intravascular coagulation (DIC). This may be manifested by small areas of hemorrhaging. The health care provider needs to be notified.

A woman calls the clinic early on a Monday morning stating that she needs a prescription for emergency contraception. On questioning by the nurse, the woman states that her boyfriend's condom was displaced during intercourse on Saturday night. The nurse should make an appointment for the woman: a. On Monday. b. By Wednesday. c. By Thursday. d. No later than Friday.

a. On Monday. Correct - The effectiveness of emergency contraception is greatest if used within 72 hours of unprotected intercourse.

A pregnant woman at 28 weeks of gestation has been diagnosed with gestational diabetes. The nurse caring for this client should know that: a. Oral hypoglycemic agents are the preferred choice to control an elevated blood sugar level. b. Dietary modifications and insulin are both required for adequate treatment. c. Glucose levels are monitored by testing urine four times a day and at bedtime. d. Dietary management involves distributing nutrient requirements over three meals and two or three snacks.

a. Oral hypoglycemic agents are the preferred choice to control an elevated blood sugar level. -Small frequent meals over a 24-hour period help decrease the risk for hypoglycemia and ketoacidosis. Insulin is the preferred medication to use, if needed, because it does not cross the placenta. Oral hypoglycemic agents can be harmful to the fetus and are less effective than insulin in achieving tight glucose control. In some women, gestational diabetes can be controlled with dietary modifications alone. Blood, not the urine glucose level, is monitored several times a day. Urine is tested for ketone content; results should be negative.

A couple delivered a full-term baby girl 3 years ago. They have been attempting pregnancy for the past 2 years, without success. When taking a history from the couple, which of the following may lead to abnormal sperm numbers and function in the male? a. His job requires him to walk most of the day, with few rest periods. b. He was infected with mumps 2 years ago after the birth of his girl. c. He was diagnosed with hypertension 2 years ago and is under medical treatment. d. His job requires him to be outside about 7 hours out of the 8 hours at work.

b. He was infected with mumps 2 years ago after the birth of his girl. Correct -Acute or chronic illness such as mumps, cirrhosis, and renal failure can impair the number and function of the sperm. Sitting for prolonged periods will elevate the scrotal temperature and impair the numbers and function of the sperm. Antihypertensives may produce abnormal ejaculation but do not interfere with the sperm number and function.

A 20-week-pregnant client attending her first prenatal visit tells the nurse at the maternity clinic that she has had vaginal bleeding and excessive nausea and vomiting for the past 3 days. The nurse assesses her blood pressure at 142/95 mm Hg, pulse 86 bpm, respirations 16 breaths/min. When the nurse helps the client onto the examining table, the abdomen looks larger than normal for a 20-week pregnancy. The nurse is aware that these are signs of: a. Ectopic pregnancy. b. Hydatidiform mole. c. Hyperemesis gravidarum. d. Preeclampsia.

b. Hydatidiform mole. Correct - Signs and symptoms of a hydatidiform mole pregnancy include a uterus that is larger than expected, vaginal bleeding, excessive nausea and vomiting, and early development of preeclampsia.

The nurse is admitting 37-week-pregnant woman with severe preeclampsia. When choosing a room for her, the nurse would put her in the: a. First room near the elevator. b. Last room at the end of the hall, close to the nurses' station. c. Room next to the nursery. d. Room across from the visitors' lounge.

b. Last room at the end of the hall, close to the nurses' station. -The mother needs to be closely monitored. However, she also needs an area that is away from external stimuli and is quiet. All the other rooms would have visitors and noise associated with them.

An ultrasound is done prior to an amniocentesis to: a. Determine fetal age. b. Locate fetal and placental position. c. Determine maternal blood pressure. d. Determine amount of fetal movement.

b. Locate fetal and placental position. Correct - Determining fetal and placental position is important prior to amniocentesis to prevent damage by the needle.

Rh factor incompatibility can occur only in which of the following situations? a. Mother is Rh-positive, father is Rh-negative b. Mother is Rh-negative, father is Rh-positive c. Mother is Rh-negative, father is Rh-negative d. Mother is Rh-positive, father is Rh-positive

b. Mother is Rh-negative, father is Rh-positive - Rh incompatibility during pregnancy is possible only when the mother is Rh-negative and the fetus is Rh-positive. For the fetus to be Rh-positive, the father must be Rh-positive. Rh-negative blood is an autosomal recessive trait and a person must inherit the same gene from both parents.

Semen analysis is a common diagnostic procedure related to infertility. When instructing a male client about this test, the nurse would tell him to: a. Ejaculate into a sterile container. b. Obtain the specimen after a period of abstinence from ejaculation for 2 to 5 days. c. Transport the specimen with the container packed in ice. d. Ensure that the specimen arrives at the laboratory within 30 minutes of ejaculation.

b. Obtain the specimen after a period of abstinence from ejaculation for 2 to 5 days. Correct - The male must ejaculate into a clean container or plastic sheath that does not contain a spermicide. He should avoid exposing the specimen to extremes of temperature, heat or cold. The specimen should be taken to the laboratory within 2 hours of ejaculation.

A 32-week-pregnant woman calls the prenatal clinic complaining of bleeding without pain or contractions. The nurse should: a. Tell her to rest for a couple of hours and call back if it does not stop. b. Tell her to go to the hospital to be evaluated. c. Make her an appointment for the next morning. d. Have her assess fetal movement for 30 minutes.

b. Tell her to go to the hospital to be evaluated. Correct - Signs of placenta previa are painless bleeding after 20 weeks of gestation. Active bleeding can occur; therefore she needs to be evaluated.

A newly diagnosed pregnant woman has diabetes mellitus, type 2. When planning the prenatal care for this woman, the nurse identifies actions based on the knowledge that: a. Insulin needs increase during the first trimester and decrease thereafter. b. The danger of diabetic ketoacidosis is highest during the second and third trimesters. c. Oligohydramnios can occur, leading to fetal distress during labor. d. Maternal blood glucose levels need to be maintained between 125 and 135 mg/dL.

b. The danger of diabetic ketoacidosis is highest during the second and third trimesters. -Maternal insulin requirements are reduced during the first trimester. Higher insulin requirements during the second and third trimesters increase the risk for ketoacidosis. Polyhydramnios is associated with diabetes during pregnancy, increasing the risk for preterm labor and postpartum hemorrhage from an overstretched uterus. Blood glucose levels should be maintained between 70 and 120 mg/dL.

A woman and her female partner have come to the clinic stating that they would like to have a baby. During the interview, the nurse discusses the possibilities with the couple. One possibility in which the woman may conceive and deliver her biologic child is: a. Egg donation. b. Therapeutic insemination. c. Surrogate parenting. d. Ovulation induction.

b. Therapeutic insemination. Correct - Therapeutic insemination may use semen of a donor. This can be used if a woman wants a biologic child without having a relationship with a male partner.

A woman called the clinic desiring to start on medroxyprogesterone acetate (Depo-Provera) for contraception. The nurse should make an appointment for the woman: a. Immediately b. Within 5 days after her next menstrual period. c. Prior to her next menstrual period. d. At the time of ovulation.

b. Within 5 days after her next menstrual period. Correct - The injection is best given within 5 days of the menstrual period. If given later in the cycle, an additional form of contraception should be used for the first week.

To adequately care for laboring women the nurse should know which stage of labor varies the most in length

first

During the first prenatal clinic visit, an enzyme-linked immunosorbent assay (ELISA) for rubella immunity was done in a woman who is 2 months pregnant. The results of the test were 6 international units/mL. During the next visit, the nurse should: a. Administer the rubella vaccine. b. Advise the woman that she is immune. c. Advise the woman to avoid anyone who may have rubella while she is pregnant. d. Advise the woman to receive the vaccine during the last trimester of the pregnancy.

c. Advise the woman that she is immune. Advise the woman to avoid anyone who may have rubella while she is pregnant. -A level of less than 8 international units/mL indicates that a person has no immunity to rubella. Pregnant women who are not immune should not receive the vaccine until the postpartum period. The vaccine poses a risk to the fetus, so it is contraindicated during pregnancy. This woman should be advised not to be exposed to the virus because the rubella virus can cross the placental barrier and infect the fetus at any time during the pregnancy.

A woman comes to the clinic stating that she is going on a hiking trip that will last 2 months. She states, "I have heard there are things that can be done to prevent me from having a period during that time." The nurse's response should be based on the knowledge that: a. There is no healthy way to skip a menstrual period. b. There is a surgery, similar to a dilation and curettage, which can be done prior to the hiking trip. c. Certain oral contraceptive pills may be taking consecutively for 2 months to delay menses. d. There is a type of oral contraceptive pill that may result in an extended menses for 1 month and the next month's will be lighter.

c. Certain oral contraceptive pills may be taking consecutively for 2 months to delay menses. Correct -When women prefer extended cycles, in which menses is delayed for a few days for special occasions or for a longer time, they take two or more oral contraceptive pill packs without taking the placebo pills for several packs or indefinitely.

A woman asks the nurse about the "new vaginal ring everyone is talking about for birth control." When counseling the woman about this method of contraception, the nurse should assess for the woman's: a. Ability to remember to insert the device every morning. b. Feelings about having to insert the device before sexual intercourse. c. Comfort level about self-insertion of the ring every 3 weeks. d. Ability to return to the clinic once a month for reinsertion.

c. Comfort level about self-insertion of the ring every 3 weeks. Correct - The woman must remove the ring after 3 weeks and insert a new ring 1 week later. The woman must be comfortable inserting the device into the vagina.

Which of the following is associated with preexisting diabetes in a pregnant woman, but not in a woman with gestational diabetes? a. Neonatal hypocalcemia b. Neonatal hypoglycemia c. Congenital malformations d. Macrosomia

c. Congenital malformations Correct - Because gestational diabetes develops after the first trimester, the critical period of major fetal organ development, it usually is not associated with an increase in major congenital malformations. Hypocalcemia, hypoglycemia, and macrosomia can occur with preexisting and gestational diabetes.

A woman is being admitted to the outpatient surgical unit for a tubal ligation. She states to the nurse, "I know I have to go through with the surgery since I have already signed all the papers, but I was thinking this morning how wonderful it would be to have another baby." The nurse's next action should be to: a. Inform the woman that the surgery unit has already been prepared and that it would be expensive to cancel the surgery at this time. b. Inform the woman that the surgery can be reversed at a later date if she should change her mind. c. Inform the surgeon of the woman's feelings. d. Document the conversation and continue to prepare the woman for surgery.

c. Inform the surgeon of the woman's feelings. Correct - Sterilization should be considered as a permanent end to fertility because reversal surgery is difficult, expensive, not always successful, and often not covered by insurance. The nurse should act as an advocate for the woman by informing the surgeon that the woman is having second thoughts about the surgery. Even though the consent forms have been signed, the woman does not have to proceed with the surgery.

During a breast exam, the midwife notes that the woman has a transdermal contraceptive patch applied to her breast. The midwife should: a. Document the appropriate use of the patch. b. Question the woman on her satisfaction with the patch. c. Inform the woman that the patch should not be applied to the breast. d. Remove the patch to complete the breast exam.

c. Inform the woman that the patch should not be applied to the breast. Correct - The patch can be applied to the abdomen, buttocks, or upper arm. It should not be applied to the breast area or areas that are rubbed by straps or waistbands.

Nurses working with adolescent women should include questions regarding sexuality when gathering data for a reproductive health history. Which of the following principles should guide the nurse when interviewing the adolescent? a. An in-depth exploration of specific sexual practices should be included for this client. b. Sexual histories are optional. c. Misconceptions and inaccurate information expressed by the adolescent should be corrected promptly. d. Questions regarding the client's sexual relationship are unnecessary if she is monogamous.

c. Misconceptions and inaccurate information expressed by the adolescent should be corrected promptly. Correct -Misinformation and erroneous beliefs cause adolescents to use ineffective methods of contraception or none at all. Opportunities to provide counseling and information must not be missed.

Most preexisting cardiac problems can be well tolerated during pregnancy if the woman has been successfully treated before the pregnancy. One condition may increase her risk for bacterial endocarditis significantly, which would require prophylactic antibiotics throughout the pregnancy, labor, and birth. This cardiac condition is: a. Atrial septal defect. b. Tetralogy of Fallot. c. Mitral valve prolapse. d. Peripartum cardiomyopathy.

c. Mitral valve prolapse. Correct - Mitral valve prolapse is a significant risk factor for bacterial endocarditis, and administering prophylactic antibiotics before labor and during labor and birth is recommended

During labor, a woman suddenly complains of increasing pain, and the electronic monitor shows no uterine activity. The abdomen is boardlike and tender and the fetal heart tones show late decelerations. The nurse should: a. Turn the mother to her side and continue to monitor the fetal heart rate. b. Assess the mother's blood pressure, temperature, pulse, and respirations. c. Notify the health care provider. d. Anticipate that the woman has moved into the second stage of labor.

c. Notify the health care provider. Correct - With abruptio placentae, the uterus may become exceedingly firm and tender. Because of decreased blood flow, the fetus will show signs of hypoxia. An immediate cesarean birth may be necessary; therefore the health care provider should be notified.

A woman with gestational diabetes is at 36 weeks' gestation. On the regular antepartal visit, the woman tells the nurse, "I am so excited. My blood sugars have gone down and I have been able to decrease the amount of insulin I need by about half." The nurse should be aware that this is an indication of a: a. Diabetic in good glycemic control. b. Fetal problem that needs further investigation. c. Placental problem that needs further investigation. d. Maternal pancreas that is increasing its insulin production.

c. Placental problem that needs further investigation. Correct -Insulin needs should increase markedly during the second and third trimesters when placental hormones reach their peak. The placental hormones initiate maternal resistance to the effects of insulin. If the insulin needs decrease, it is a sign that the placental production of the hormones has decreased and the placenta may be failing.

Signs of a threatened abortion are noted in a woman at 8 weeks of gestation. Which of the following is an appropriate management approach for this type of abortion? a. Prepare the woman for a dilation and curettage (D&C) b. Place the woman on bed rest for at least 1 week and reevaluate. c. Prepare the woman for an ultrasound to determine the integrity of the gestational sac. d. Comfort the woman by telling her that if she loses this baby, she can try to get pregnant again in about 1 month.

c. Prepare the woman for an ultrasound to determine the integrity of the gestational sac. Correct -A D&C is not considered until signs of progress to inevitable abortion are noted or the contents expelled are incomplete. Bed rest is not recommended for this woman, just a decrease in activities. Telling the woman she can get pregnant again soon is not a therapeutic response because it discounts the importance of this pregnancy. If the pregnancy is lost, she should be guided through the grieving process.

A pregnant woman at 14 weeks of gestation is admitted to the hospital with a diagnosis of hyperemesis gravidarum. The primary goal of her treatment at this time would be to: a. Rest the gastrointestinal tract by restricting all oral intake for 48 hours. b. Reduce emotional stress by encouraging the woman to discuss her feelings. c. Reverse fluid, electrolyte, and acid-base imbalances that are present. d. Restore the woman's ability to take and retain oral fluid and foods.

c. Reverse fluid, electrolyte, and acid-base imbalances that are present. Correct -Fluid, electrolyte, and acid-base imbalances present the greatest immediate danger to the well-being of the maternal-fetal unit. Options A, B, and D are all components of treatment but do not represent immediate care for patients with hyperemesis gravidarum.

A woman taking an oral contraceptive as her birth control method of choice should notify her health care provider immediately if she notes which of the following? a. Breast tenderness and swelling b. Weight gain c. Swelling and pain in one of her legs d. Mood swings

c. Swelling and pain in one of her legs Correct - Leg pain and swelling (edema) may indicate thrombophlebitis and should be reported immediately. The other choices are all expected side effects of oral contraceptive pills, temporary in nature, and usually subside within a few cycles.

When a nurse is unsure about how to perform a client care procedure, the BEST action would be to: ask another nurse. discuss the procedure with the client's physician. look up the procedure in a nursing textbook. consult the agency procedure manual and follow the guidelines for the procedure.

consult the agency procedure manual and follow the guidelines for the procedure. ' Each nurse is responsible for his or her own practice. Relying on another nurse may not always be safe practice. Each nurse is obligated to follow the standards of care for safe client care delivery. Physicians are responsible for their own client care activity. Nurses may follow safe orders from physicians, but they are also responsible for the activities that they as nurses are to carry out. Information provided in a nursing textbook is basic information for general knowledge and may not reflect the current standard of care or individual state or hospital policies. It is always best to follow the agency's policies and procedures manual when seeking information on correct client procedures. These policies should reflect the current standards of care and state guidelines.

A 65-year-old woman, G6 P6006, is complaining of increasing stress incontinence and pelvic pressure and fullness. Pelvic examination reveals a bulging in the anterior vaginal wall. This woman is most likely experiencing: uterine prolapse. rectocele. cystocele. vesicovaginal fistula.

cystocele Prolapse or downward displacement of the uterus could result in protrusion of the uterus through the vagina. Rectocele would result in herniation of the rectal wall through the posterior vagina. Clinical manifestations would relate to alterations in bowel elimination. This is the classic clinical manifestations of cystocele. A vesicovaginal fistula is an abnormal passage between the bladder and the vagina, resulting in urinary incontinence and excoriation of the vaginal mucosa.

A woman who is 6 weeks postpartum and lactating is being counseled about contraception. She states that she desires to have a type of birth control that she does not have to "think about." She has three children, so it should be effective and she is in a monogamous relationship. She has a history of blood clots. One appropriate choice of birth control that the nurse can recommend is: a. A combination pill. b. Male condoms. c. Tubal ligation. d. An intrauterine device.

d. An intrauterine device. Correct -Once they are inserted, IUDs provide long-term, continuous contraception without the need to take pills, have injections, or do something else before or during intercourse. They are appropriate for many women who cannot use hormonal contraception because of other problems, such as a history of blood clots. They are safe for use during lactation. Tubal ligation should be considered a permanent procedure; the woman did not request that type of contraception.

When obtaining a reproductive health history from a female patient, the nurse should: a. Limit the time spent on exploration of intimate topics. b. Avoid asking questions that might embarrass the patient. c. Use only accepted medical terminology when referring to body parts and functions. d. Explain the purpose for the question asked and how the information will be used.

d. Explain the purpose for the question asked and how the information will be used. Correct - Sufficient time must be spent on gathering relevant data, even if it may be embarrassing for the patient or the nurse or involves intimate topics. Always use terms the patient can understand.

During a health history interview, a woman states that she thinks that she has "bumps" on her labia. She also states that she is not sure how to check herself. The correct response would be to: reassure the woman that the examination will not reveal any problems. explain the process of vulvar self-examination to the woman and reassure her that she will become familiar with normal and abnormal findings during the examination. reassure the woman that "bumps" can be treated. reassure her that most women have "bumps" on their labia.

explain the process of vulvar self-examination to the woman and reassure her that she will become familiar with normal and abnormal findings during the examination. This statement is inappropriate and may be untrue. During assessment and evaluation the responsibility for self-care, health promotion, and enhancement of wellness is emphasized. The pelvic examination provides a good opportunity for the practitioner to emphasize the need for regular vulvar self-examination. Because the nurse is unsure of the cause of this client's discomfort, this comment would be incorrect. This statement is not accurate and should not be used in this situation.

When obtaining a reproductive health history from a female patient, the nurse should: limit the time spent on exploration of intimate topics. avoid asking questions that may embarrass the patient. use only accepted medical terminology when referring to body parts and functions. explain the purpose for the questions asked and how the information will be used.

explain the purpose for the questions asked and how the information will be used. Sufficient time must be spent on gathering relevant data. All questions should be asked, even if it may be embarrassing for the patient or the nurse, or if it involves intimate topics. Always use terms the patient can understand. Explanation of the purpose for the questions asked while obtaining a reproductive health history will help to gather honest and relevant data.

A mother's household consists of her husband, his mother, and another child. She is living in a/an: extended family. single-parent family. married-blended family. trinuclear family.

extended family. An extended family includes blood relatives living with the nuclear family. Both parents and a grandparent are living in this extended family. Married-blended refers to families reconstructed after divorce. Both parents and a grandparent make up an extended family.

Nurse practicing in perinatal setting should promote kangaroo care regardless of infants gestational age, this intervention should

help infants to interact directly with their parents and enhances their temperature regulation

Main role of surfactant in the neonate

helps the lungs remain expanded after the initiation of breathing works by reducing surface tension in the lung allows the lung to remain slightly expanded decreases amount of work required for inspiration

Danger signs of Pregnancy

o Bleeding o Abdominal pain o Visual changes o Hypertension o Temperature > 101° F o Edema in face and legs o Absence or decrease in fetal movement o Gush of fluid from vagina o Dizziness, blurred vision o Persistent vomiting o Muscular irritability or convulsions o Oliguria o Dysuria o Epigastric pain

The nurse providing care for the laboring woman should understand that amnioinfusion is used to treat:

o Variable decelerations- caused by cord pressure, used to help when there is no amniotic fluid to relieve that pressure

On the 2nd PP day, a client complains that shes urinating more than when she was pregnant. Which is the primary cause of increased urinary output post delivery? 1. postpartum diuresis

occurs as the body starts to reduce the extracellular fluid volume that increased during preg renal plasma flow and GFR also increase slightly until approx 1 week PP Renal malfunctioning is more likely to decrease urinary output, not increase it increase PP fluid intake and breast feeding arent major causes of PP diuresis

When providing care to a young single woman just diagnosed with acute pelvic inflammatory disease, the nurse should: point out that inappropriate sexual behavior caused the infection. position the woman in a semi-Fowler position. explain to the woman that infertility is a likely outcome of this type of infection. tell her that antibiotics need to be taken until pelvic pain is relieved.

position the woman in a semi-Fowler position Although sexual behavior may have contributed to the infection, the nurse must discuss these practices in a nonjudgmental manner and provide information about prevention measures. The position of comfort is the semi-Fowler position. In addition, the foot of the bed could be elevated to keep the uterus in a dependent position and reduce discomfort. Until treatment is complete and healing has occurred, the outcome is unknown and should not be suggested. The nurse should emphasize that medication must be continued until follow-up assessment indicates that the infection has been treated successfully.

A new father wants to know what medication was put into his infants eyes and why it was needed. The nurse explains to the father that the purpose of the ophthalmic ointment is to

prevent gonorrhea and chlamydia infection of the infant's eyes potentially from the birth canal

In planning for home care of a woman with preterm labor, the nurse needs to address what concern:

prolonged bed rest may cause negative physiological effects

Healthy People 2020 goals include (select all that apply): promoting quality of life. promoting healthy behaviors in middle adulthood. attaining high-quality, longer lives. eliminating health disparities. creating social and physical environments that promote health.

promoting quality of life,attaining high-quality, longer lives, eliminating health disparities.,creating social and physical environments that promote health. Healthy People 2020 promotes healthy behaviors across all life stages. Healthy People provides science-based 10-year national objectives for improving the health of all Americans. It has four overarching goals: (1) attaining high-quality, longer lives free of preventable disease, disability, injury, and premature death; (2) achieving health equity, eliminating disparities, and improving the health of all groups; (3) creating social and physical environments that promote good health for all; and (4) promoting quality of life, healthy development, and healthy behaviors across all life stages. The goals of Healthy People 2020 are based on assessments of major risks to health and wellness, changes in public health priorities, and issues related to the health preparedness and prevention of our nation.

Evidence-based practice is best described as: gathering evidence of mortality and morbidity in children. meeting physical and psychosocial needs of the family in all areas of practice. using a professional code of ethics as a means for professional self-regulation. providing care based on evidence gained through research and clinical trials.

providing care based on evidence gained through research and clinical trials This will assist the nurse in determining areas of concern and potential involvement. It is not possible to meet all needs of the patient and family in all areas of practice. The nurse is an advocate for the family. This is part of professional role and licensure. Evidence-based practice is providing care based on evidence gained through research and clinical trials. Practitioners must use the best available information on which to base their interventions.

The nurse should refer the patient for further testing if she noted this on inspection of the breasts of a 55-year-old woman: left breast slightly smaller than right breast. eversion (elevation) of both nipples. bilateral symmetry of venous network, which is faintly visible. small dimple located in the upper outer quadrant of the right breast.

small dimple located in the upper outer quadrant of the right breast. In many women, one breast is smaller than the other. Eversion of both nipples is a normal finding. Faintly visible venous network is a normal finding. A small dimple is an abnormal finding and should be further evaluated.

A 42-year-old woman asks the nurse about mammograms, now that she is "getting older." The nurse should tell her that: the American Cancer Society recommends mammograms every 1 to 2 years for women ages 40 to 49. the best time to perform a mammogram is just before a menstrual period. regular mammograms reduce the need to perform breast self-examination (BSE). mammograms can confirm the diagnosis for breast cancer.

the American Cancer Society recommends mammograms every 1 to 2 years for women ages 40 to 49. The current guidelines indicate that a mammogram should be performed every 1 to 2 years on women between the ages of 40 and 49. Mammograms are best performed beginning at about 1 week after menstruation when the breasts are the least tender. Mammograms are not a substitute for BSE, which should still be performed every month. A biopsy of cells from suspicious lesions is required to confirm a diagnosis of cancer.

When nurses help their expectant mother assess the daily fetal movement counts, they should be aware that:

the fetal alarm signal should go off when the fetal movements stop entirely for 12 hours

What marks the narrowest diameter through which the fetus must pass?

the ischial spines (zero station) -if the presenting part is higher than the ischial spines, a negative number is assigned

Evaporation Heat Loss

the loss of heat that occurs when a liquid is converted to a vapor

Referred pain occurs when

the pain that originates in the uterus radiates into the abdomen

Fetal Presenting Part

the position of the fetus that is felt through the cervix on vaginal examination determines the presentation. Can be cephalic, breech, or shoulder

To help patients manage discomfort and pain during labor, nurses should be aware

the predominant pain of the first stage of labor is the visceral pain located in the lower portion of the abdomen- this pain comes from cervical changes, distention of the lower uterine segment, and uterine ischemia

A 26-year-old woman is considering Depo-Provera as the form of contraception that is best for her since she does not like to worry about taking a pill every day. To assist this woman with decision making concerning this method of contraception, the nurse would tell her that Depo-Provera: is a combination of progesterone and estrogen. is a small adhesive hormonal birth control patch that is applied weekly. thickens and decreases cervical mucus, thereby inhibiting sperm penetration and ovulation. has an effectiveness rate in preventing pregnancy of 99% when used correctly.

thickens and decreases cervical mucus, thereby inhibiting sperm penetration and ovulation. The male must ejaculate into a clean container or a plastic sheath that does not contain a spermicide. An ejaculated sample should be obtained after a period of abstinence to get the best results. He should avoid exposing the specimen to extremes of temperature, either heat or cold. The specimen should be taken to the laboratory within 2 hours of ejaculation.

A woman has requested an epidural for her pain. She is 5 cm dilated and 100% effaced. The baby is in a vertex position and is engaged. The nurse increases the woman's IV fluid for a preprocedural bolus. She reviews her lab values and noes her hemoglobin is 12 g/dl hematocrit is 38% and platelets are 67,000 and white

thrombocytopenia


Related study sets

Music History Final - Short Answer

View Set

Theories of Personality Quiz Review 1

View Set

Business Law - Tradition Contract

View Set

Chapter 3-2 Measure of Variation

View Set

SPRING - QGT6 - EQ: How was the judicial system politicized against Mexican-Americans and Latinos? - Source: ZOOT SUIT (Act 2 Scenes 4-6) - chakshiri

View Set